Sei sulla pagina 1di 4476

Q0001:What does it mean for genetic code to be commaless?

Read from a fixed starting point as a continuous sequence of


bases
Q0002:What does it mean for genetic code to be non-
overlapping?
Read from a fixed starting point
Q0003:What does it mean for genetic code to be universal?
Genetic code is conserved throughout evolution
Q0004:What are the properties of the genetic code?
1. Unambiguous;2. Degenerate/redundant;3.
Commaless/nonoverlapping;4. Universal
Q0005:When is genetic code not commaless/nonoverlapping?
In some viruses
Q0006:What are exceptions to universality of genetic code?
1. Mitochondria;2. Archaebacteria;3. Mycoplasma;4. Some
yeasts
Q0007:Name that mutation: Same amino acid; often with a
base change in 3rd position of codon
Silent mutation
Q0008:What kind of mutation is called: silent
Same amino acid; often with a base change in 3rd position of
codon
Q0009:What mutation is masked by tRNA wobble?
Silent mutations
Q0010:Name that mutation: Changed amino acid whose
structure is dissimilar to proper amino acid
Missense mutation (not conservative)
Q0011:Name that mutation: Changed amino acid whose
structure is similar to proper amino acid
Conservative missense mutation
Q0012:What kind of mutation is called: missense
Amino acid is changed. If the structure of the new amino acid
is similar to the original; it is called conservative.
Q0013:Name that mutation: Change resulting in early stop
codon
Nonsense mutation;(Mnemonic: Stop the nonsense!)
Q0014:What kind of mutation is called: nonsense
Change resulting in early stop codon;(Mnemonic: Stop the
nonsense!)
Q0015:Name that mutation: change resulting in misreading of
all nucleotides downstream; usually resulting in a truncated
protein
Frame shift mutation
Q0016:What kind of mutation is called: frameshift
change resulting in misreading of all nucleotides downstream;
usually resulting in a truncated protein
Q0017:Mutations ordered by decreasing severity of damage
1. Nonsense;2. Missense;3. Silent
Q0018:Eukaryotic genome: single/multiple origins of
replication
multiple
Q0019:Prokaryotic genome: single/multiple origins of
replication
single
Q0020:Eukaryotic genome: Trigger for replication
Consensus sequence of AT-rich base pairs
Q0021:Prokaryotic genome: Describe DNA replication
Continuous bidirectional DNA synthesis on leading strand
and discontinuous (Okazaki fragments) on lagging strand
Q0022:Enzyme function: DNA topoisomerases
Create a nick in the helix to relieve supercoils
Q0023:DNA Topoisomerase I: Mechanism
cuts one strand; passes the other through it then reanneals the
cut strand
Q0024:DNA Topoisomerase II: Mechanism
cuts both strands; and passes an unbroken double strand
through it then reanneals the cut strand
Q0025:Enzyme function: Primase
Makes an RNA primer on which DNA polymerase III can
initiate replication
Q0026:DNA polymerase III: Mechanism
1. Adds deoxynucleotides to the 3' end until it reaches primer
of preceding fragment;2. 3' to 5' exonuclease activity
"proofreads" each added nucleotide
Q0027:DNA polymerase III: Which direction does it read?
3' to 5'
Q0028:DNA polymerase III: Which direction does it write?
5' to 3'
Q0029:DNA polymerase III: Which direction does it
proofread?
3' to 5'
Q0030:Enzyme function: DNA polymerase III
Elongates the chain
Q0031:Enzyme function: DNA polymerase I
Degrades RNA primer and fills in the gap with DNA
Q0032:DNA polymerase I: Which direction does it read?
3' to 5'
Q0033:DNA polymerase I: Which direction does it write?
5' to 3'
Q0034:DNA polymerase I: Which direction does it
proofread?
5' to 3'
Q0035:Enzyme function: DNA helicase
Separates the two strands of DNA into single strands
allowing for replication to occur. The position of these
separated strands is called the replication fork.
Q0036:Types of DNA repair
Single stranded;1. Nucleotide excision repair;2. Base excision
repair;3. Mismatch repair;Double stranded;1. Nonhomologous
end joining
Q0037:Nucleotide excision repair: Mechanism
1. Specific endonucleases release the oligonucleotide
containing damaged bases;2. DNA polymerase and ligase fill
and reseal the gap; respectively
Q0038:In what condition is nucleotide excision repair
mutated?
Xeroderma pigmentosa (dry skin with melanoma and other
cancers)
Q0039:Base excision repair: Mechanism
1. Specific glycosylases recognize and remove damaged
bases;2. AP endonuclease cuts DNA at apyrimidinic site;3.
Empty sugar is removed;4. Gap is refilled and resealed
Q0040:Mismatch repair: Mechanism
1. Unmethylated; newly synthesized string is recognized;2.
Mismatched nucleotides are removed;3. Gap is refilled and
resealed
Q0041:In what condition is mismatch excision repair
mutated?
Hereditary Nonpolyposis Colon Cancer
Q0042:Nonhomologous end joining: Mechanism
Brings together two ends of DNA fragments (no requirement
for homology)
Q0043:What is on the 5' end of a nucleotide
Triphosphate
Q0044:What is on the 3' end of a nucleotide
Hydroxyl group
Q0045:True/False: DNA is synthesized 5' to 3'
True
Q0046:True/False: DNA is synthesized 3' to 5'
False
Q0047:True/False: RNA is synthesized 5' to 3'
True
Q0048:True/False: RNA is synthesized 3' to 5'
False
Q0049:True/False: Protein synthesis proceeds 5' to 3'
True
Q0050:True/False: Protein synthesis proceeds 3' to 5'
False
Q0051:Types of RNA and their important qualities
Massive; Rampant; Tiny;mRNA is the largest type;rRNA is
the most abundant type;tRNA is the smallest type
Q0052:What does eukaryotic RNA polymerase I make?
rRNA
Q0053:What does eukaryotic RNA polymerase II make?
mRNA
Q0054:What does eukaryotic RNA polymerase III make?
tRNA
Q0055:Which RNA polymerase makes rRNA?
eukaryotic RNA polymerase I and prokaryotic RNA
polymerase
Q0056:Which RNA polymerase makes mRNA?
eukaryotic RNA polymerase II and prokaryotic RNA
polymerase
Q0057:Which RNA polymerase makes tRNA?
eukaryotic RNA polymerase III and prokaryotic RNA
polymerase
Q0058:True/False: RNA polymerase proofreads.
False
Q0059:True/False: RNA polymerase does not proofread.
True
Q0060:Special points about RNA polymerase II
1. Opens DNA at promoter site;2. Inhibited by alpha-
amanitin
Q0061:What does alpha-amanitin do?
Inhibits RNA polymerase II leading to hepatic necrosis
Q0062:mRNA initiation codons
1. AUG (inAUGurates protein synthesis);2. GUG (rarely)
Q0063:What does the mRNA initiation codon code for?
Methionine in eukaryotes. formyl-methionine in prokaryotes.
Q0064:mRNA stop codons
1. UGA (U Go Away);2. UAA (U Are Away);3. UAG (U
Are Gone)
Q0065:Define promoter of gene expression.
Site where RNA polymerase and multiple other transcription
factors bind to DNA upstream from gene locus
Q0066:What characterizes a promoter of gene expression?
AT-rich upstream sequence with TATA and CAAT boxes
Q0067:What is the result of promoter mutation?
Dramatic decrease in amount of gene transcribed
Q0068:Define enhancer of gene expression.
Stretch of DNA that alters gene expression by binding
transcription factors. May be located close to; far from; or
even within the gene whose expression it regulates.
Q0069:Define operator of gene expression
Site where repressors bind
Q0070:What is alternative splicing?
Rearrangement of exons to make unique proteins
Q0071:What is the sequence of mRNA splicing?
1. Primary transcript combines with snRNP ("snerp") to form
spliceosome;2. Lariat-shaped intermediate is generated;3.
Lariat is released to remove intron precisely and join two
exons
Q0072:Where and when does eukaryotic RNA processing
happen?
In the nucleus after transcription
Q0073:What is the initial RNA transcript called?
heterogeneous nuclear RNA (hnRNA)
Q0074:What are the steps in processing hnRNA to make
mRNA? (Note: This is more than splicing.)
1. Capping on 5' end with 7-methyl-G;2. Polyadenylation on
3' end (approximately 200 As);3. Splicing out of introns
Q0075:How many nucleotides does tRNA contain?
75 to 90 nucleotides
Q0076:What sequence does every tRNA share at the 3' end?
CCA along with a high percentage of chemically modified
bases
Q0077:Amino acid binding to tRNA: Where (on the tRNA)
and how?
Where: 3' end;How: Covalently
Q0078:What is the enzyme involved in processing tRNA
Aminoacyl tRNA synthetase (uses 1 ATP)
Q0079:Aminoacyl tRNA synthetase: Mechanism
1. Scrutinizes amino acid before it binds to tRNA;2. Binds
AMP-amino group to 3' end of tRNA;3. Scrutinizes amino
acid again. If incorrect; bond is hydrolyzed.
Q0080:What is wrong with a mischarged tRNA
Reads the regular bond but inserts wrong amino acid.
Q0081:Which position on the codon is the wobble position?
3rd position
Q0082:Names of the steps in protein synthesis
1. Initiation;2. Elongation;3. Termination
Q0083:Sequence of events in the initiation step of protein
synthesis.
1. Initiation factors assemble the 40S ribosomal subunit with
the initiator tRNA;2. mRNA and (60S?) ribosomal subunit
combine with the 40S subunit;3. Initiation factors are released.
Q0084:Sequence of events in the elongation step of protein
synthesis.
1. Aminoacyl tRNA binds to the A site;2. Peptidyltransferase
catalyzes peptide bond formation;3. Peptidyltransferase
transfers growing polypeptide to amino acid in A site;4.
Ribosome advances three nucleotides toward 3' end of RNA
moving peptidyl RNA to P site.
Q0085:Sequence of events in the termination step of protein
synthesis.
1. Completed protein is released from ribosome;2. Ribosome
dissociates.
Q0086:Role of ATP in protein synthesis
ATP does tRNA Activation (charging)
Q0087:Role of GTP in protein synthesis
GTP does tRNA Going places (aka translocation) and
Gripping
Q0088:Role of A site in protein synthesis
A site holds incoming Aminoacyl tRNA.
Q0089:Role of P site in protein synthesis
P site accomodates growing Peptide.
Q0090:Role of E site in protein synthesis
E site holds Empty tRNA as it Exits
Q0091:Which post-translational modification involves
removal of N or C terminal pro-peptides from zymogens to
generate mature proteins?
Trimming
Q0092:What happens in post-translational trimming?
removal of N or C terminal pro-peptides from zymogens to
generate mature proteins
Q0093:Which post-translational modification involves
phosphorylation?
post-translational covalent alteration
Q0094:What happens during post-translational covalent
alterations?
Either;1. Phosphorylation;2. Glycosylation;3. Hydroxylation
Q0095:Which post-translational modification involves
glycosylation?
post-translational covalent alteration
Q0096:Which post-translational modification involves
hydroxylation?
post-translational covalent alteration
Q0097:What happens during proteasomal degradation?
Attachment of ubiquitin to defective proteins to tag them for
breakdown.
Q0098:Ubiquitin or Ubiquinone: Proteosomal degradation
Ubiquitin
Q0099:Ubiquitin or Ubiquinone: Coenzyme Q in oxidative
phosphorylation
Ubiquinone
Q0100:Where in the cell does the following occur: Fatty acid
oxidation (beta-oxidation)
Mitochondria
Q0101:Where in the cell does the following occur: acetyl-CoA
production
Mitochondria
Q0102:Where in the cell does the following occur: Krebs cycle
Mitochondria
Q0103:Where in the cell does the following occur: Glycolysis
Cytoplasm
Q0104:Where in the cell does the following occur: Fatty acid
synthesis
Cytoplasm
Q0105:Where in the cell does the following occur: Hexose
Monophosphate Shunt
Cytoplasm
Q0106:Where in the cell does the following occur: Protein
Synthesis
Rough endoplasmic reticulum in the cytoplasm
Q0107:Where in the cell does the following occur: Steroid
synthesis
Smooth endoplasmic reticulum in the cytoplasm
Q0108:Where in the cell does the following occur:
Gluconeogenesis
Pathway has steps in the mitochondria and in the cytoplasm
Q0109:Where in the cell does the following occur: Urea cycle
Pathway has steps in the mitochondria and in the cytoplasm
Q0110:Where in the cell does the following occur: Heme
synthesis
Pathway has steps in the mitochondria and in the cytoplasm
Q0111:What type of bonds hold the phosphoryls together in
ATP; and how much energy are the bonds worth?
Phosphoanhydride bonds are worth 7 kilocalories per mole
(but only between the alpha and beta and the beta and the
gamma; thus AMP's phosphoryl is not cleaved off for energy)
Q0112:How many ATP molecules are produced by aerobic
metabolism of glucose?
38 via the Malate shuttle; and 36 via the G3P shuttle.
Q0113:In aerobic metabolism of glucose; which pathway
produces 38 ATP?
Malate shuttle
Q0114:In aerobic metabolism of glucose; which pathway
produces 36 ATP?
G3P shuttle
Q0115:How much ATP is produced by anaerobic glycolysis?
2 ATP per glucose
Q0116:What is this molecule an activated carrier of?: ATP
Phosphoryls
Q0117:What is this molecule an activated carrier of?: NADH
Electrons
Q0118:What is this molecule an activated carrier of?: NADPH
Electrons
Q0119:What is this molecule an activated carrier of?: FADH2
Electrons
Q0120:What is this molecule an activated carrier of?:
Coenzyme A
Acyl
Q0121:What is this molecule an activated carrier of?:
Lipoamide
Acyl
Q0122:What is this molecule an activated carrier of?: Biotin
CO2
Q0123:What is this molecule an activated carrier of?:
Tetrahydrofolate
1-carbon units
Q0124:What is this molecule an activated carrier of?: S-
adenosyl-methionine
Methyl groups
Q0125:What is this molecule an activated carrier of?:
Thiamine Pyrophosphate
Aldehydes
Q0126:What activated carriers carry: Phosphoryl
ATP and GTP
Q0127:What activated carriers carry: Electrons
1. NADH;2. NADPH;3. FADH2
Q0128:What activated carriers carry: Acyl
1. Coenzyme A;2. Lipoamide
Q0129:What activated carriers carry: CO2
Biotin
Q0130:What activated carriers carry: 1-carbon units
1. Tetrahydrofolates (originally as formyl then methyl);2.
Biotin (as CO2);3. S-adenosyl-methionine (as CH3)
Q0131:What activated carriers carry: CH3 groups
1. S-adenosyl-methionine;2. N5-methyl-THF
Q0132:What activated carriers carry: Formyl groups
N10-formyl-THF
Q0133:What activated carriers carry: Aldehydes
Thiamine Pyrophosphate
Q0134:ATP and methionine react to form what?
S-adenosyl-methionine
Q0135:What reacts to yield S-adenosyl-methionine?
ATP and methionine
Q0136:What vitamin is necessary for regeneration of S-
adenosyl-methionine?
Vitamin B12
Q0137:When is NAD used?
Catabolic processes to carry reducing equivalents away as
NADH
Q0138:When is NADPH used?
1. Anabolic process (steroid and fatty acid synthesis);2.
Respiratory burst;3. P-450
Q0139:Where does NADPH come from?
HMP shunt
Q0140:What disease results from NADPH oxidase
deficiency?
Chronic Granulomatous Disease
Q0141:This enzyme phosphorylates glucose with high
affinity.
Hexokinase (as opposed to glucokinase)
Q0142:This enzyme phosphorylates glucose with low
affinity.
Glucokinase (as opposed to hexokinase)
Q0143:This enzyme phosphorylates glucose with a low
capacity.
Hexokinase (as opposed to glucokinase)
Q0144:This enzyme phosphorylates glucose and is feedback
inhibited by Glucose-6-Phosphate.
Hexokinase (as opposed to glucokinase)
Q0145:This enzyme phosphorylates glucose with a high
capacity.
Glucokinase (as opposed to hexokinase)
Q0146:This enzyme phosphorylates glucose and is not
feedback inhibited.
Glucokinase (as opposed to hexokinase)
Q0147:Glucokinase: Where is it found and why does it do
what it does?
Found in the liver and pancreatic beta cells. Phosphorylates
glucose to sequester it after a big meal.
Q0148:Hexokinase: Where is it found and why does it do
what it does?
Found in every cell's cytoplasm. Phosphorylates glucose to
proceed with glycolysis.
Q0149:What are the net reactants and products in glycolysis.
Reactants;1. Glucose;2. 2 Phosphates;3. 2 ADP;4. 2
NAD;Products;1. 2 Pyruvate;2. 2 ATP;3. 2 NADH;4. 2
H+;5. 2 H20
Q0150:What are the rate limiting steps of glycolysis?
1. Hexokinase (Glucose to Glucose-6-P);2.
*Phosphofructokinase-1 (Fructose-6-P to Fructose-1;6-
BP);3. Pyruvate kinase (Phosphoenolpyruvate to Pyruvate)
Q0151:Phosphofructokinase-1: What does it do; and what
stimulates and inhibits it?
PFK-1 1-phosphorylates fructose-6-phosphate to produce
Fructose-1;6-Bisphosphate;Inhibited by;1. ATP (don't need
more of me);2. Citrate (my cycle is going well);Stimulated
by;1. AMP (Hey; we need more ATP);2. Fructose-2;6-BP
(The fact that I'm being made means there's tons of glucose.)
Q0152:Pyruvate kinase: What does it do; and what stimulates
and inhibits it?
Pyruvate kinase converts phosphoenolpyruvate to pyruvate;
thereby producing two ATP;Inhibited by;1. ATP (don't need
more of me);2. Alanine (I came from pyruvate; so we don't
need any more.);Stimulated by;1. Fructose-1;6-BP (I was told
we needed more ATP; so here I am; so you better move the
line along.)
Q0153:Pyruvate dehydrogenase: What does it do; and what
stimulates and inhibits it?
Pyruvate dehydrogenase converts pyruvate to acetyl-coA;
and produces NADH and CO2;Stimulated by: excess
pyruvate?;Inhibited by;1. NADH (Listen; seriously; we don't
need anymore of me.);2. NADH (You produce NADH; soon
there'll be more of me.);3. Acetyl-CoA (Enough of me; save
your pyrvuate.)
Q0154:What disease state is glycolytic enzyme deficiency
generally associated with?
Hemolytic anemia
Q0155:What is the mechanism of hemolytic anemia in
someone with glycolytic enzyme deficiency?
1. Lack of glycolysis leads to lack of ATP in RBCs;2. Lack of
ATP leads to inactivity of Na; K-ATPase pump;3. Lack of
the pump leads to sodium influx;4. Water follows sodium into
the cell;5. The cell swells and bursts.
Q0156:What are the two most common glycolytic enzyme
deficiencies?
Pyruvate kinase (95% of cases) followed by glucose
phosphate isomerase (4% of cases)
Q0157:What are the 5 cofactors necessary for pyrvuate
dehydrogenase?
Lipoic acid plus the first four B vitamins in their active
forms;1. B1: TPP;2. B2: FAD;3. B3: NAD;4. B5: CoA
Q0158:What are the 5 cofactors necessary for alpha-
ketoglutarate dehydrogenase?
Lipoic acid plus the first four B vitamins in their active
forms;1. B1: TPP;2. B2: FAD;3. B3: NAD;4. B5: CoA
Q0159:What are the net reactants and products in the reaction
that Pyruvate Dehydrogenase catalyzes?
Reactants;1. Pyruvate;2. CoA;3. NAD;Products;1. Acetyl
CoA;2. CO2;3. NADH
Q0160:What activates and what inhibits pyruvate
dehydrogenase?
Activated by exercise; which stimulates;1. Increased
NAD/NADH ratio (We need more NADH.);2. Increased
ADP (We need more ATP.);3. Ca2+ (More of me leads
muscles to contract; and I'm taken up by mitochondria where
I tell PDH that we need more ATP.);Inhibited by;1. NADH
(No more of me please);2. ATP (likewise);3. Acetyl CoA
(ditto)
Q0161:Lipoamide or lipoate: Which carries aldehydes?
Lipoamide
Q0162:Lipoamide or lipoate: Which is a cofactor for pyruvate
dehydrogenase?
Lipoate (Lipoic acid)
Q0163:What toxin inhibits lipoic acid?
Arsenic
Q0164:What is the presentation of arsenic toxicity?
1. Vomiting;2. Rice water stools;3. Garlic breath
Q0165:Pyruvate dehydrogenase deficiency: Mechanism
Backup of pyruvate and alanine leads to lactic acidosis.
Q0166:Pyruvate dehydrogenase deficiency: Congenital or
Acquired
Both. Acquired cases happen in cases of B1 deficiency (such
as in alcoholics.)
Q0167:Pyruvate dehydrogenase deficiency: Presentation
Lactic acidosis and neurologic defects
Q0168:Pyruvate dehydrogenase deficiency: Treatment
Increased intake of ketogenic nutrients (such as high fat
content or increased lysine and leucine)
Q0169:What are the miscellaneous fates of pyruvate; and
what are the end products used for?
1. Alanine: Carries amino groups to the liver from muscle;2.
Oxaloacetate: Replenishes TCA cycle or is used
gluconeogenesis;3. Acetyl-CoA: Used in TCA cycle;4.
Lactate: No good use
Q0170:Which tissues and organs primarily convert pyruvate
into lactate?
1. RBCs and WBCs;2. Lens and cornea;3. Renal medulla;4.
Testes
Q0171:What enzymes and cofactors are used in conversion of
pyruvate to alanine?
Enzyme: Alanine Transaminase (ALT);Cofactors: None
Q0172:What enzymes and cofactors are used in conversion of
pyruvate to oxaloacetate?
Enzyme: Pyruvate Carboxylase (contains biotin and
magnesium);Cofactors: CO2 and ATP
Q0173:What are the reactants and products in the reaction
catalyzed by pyruvate carboxylase?
Reactant;Pyruvate (with CO2 and
ATP);Product;Oxaloacetate
Q0174:What are the reactants and products in the reaction
catalyzed by lactate dehydrogenase?
This reaction is reversible; so the products can switch with
the reactants;Reactants;1. Pyruvate;2. NADH
(rehydrogenates in this direction);3. H+;Products;1. Lactate;2.
NAD
Q0175:Where do the various pyruvate transformation
reactions happen?
Cytosol;1. ALT (Alanine to/from pyruvate);2. LDH (Lactate
to/from pyruvate);Mitochondria;1. Pyruvate carboxylase
(pyruvate to oxaloacetate);2. Pyruvate dehydrogenase
(pyruvate to acetyl-coa)
Q0176:Where does the Cori Cycle happen?
In the liver and muscle/RBCs;Liver: Pyruvate converts to
glucose;Muscle/RBCs: Glucose converts to Pyruvate
Q0177:What is the purpose of the Cori cycle?
Transfers excess reducing equivalents from RBCs and the
muscle to liver so they can function anaerobically
Q0178:What reaction does citrate synthase catalyze?
Oxaloacetate and acetyl coA combine to yield citrate.
Q0179:What is the order of the citric acid cycle beginning at
citrate?
CAn I Keep Selling Sex For Money; Officer?;1. Citrate;2. cis-
Aconitate;3. Isocitrate;4. alpha-Ketoglutarate;5. Succinyl
CoA;6. Succinate;7. Fumarate;8. Malate;9. Oxaloacetate
Q0180:What is the order of the citric acid cycle beginning at
cis-aconitate?
1. cis-Aconitate;2. Isocitrate;3. alpha-ketoglutarate;4. succinyl
coA;5. succinate;6. fumarate;7. money;8. oxaloacetate;9.
citrate
Q0181:What is the order of the citric acid cycle beginning at
isocitrate?
1. isocitrate;2. alpha-ketoglutarate;3. succinyl coa;4.
succinate;5. fumarate;6. malate;7. oxaloacetate;8. citrate;9. cis-
aconitate
Q0182:What is the order of the citric acid cycle beginning at
alpha-ketoglutarate?
1. alpha-ketoglutarate;2. succinyl coA;3. succinate;4.
fumarate;5. malate;6. oxaloacetate;7. citrate;8. cis-aconitate;9.
isocitrate
Q0183:What is the order of the citric acid cycle beginning at
succinyl coA?
1. succinyl coA;2. succinate;3. fumarate;4. malate;5.
oxaloacetate;6. citrate;7. cis-aconitate;8. isocitrate;9. alpha-
ketoglutarate
Q0184:What is the order of the citric acid cycle beginning at
succinate?
Sex Feels Marvelous Over Cordelia And If Kruti Sucks-a-
Neil;1. Succinate;2. Fumarate;3. Malate;4. Oxaloacetate;5.
Citrate;6. cis-aconitate;7. Isocitrate;8. alpha-ketoglutarate;9.
succinyl coA
Q0185:What is the order of the citric acid cycle beginning at
fumarate?
1. fumarate;2. malate;3. oxaloacetate;4. citrate;5. cis-
aconitate;6. isocitrate;7. alpha-ketoglutarate;8. succinyl coA;9.
succinate
Q0186:What is the order of the citric acid cycle beginning at
malate?
1. malate;2. oxaloacetate;3. citrate;4. cis-aconitate;5.
isocitrate;6. alpha-ketoglutarate;7. succinyl coA;8.
succinate;9. fumarate
Q0187:What is the order of the citric acid cycle beginning at
oxaloacetate?
1. oxaloacetate;2. citrate;3. cis-aconitate;4. isocitrate;5. alpha-
ketoglutarate;6. succinyl coA;7. succinate;8. fumarate;9.
malate
Q0188:What stimulates and inhibits citrate synthase?
Stimulate: Nothing;Inhibit: ATP
Q0189:What stimulates and inhibits isocitrate dehydrogenase?
Stimulate: ADP;Inhibit;1. ATP;2. NADH
Q0190:What stimulates and inhibits alpha-ketoglutarate
dehydrogenase?
Stimulate: Nothing;Inhibit;1. ATP;2. NADH;3. Succinyl CoA
Q0191:Which steps in the citric acid cycle produce CO2?
The steps where carbons are lost; the two structures after
isocitrate each have one less carbon than the last;1. Isocitrate
to alpha-ketoglutarate;2. alpha-ketoglutarate to succinyl coA
Q0192:Which steps in the citric acid cycle produce reducing
equivalents?
The only step that produces FADH2 is the only one that also
yields an F product;1. Isocitrate to alpha ketoglutarate (1
NADH);2. alpha-ketoglutarate to succinyl coA (1 NADH);3.
Succinate to Fumarate (1 FADH2);4. Malate to Oxaloacetate
(1 NADH)
Q0193:Which steps in the citric acid cycle produce ATP?
None; however 1 GTP is produced from the conversion of
Succinyl CoA to Succinate.
Q0194:How much ATP is produced by the citric acid cycle
per molecule of acetyl coA?
12 ATP;3 NADH x 3 ATP/NADH= 9 ATP;1 FADH2 x 2
ATP/FADH2 = 2 ATP;1 GTP x 1 ATP/GTP = 1 ATP;The
total is 12 ATP
Q0195:How much ATP is produced by the citric acid cycle
per molecule of glucose?
24;1 cycle;3 ATP/NADH= 9 ATP;1 FADH2 x 2
ATP/FADH2 = 2 ATP;1 GTP x 1 ATP/GTP = 1 ATP;The
total is 12 ATP per acetyl coA. However; there are 2 acetyl
coA molecules produced per glucose molecule. Thus the total
is 24.
Q0196:Name the complexes and important coenzymes and
cytochromes in the electron transport chain.
1. Complex I;2. Coenzyme Q;3. Complex III;4. Cytochrome
C;5. Complex IV;6. Complex V
Q0197:Where in the electron transport chain do NADH and
FADH2 release their electrons?
Complex I
Q0198:Where in the electron transport chain is O2 reduced to
2H2O?
Complex IV
Q0199:Where in the electron transport chain is ADP
converted to ATP?
Complex V aka ATP synthase aka mitochondrial ATPase
Q0200:Name three classes of oxidative phosphorylation
poisons.
1. Electron transport inhibitors;2. ATPase inhibitors;3.
Uncoupling agents
Q0201:What is the mechanism of electron transport
inhibitors?
1. Directly inhibit electron transport causing;2. Decreased
protein gradient and decrease in O2 consumption; thereby;3.
Blocking ATP synthesis
Q0202:What is the mechanism of ATPase inhibitors?
1. Directly inhibit mitochondrial ATPase causing;2. Increased
protein gradient and increased oxygen consumption; but no
ATP is produced because electron transport stops.
Q0203:What is the mechanism of uncoupling agents?
"Uncouples" ATP synthesis from gradient production;1.
Increase permeability of membrane;2. Proton gradient
decreases; but oxygen consumption increases; as the gradient
is not being maintained;3. ATP synthesis stops; but electron
transport continues.
Q0204:What is rotenone?
An electron transport inhibitor.
Q0205:What is the mechanism of CN?
Electron transport inhibition
Q0206:What is the mechanism of CO?
Electron transport inhibition
Q0207:What is antimycin A?
An electron transport inhibitor.
Q0208:What is the mechanism of oligomycin?
ATPase inhibition
Q0209:What is the mechanism of thermogenin?
Uncoupling protein OR UCP which is an uncoupling agent
Q0210:Where is thermogenin found?
Brown adipose tissue
Q0211:What is the mechanism of 2;4-dinitrophenol?
Uncoupling agent
Q0212:Name three uncoupling agents
1. UCPs (such as Thermogenin);2. 2;4-dinitrophenol;3.
aspirin
Q0213:Name the irreversible enzymes in gluconeogenesis; and
where they are found.
Pathway Produces Fresh Glucose;All the enzymes are found
only in the liver; kidney; and intestinal epithelium;1. Pyruvate
carboxylase in the mitochondria;2. PEP carboxykinase in the
cytosol;3. Fructose-1;6-bisphosphatase in the cytosol;4.
Glucose-6-Phosphatase in the endoplasmic reticulum
Q0214:Name the irreversible enzymes in glycolysis.
1. Hexokinase;2. Phosphofructokinase-1;3. Pyruvate kinase;4.
Pyruvate dehydrogenase
Q0215:What are the requirements of PEP carboxykinase?
GTP
Q0216:Where does the pentose phosphate pathway happen?
Cytoplasm of Red Blood Cells; and in lactating mammary
glands; liver; and adrenal cortex (all sites of fatty acid or
steroid synthesis except RBCs)
Q0217:How much ATP is used in the pentose phosphate
shunt?
Q0218:What are the main products of the pentose phosphate
shunt and their uses?
1. NADPH (for fatty acid and steroid synthesis; glutathione
reduction; and cytochrome P-450);2. Ribose-5-phosphate (for
nucleotide synthesis);3. G3P and F6P (glycolytic
intermediates)
Q0219:What are the key enzymes of the pentose phosphate
shunt and are the reactions reversible or irreversible?
1. Glucose-6-phosphate dehydrogenase (irreversible);2.
Transketolase (reversible)
Q0220:What does transketolase require?
Thiamine (Vitamin B1)
Q0221:What is the rate-limiting enzyme in the Pentose
phosphate pathway?
Glucose-6-Phosphate Dehydrogenase
Q0222:What is glutathione used for?
Detoxification of free radicals and peroxides.
Q0223:What does NADPH deficiency in RBCs result in?
Hemolytic anemia
Q0224:Name some oxidizing agents that someone with a
G6PD deficiency is vulnerable to.
1. Fava beans;2. Sulfonamides;3. Primaquine;4.
Antituberculosis drugs
Q0225:What protection does G6PD deficiency provide?
Protection against malaria
Q0226:Which group is more likely to have G6PD deficiency?
Blacks
Q0227:What are Heinz bodies?
altered Hemoglobin precipitates within RBCs; found in G6PD
deficiency
Q0228:What histologic change is seen in G6PD deficiency
Heinz bodies within red blood cells
Q0229:What is the etiology of fructose intolerance?
1. Lack of aldolase B;2. Build up of Fructose-1-Phosphate;3.
Decrease in available phosphate;4. Inhibition of
glycogenolysis and gluconeogenesis
Q0230:What is the clinical presentation of fructose
intolerance?
hypoglycemia; jaundice; cirrhosis; and vomiting
Q0231:What is the difference in presentation between von
Gierke's disease and fructose intolerance?
Both have hypoglycemia; jaundice; cirrhosis and vomiting;von
Gierke's disease also has lactic acidosis whereas fructose
intolerance does not.
Q0232:What is the treatment for fructose intolerance?
Decreased intake of both fructose and sucrose.
Q0233:What is the etiology of essential fructosuria?
Defect in fructokinase leading to lack of metabolism of
fructose. Benign and asymptomatic
Q0234:What is the clinical presentation of essential
fructosuria?
Fructose appears in the blood and urine
Q0235:Which is more serious; essential fructosuria or fructose
intolerance?
Fructose intolerance; because it depletes the cells of
phosphate.
Q0236:What is the etiology of classic galactosemia?
1. Absence of galactose-1-phosphate uridyl transferase;2.
Build up of toxic substances including galactitol
Q0237:What is the presentation of classic galactosemia?
Early;1. Galactosemia;2. Galactosuria;3. Vomiting;4.
Diarrhea;5. Jaundice;Late;1. Cataracts;2.
Hepatosplenomegaly;3. Mental retardation
Q0238:How does galactokinase deficiency present?
1. Galactosemia;2. Galactosuria;More severe symptoms such
as cataracts; hepatosplenomegaly and mental retardation can
follow.
Q0239:What is the treatment for classic galactosemia?
Exclude galactose and lactose from the diet.
Q0240:What enzyme converts galactose to galactitol?
Aldose reductase
Q0241:What does aldose reductase do?
Converts galactose to galactitol
Q0242:What enzyme converts Galactose to galactose-1-
phosphate?
Galactokinase
Q0243:What enzyme converts Galactose-1-Phosphate to
Glucose-1-Phosphate?
Uridyl transferase
Q0244:What enzyme converts UDP-galactose to UDP-
glucose?
4-epimerase
Q0245:What does galactokinase do?
converts Galactose to galactose-1-phosphate
Q0246:What does 4-epimerase do?
converts between UDP-galactose and UDP-glucose
Q0247:What does Uridyl transferase do?
1. converts UDP-glucose to UDP-galactose;2. converts
Galactose-1-Phosphate to Glucose-1-Phosphate
Q0248:What enzyme converts UDP-glucose to UDP-
galactose?
Uridyl transferase
Q0249:Which groups are more likely to be lactose intolerant?
1. Blacks;2. Asians
Q0250:What is the etiology of lactose intolerance?
Loss of brush-border lactase
Q0251:How does lactose intolerance present?
1. Bloating;2. Cramps;3. Osmotic diarrhea
Q0252:What is the treatment for lactose intolerance?
Avoid milk or add lactase pills to the diet
Q0253:What are the essential amino acids?
PVT TIM HALL;1. Phenylalanine;2. Valine;3. Threonine;4.
Tryptophan;5. Isoleucine;6. Methionine;7. Histidine;8.
Alanine;9. Leucine;10. Lysine
Q0254:What are the conditionally essential amino acids; and
why are they conditionally essential?
The condition is age. They are necessary early in life during
growth;Mnemonic: Babies CRY for Help;1. Cysteine;2.
aRginine;3. tYrosine;4. Histidine
Q0255:Cysteine or Cystine: The amino acid
Cysteine
Q0256:Cysteine or Cystine: Two copies of the amino acid
joined by a disulfide bond
Cystine
Q0257:Is the following amino acid essential or inessential; and
is it glucogenic; ketogenic; or both?: Phenylalanine
Essential;Both glucogenic and ketogenic
Q0258:Is the following amino acid essential or inessential; and
is it glucogenic; ketogenic; or both?: Valine
Essential;Glucogenic
Q0259:Is the following amino acid essential or inessential; and
is it glucogenic; ketogenic; or both?: Tryptophan
Essential;Both glucogenic and ketogenic
Q0260:Is the following amino acid essential or inessential; and
is it glucogenic; ketogenic; or both?: Threonine
Essential;Both glucogenic and ketogenic
Q0261:Is the following amino acid essential or inessential; and
is it glucogenic; ketogenic; or both?: Isoleucine
Essential;Both glucogenic and ketogenic
Q0262:Is the following amino acid essential or inessential; and
is it glucogenic; ketogenic; or both?: Methionine
Essential;Glucogenic
Q0263:Is the following amino acid essential or inessential; and
is it glucogenic; ketogenic; or both?: Histidine
Essential;Glucogenic
Q0264:Is the following amino acid essential or inessential; and
is it glucogenic; ketogenic; or both?: Arginine
Essential;Glucogenic
Q0265:Is the following amino acid essential or inessential; and
is it glucogenic; ketogenic; or both?: Leucine
Essential;Ketogenic
Q0266:Is the following amino acid essential or inessential; and
is it glucogenic; ketogenic; or both?: Lysine
Essential;Ketogenic
Q0267:Is the following amino acid essential or inessential; and
is it glucogenic; ketogenic; or both?: Tyrosine
Conditionally essential (during life and early
growth);(Phenylalanine and Tetrahydrobiopterin produce
tyrosine and dihydrobiopterin);Both glucogenic and ketogenic
Q0268:Is the following amino acid essential or inessential; and
is it glucogenic; ketogenic; or both?: Glutamate
Inessential (made from alpha-ketoglutarate);Glucogenic
Q0269:Is the following amino acid essential or inessential; and
is it glucogenic; ketogenic; or both?: Aspartate
Inessential (made from asparagine or oxaloacetate by aspartate
aminotransferase);Glucogenic
Q0270:Is the following amino acid essential or inessential; and
is it glucogenic; ketogenic; or both?: Proline
Inessential (Glutamate makes proline and
ornithine);Glucogenic
Q0271:Is the following amino acid essential or inessential; and
is it glucogenic; ketogenic; or both?: Glycine
Inessential (synthesized during reactions involving
tetrahydrofolate);Glucogenic
Q0272:Is the following amino acid essential or inessential; and
is it glucogenic; ketogenic; or both?: Cysteine
Conditionally essential (during life and early
growth);(Methionine begets S-adenosyl methionine which
begets intermediates which beget cysteine);Glucogenic
Q0273:Is the following amino acid essential or inessential; and
is it glucogenic; ketogenic; or both?: Alanine
Inessential (made from pyruvate by alanine aminotransferase
in the Cori cycle);Glucogenic
Q0274:Is the following amino acid essential or inessential; and
is it glucogenic; ketogenic; or both?: Serine
Inessential (made from a descendant of 3PG and with an
amine group from glutamate);Glucogenic
Q0275:Is the following amino acid essential or inessential; and
is it glucogenic; ketogenic; or both?: Glutamine
Inessential (made from glutamate);Glucogenic
Q0276:Is the following amino acid essential or inessential; and
is it glucogenic; ketogenic; or both?: Asparagine
Inessential (made from aspartate);Glucogenic
Q0277:Which amino acids are acidic?
Aspartate and glutamate are negatively charged at body pH
Q0278:Which amino acids are basic?
Arginine; Lysine and Histidine;Arginine and Lysine are
increased in histones which bind negatively charged
DNA;Histidine has no charge at body pH.
Q0279:Zinc deficiency: Presentation
"Delayed wound healing; hypogonadism; and decreased adult
hair (axillary; facial; pubic)"
Q0280:Zinc deficiency: Predisposes to what?
Alcoholic cirrhosis
Q0281:Ethanol metabolism: All steps with enzymes and
cofactors
"Step 1: Ethanol is oxidized by NAD (forming NADH) to
acetaldehyde using alcohol dehydrogenase. Step 2:
Acetaldehyde is oxidized by NAD (forming NADH) to
acetate using acetaldehyde dehydrogenase."
Q0282:Ethanol metabolism: Limiting reagent
NAD+
Q0283:Ethanol metabolism: Order of kinetics of alcohol
dehydrogenase
Zero-order kinetics
Q0284:Disulfiram: Mechanism
"Disulfiram inhibits acetaldehyde dehydrogenase; leading to
an accumulation of acetaldehyde; leading to increased
hangover symptoms."
Q0285:Which drug inhibits acetaldehyde dehydrogenase?
Disulfiram
Q0286:Ethanol hypoglycemia: mechanism
"1. Ethanol metabolism increases NADH/NAD ratio in the
liver. 2. Pyruvate and oxaloacetate are reduced by NADH
respectively to lactate and malate. 3. Decreased pyruvate and
oxaloacetate leads to decreased gluconeogenesis. 4. Decreased
gluconeogenesis leads to hypoglycemia."
Q0287:What are the consequences of the altered
NADH/NAD ratio seen in alcoholics?
"Short-term: Hypoglycemia; Long-term: Hepatic fatty
change"
Q0288:What is the mechanism behind chronic fatty change in
alcoholics?
"1. Ethanol metabolism leads to an increased NADH/NAD
ratio in the liver. 2. This ratio prefers fatty acid synthesis
over glycolysis."
Q0289:Kwashiorkor: Clinical picture
Small child with a swollen belly and depigmented hair.
Q0290:Kwashiorkor: Clinical presentation
"Kwashiorkor results from protein-deficient MEALS.
Malabsorbtion; Edema; Anemia; Liver (fatty change); Skin
lesions"
Q0291:Protein malnutrition leads to what disease?
Kwashiorkor (as opposed to Marasmus from energy
malnutrition)
Q0292:Energy malnutrition leads to what disease?
Marasmus (as opposed to Kwashiorkor from protein
malnutrition)
Q0293:Marasmus: Clinical presentation
"Tissue and muscle wasting; loss of subcutaneous fat; and
variable edema"
Q0294:"Chromatin structure: In the beads on a string analogy;
what are the beads?"
"Start with a nucleosome core made up of an 8 histone cube
(two each of positively-charged histones H2A; H2B; H3; and
H4). Negatively charged DNA loops twice around
nucleosome core."
Q0295:"Chromatin structure: In the beads on a string analogy;
what is the string and how long is it?"
Histone H1 ties the nucleosomes together in a 30-nm fiber
string
Q0296:Chromatin structure: What histones are included and
which of these are not in the nucleosome core?
"H1 (only one not in the core); H2A; H2B; H3; and H4"
Q0297:Heterochromatin or Euchromatin: Which is more
condensed?
Heterochromatin. Euchromatin is less condensed.
Q0298:Heterochromatin or Euchromatin: Which is less
condensed?
Euchromatin. Heterochromatin is more condensed.
Q0299:Heterochromatin or Euchromatin: Which is
transcriptionally active?
"Euchromatin (""eu"" means true; so think ""truly
transcribed"")"
Q0300:Heterochromatin or Euchromatin: Which is
transcriptionally inactive?
Heterochromatin
Q0301:Name the purines.
Adenine and Guanine
Q0302:Name the pyrimidines.
"Cytosine; Uracil; Thymine"
Q0303:Which base pair bond has 3 Hydrogen bonds?
Guanine to Cytosine
Q0304:Which base pair bond has 2 Hydrogen bonds?
Adenine to Thymine
Q0305:How many Hydrogen bonds does the Guanine to
Cytosine pairing have?
3
Q0306:How many Hydrogen bonds does the Adenine to
Thymine pairing have?
2
Q0307:Which amino acids are necessary for purine synthesis?
"Glycine; Aspartate; Glutamine"
Q0308:"In nucleic acids; what kind of substitution is a
transition?"
"TransItion = Identical type (Purine for purine or pyrimidine
for pyrimidine")
Q0309:"In nucleic acids; what kind of substitution is a
transversion?"
"TransVersion = conVersion between types (Purine for
pyrimidine or vice versa")
Q0310:What does it mean for genetic code to be
unambiguous?
Each codon specifies only one amino acid.
Q0311:What does it mean for genetic code to be degenerate?
More than one codon may code for the same amino acid.
Q0312:What does it mean for genetic code to be redundant?
More than one codon may code for the same amino acid.
Q0313:Which amino acid is coded by only one codon?
Methionine
Q0314:"~ average pKa of carboxyl group on AA"
2.3
Q0315:"~ pKa of side chain of Aspartic Acid"
"<4"
Q0316:"~ pKa of side chain of Glutamic Acid"
">4"
Q0317:"~ pKa of side chain of Histidine"
6
Q0318:"~ pKa of side chain of Cysteine"
8
Q0319:"~ average pKa of amino group on AA"
9.6
Q0320:"~ pKa of side chain of Tyrosine"
10
Q0321:"~ pKa of side chain of Lysine"
10.5
Q0322:"~ pKa of side chain of Arginine"
12.5
Q0323:"An acid with a pKa of x serves as a buffer best at x +
what?"
"positive or negative 1 (equal amounts of charged and
uncharged acid)"
Q0324:"Trypsin cleaves peptides at which side of what
residues?"
"C-terminal of lysine or arginine (the most basic amino acids)"
Q0325:"Cyanogen bromide cleaves peptides at which side of
what residues?"
"C-terminal of methionine"
Q0326:"Pepsin cleaves peptides at which side of what
residues?"
"C-terminal side of tyrosine; phenylalanine; and tryptophan
(all have phenyl groups; these are the same bonds as
chymotrypsin. Pepsin's action ceases when the NaHCO3
raises the pH of the intestinal contents)"
Q0327:"Chymotrypsin cleaves peptides at which side of
what residues?"
"C-terminal side of tyrosine; phenylalanine; and tryptophan
residues (all have phenyl groups; these are the same bonds as
pepsin; whose action ceases when the NaHCO3 raises the pH
of the intestinal contents)."
Q0328:"# of aas in one turn of alpha-helix"
3.6
Q0329:"Amino acids that disrupt alpha-helix"
"proline; many charged aas; bulky side chains"
Q0330:"Which reagent sequentially removes N-terminal
residues from a polypeptide?"
"Phenylisothiocyanate (Edman degradation)"
Q0331:"Which reagent sequentially removes C-terminal
residues from a polypeptide?"
"Carboxypeptidase"
Q0332:"What kind of inheritance and mutation is the alpha-1-
antitrypsin deficiency?"
"Autosomal recessive; single purine substitution (GAG to
AAG)"
Q0333:"Anode: What does it attract?"
"Anions"
Q0334:"Anode: What does it contain?"
"Cations"
Q0335:"Cathode: What does it attract?"
"Cations"
Q0336:"Cathode: What does it contain?"
"Anions"
Q0337:"Inhibitors of electron transport from FMNH2 to
Coenzyme Q"
"Amytal and Rotenone"
Q0338:"Inhibitors of electron transport from Cytochrome b
to Cytochrome c"
"Antimycin A"
Q0339:"Inhibitors of electron transport from Cytochrome
a+a3 to Oxygen"
"Cyanide; CO; and Sodium azide"
Q0340:"Where do GLUT1 receptors predominate over other
GLUT receptors?"
"RBCs"
Q0341:"Where do GLUT4 receptors predominate over other
GLUT receptors?"
"Adipose tissue and skeletal muscle"
Q0342:"Which tissues have cotransport of glucose?"
"Epithelial cells of the intestine; renal tubular cells; and
choroid plexus"
Q0343:"Which tissues (7) need glucose as fuel?"
"Brain; RBCs; Renal medulla; lens; cornea; testes; exercising
muscle"
Q0344:"Where is pyruvate carboxylase found and not
found?"
"Found in mitochondria of liver and kidney cells; not foudn in
mitochondria of muscle"
Q0345:"Where is Fructose 1-6 bisphosphatase found?"
"Liver and kidney"
Q0346:"What is the Cori cycle?"
"Lactate in muscle is shuttled to liver where it is turned into
glucose."
Q0347:"How does glucagon stimulate gluconeogenesis?"
"Regulation of F2;6-BP and inactivation of Pyruvate Kinase
via elevation of cAMP-dependent protein kinase A."
Q0348:"This oxidation accounts for about two thirds of the
total oxygen consumption and ATP production in most
animals; including humans."
"Oxidation of acetyl coA to CO2 and H2O."
Q0349:"What inhibits pyruvate dehydrogenase?"
"Acetyl CoA and NADH (no need for more of either). These
activate PD kinase (Phosphorylates enzyme with ATP;
which must be in abundance; so no more is needed)"
Q0350:"What stimulates pyruvate dehydrogenase?"
"ADP (need more ATP. Inhibits PD kinase and stimulates PD
phosphatase.)"
Q0351:"Which is active?: Phosphorylated or
dephosphorylated pyruvate dehydrogenase"
"Dephosphorylated."
Q0352:"What inhibits citrate synthase?"
"ATP and NADH (no need for more of either); Succinyl CoA
(""Slow down partner; the guys ahead of you are trying to do
their job!""); Acyl CoA fatty acid derivatives (Citrate
provides acetyl CoA to synthesize fatty acids and activates
acetyl CoA carboxylase; rate limiting enzyme of fatty acid
synthesis)."
Q0353:"Where in glycolysis and TCA does CO2 come off?"
"3 places: Pyruvate to Acetyl CoA; Isocitrate to alpha-
ketoglutarate; and alpha-ketoglutarate to Succinyl CoA"
Q0354:"What is the rate-limiting step of the TCA?"
"Isocitrate to alpha-ketoglutarate by isocitrate
dehydrogenase"
Q0355:"What activates isocitrate dehydrogenase?"
"ADP"
Q0356:"What inhibits isocitrate dehydrogenase?"
"ATP and NADH"
Q0357:"Sources of Succinyl CoA"
"TCA intermediate; and from odd chained fatty acids; and
from propionyl coA from metabolism of branched-chain
amino acids."
Q0358:"Uses of Succinyl CoA"
"TCA intermediate; and biosynthesis of heme"
Q0359:"Where in the TCA does NADH come from?"
"Pyruvate to Acetyl CoA; Isocitrate to alpha-ketoglutarate;
alpha-ketoglutarate to succinyl coA; Malate to Oxaloacetate"
Q0360:"Where in the TCA does FADH2 come from my
dear?"
"Succinate to fumarate my sweet."
Q0361:"Why is FAD used to oxidize succinate?"
"Succinate is not powerful enough to reduce NAD."
Q0362:"What are the important products of the HMP
pathway?"
"2 NADPH; Ribose; and glyceraldehyde-3-Phosphate and
Fructose-6-phosphate"
Q0363:"Which major metabolic reactions require Thiamine as
a cofactor?"
"TCA: Pyruvate dehydrogenase and alpha-ketoglutarate
dehydrogenase; HMP shunt: Transketolase"
Q0364:"What is NADPH used for?"
"1. Reductive biosynthesis (eg fatty acids and steroids) 2.
Reduction of oxygen directly (myeloperoxidase system's
famed respiratory burst) and hydrogen peroxide indirectly
(through reduction of glutathione) 3. Cytochrome P-450
mono-oxygenase system"
Q0365:"What is the famed respiratory burst?"
"The rapid conversion of O2 to superoxide using NADPH."
Q0366:"What disease process is due to a missing respiratory
burst?"
"Chronic granulomatous disease"
Q0367:"Where is the mutation for G6PD?"
"Point mutation in coding region of the G6PD gene (X-
linked)"
Q0368:"What is the relation of polyols to sugars?"
"Polyols are monosaccharides where the carbonyl group is
reduced to an alcohol."
Q0369:"What is a glycoside?"
"Carbohydrate attached to non-carbohydrate structures."
Q0370:"What is a reducing sugar?"
"A monosaccharide where the anomeric carbon (Carbon 1) is
free."
Q0371:"What is the result of lack of disaccharidase activity of
intestinal mucosa?"
"Osmotically active disaccharides suck water out of mucosa
causing osmotic diarrhea."
Q0372:"Where is fructokinase found?"
"Liver (processes most dietary fructose); kidney; small
intestine"
Q0373:"Why is fructose metabolism faster than glucose
metabolism?"
"Bypasses PFK; major regulatory step of glycolysis."
Q0374:"What enzyme is missing in hereditary fructose
intolerance?"
"Aldolase B"
Q0375:"What does aldose reductase do?"
"Reduces glucose to sorbitol"
Q0376:"Where is aldose reductase found?"
"Lens; retina; Schwann cells; kidney; placenta; RBCs; and
gonads"
Q0377:"What does sorbitol dehydrogenase do?"
"Oxidizes sorbitol to fructose."
Q0378:"Where is sorbitol dehydrogenase found?"
"Liver and gonads (ovaries; seminal vesicles; sperm)"
Q0379:"Mechanism of sorbitol toxicity"
"Extra glucose freely enters cells containing aldose reductase
which converts it to sorbitol. Sorbitol may not pass through;
and low or absent sorbitol dehydrogenase prevents it from
being changed to fructose. Strong osmotic effects lead to
swelling and damage."
Q0380:"Chondroitin Sulfate: Where found?/Distinguishing
characteristic from other GAGs"
"Cartilage; tendons; ligaments; aorta. Most abundant GAG in
body."
Q0381:"Chondroitin Sulfate: Use/Mechanism"
"Form proteoglycan aggregates. Cartilage: Bind collagen and
hold fibers in a tight; strong network"
Q0382:"Dermatan Sulfate: Where found?/Distinguishing
characteristic from other GAGs"
"Found in skin; blood vessels; and heart valves"
Q0383:"Dermatan Sulfate: Use/Mechanism"
Q0384:"Keratan Sulfate: Where found?/Distinguishing
characteristic from other GAGs"
"Found in cartilage proteoglycan aggregates with chondroitin
sulfate; and in cornea. Most heterogeneous GAG."
Q0385:"Keratan Sulfate: Use/Mechanism"
Q0386:"Heparin: Where found?/Distinguishing characteristic
from other GAGs"
"Intracellular compound (unlike other GAGs). Found in mast
cells of artery walls; especially in lungs; liver; and skin"
Q0387:"Heparin: Use/Mechanism"
"Anticoagulant"
Q0388:"Heparan Sulfate: Where found?/Distinguishing
characteristic from other GAGs"
"Extracellular; unlike heparin. Found in basement membrane
and as a ubiquitous component of cell surfaces."
Q0389:"Heparan Sulfate: Use/Mechanism"
Q0390:"Hyaluronic Acid: Where found?/Distinguishing
characteristic from other GAGs"
"Found in synovial fluid of joints; vitreous humor f eye;
umbilical cord; and loose connective tissue. Unlike other
GAGs: Unsulfated; not covalently attached to protein; and
only GAG not limited to animal tissue; but also found in
bacteria."
Q0391:"Hyaluronic Acid: Use/Mechanism"
"Lubricant and shock absorber"
Q0392:"Hunter's Syndrome vs Hurler's Syndrome: Enzyme
deficiency"
"Hunter's: Iduronate sulfatase; Hurler's: alpha-L-iduronidase"
Q0393:"Hunter's Syndrome vs Hurler's Syndrome: Corneal
clouding?"
"Hunter's: No; Hurler's: Yes"
Q0394:"Hunter's Syndrome vs Hurler's Syndrome: Mental
retardation?"
"Both (Hunter's ranges from mild to severe)"
Q0395:"Hunter's Syndrome vs Hurler's Syndrome: Physical
deformity?"
"Hunter's: Mild to severe; Hurler's: Dwarfing; coarse facial
features; (gargoylism)"
Q0396:"Hunter's Syndrome vs Hurler's Syndrome: Which
GAGs' degradation is affected?"
"Both: Dermatan sulfate and Heparan sulfate"
Q0397:"Hunter's Syndrome vs Hurler's Syndrome: Severity?"
"Hunter's: Less Hurler's: More"
Q0398:"Hunter's Syndrome vs Hurler's Syndrome:
Inheritance?"
"Hunter's: X-linked Recessive; Hurler's (and all other
mucopolysaccharidoses): Autosomal recessive"
Q0399:"Hunter's Syndrome vs Hurler's Syndrome: Aggressive
behavior?"
"Hunter's: Yes; Hurler's: No"
Q0400:"Mnemonic for Hurler's syndrome: HURLERS. What
does it stand for?"
"H: Hepatosplenomegaly/Heparan and Dermatan sulfate;
U:Ugly facies; R: aRteries filled with GAGs; L: L-
iduronidase; E: Eyes clouded; early death; R:
Retardation/Respiratory obstruction; S: Short/stubby fingers"
Q0401:"I-Cell disease: Pathophysiology"
"Inability of cell to phosphorylate mannose residues on
glycoproteins indicating that they are lysosome bound."
Q0402:"I-Cell disease: Presentation"
"Skeletal abnormalities; restricted joint movement; coarse
facial features; severe psychomotor impairment; death by 8
years"
Q0403:"Refsum Disease: Pathophysiology"
"Inability to degrade phytanic acid; resulting in accumulation
in plasma and tissues"
Q0404:What is PKU?
think smelly; retarded babies
Q0405:What's one reason that binging on booze is a bad idea
(aside from the ugly people you might sleep with;)?
alcohol-> increased NADH -> decreased gluconeogenesis ->
acidosis -> huge ER bill
Q0406:What is Kwashiorkor?
think Starvin' Marvin
Q0407:What's another reason for not being an alky (besides
the meetings)?
pellegra- vitamin B3 deficit that gives you a rash; the shits;
and altered mental status (even when sober)
Q0408:Why is my urine black and what the Hell are these
black dots on my eyes?!
alkaptonuria
Q0409:What is familial hypercholesterolemia?
defective LDL receptors-> accelerated atherosclerosis &
xanthomas
Q0410:Name the fat soluble vitamins; Fat Ass!
think Eating Donuts Adds Kilocalories!
Q0411:Why could a person be deficient in fat soluble
vitamins (esp considering that most of us have plenty of
space to store these buggers!)?
think malabsorption- sprue; CF; too much of Mom's mineral
oil tx (a spoon a day keeps the enema away!)
Q0412:What do B vitamin deficiencies result in (other than
pernicious anemia)?
dermatitis; glossitis; shits
Q0413:What is beriberi? Hint: It's not a Voodoo curse.
vitamin B1 deficiency; spell it ber1ber1 (1=i); B1 is required
for TPP (generates pyruvate) & transketolase (HMP shunt)
Q0414:What's the difference b/w wet & dry beriberi (other
than that not so fresh feeling)?
dry= polyneuritis; muscle wasting;wet=dilated
cardiomyopathy; edema
Q0415:What happens when you don't get your riboflavin
(B2) on?
it's important to have ribo-"flava" (not just b/c the chicks dig
it) but FAD & FMN come from it; flava is not just a FAD
but a Functionally Mandatory Necessity! not having flava
causes angular stomatitis; cheliosis; & corneal vascularization
(chicks don't dig this)
Q0416:What the Hell is Pantothenate? Is that the crap in
Pantene that will give my hair lusterous shine upon one
washing?
it's B5. it helps make CoA & fatty acid synthase (no wonder
why i'm so damn sexy!); lack of B5 gives you dermatitis;
enteritis; alopecia & adrenal insufficiency. com'on girls; no
guy wants a flaky skinned; bald girlfriend who's adrenal glands
don't put out; so take your vitamins!
Q0417:Was I absent the day we learned about pyridoxine
(B6) or do I merely have a B6 deficiency?
I was probably in class the day they taught this but suffering
from convulsions/hyperirritablity due to my B6 deficiency
brought on by the stresses of med school. it turns out that B6
is needed for ALT; AST (transamination); decarboxylation; &
heme synthesis.
Q0418:Why is B12 important (for the umpteenth thousand
time)?
B12 (aka cobalamin)required for homocysteine methylation &
methylmalonyl-CoA handling; decreased homocysteine->
decreased methionine-> messed up myelin & increased
methylmalonyl-CoA-> increased methylmalonic acid->
messed up myelin; vegetarians eat your heart out (no really;
b/c its full of the B12 you'll need to thwart off macrocytic;
megaloblastic anemia); US causes are due to malabsorption (vs
dietary insufficiency); think sprue; Crohn's; pernicious
anemia; do a Schilling test
Q0419:Why is folic acid so important? Does it justify all of
the public service announcements?!
geeze; it's only important if you want to synthesize
DNA/RNA! why the concern??
Q0420:What's biotin? Sounds like some tree hugging herbal
medicine crap!
biotin is needed for carboxylating (eg oxaloacetate; malonyo-
CoA; methylmalonyl-CoA); deficits lead to dermatitis &
enteritis due to antibiotic use or ingesting raw eggs (Rocky
must have had some mad IBS!)
Q0421:Why do we need vitamin D?
b/c we don't want rickets! there is such thing as too much of a
good thing; though- too much vitamin D-> hypercalcemia;
stupor (think sarcoidosis)
Q0422:Why does Mom always shove vitamin C down your
throat?
no Mom wants a kid w/scurvy unless she's British. vitamin C
cross links collagen for healing; facilitates iron absorption; &
needed for dopamine synthesis (is this why the British are so
static?)
Q0423:What does vitamin E do?
protects RBCs
Q0424:What does vitamin K do?
K is for koagulation (spelling proficiency wasn't a requirement
for med school matriculation); intestinal coodies are required
for its synthesis (this is why babies & pts on broad spectrum
anti-biotics have increased PT & PTT; warfin is it's nemesis
(warfin is at war w/ vitamin K)
Q0425:Who are vitamin K's dependents?
after much investigation; the family court ruled that vitamin K
is responsible supporting its progenous clotting factors II;
VII; IX; X; & protein C (until age 18 whereby his progeny
will bleed to death)
Q0426:What vitamin keeps your testicles plump and your
hair flowing? (Guys; take notes!)
zinc; aside from small balls and baldness; lack of zinc will
cause delayed wound healing & predispose you to alcoholic
cirrhosis!
Q0427:Explain ethanol metabolism (and don't say you're too
drunk to remember!)
ethanol-> acetaldehyde-> acetate; requires alcohol
dehydrogenase; acetaldehyde dehydrogenase; NAD+; NADH;
NAD+ is limiting reagent
Q0428:What gives you a hang-over? Hint: the answer isn't St.
Ides Malt Liquor (although this is justifiable).
saturation of acetylaldehyde dehydrogenase; this is how
antabuse works
Q0429:What is marasmus?
tissue/muscle wasting due to energy malnutrition (compare
w/Kwashiorkor)
Q0430:What is a nucleosome?
it's the DNA AND core histones; the "beads" on the string
that; altogether; comprise what's called chromatin
Q0431:What's so cool about H1 (histone 1)?
it's the histone that ties all the nucleosomes together. H1 is a
rebel; not part of the core b/c it's too cool for the core.
Q0432:What is heterochromatin? Hint: chromatin does not
have a sexual preference.
this is the transcriptionally abstaining form of chromatin. it's
very uptight (looped around histones). it's not promiscuous
like that loose slut euchromatin.
Q0433:Name the purines?
think "pure As Gold"- A;G
Q0434:What nucleotides bind to which?
G-C (strongest); A-T
Q0435:What goes into making a good purine?
besides sugar and spice and everything nice; purines require
glycine; aspartate; and glutamine
Q0436:What's this difference b/w a transition mistake & a
transversion mistake?
transItion= Identical substitute;transVersion= conVersion b/w
types
Q0437:Why will mother nature never receive a Pulitzer Prize?
b/c her writing is redundant and lacks punctuation. in her
defense; her writing is also unambiguous & used universally
Q0438:What's a silent mutation?
it's more palatable than a missense or nonsense mutation.
think "it is better to remain silent & be thought a fool then to
speak & remove all doubt". silent mutations are often the
result of a tRNA wobble at the 3rd position (damn it tRNA;
switch to decaf!)
Q0439:What is a missense mutation?
it's replacing one aa with a similar aa. kind of like substituting
a democrat with a republican.
Q0440:What is a nonsense mutation?
think stop the nonsense
Q0441:What is a frame shift mutation?
this is really bad. its when your tRNA starts reading The
Oddessy but becomes impatient & settles for the Cliff Notes.
Q0442:How are DNA topoisomerases & conditioner alike?
they both remove those pesky tangles!
Q0443:Why is DNA so codependent?
DNA; like many of us; needs the motivation of another to
function. primase is the muse of DNA. she (or he) makes the
RNA primer on which DNA polymerase III can begin
replication.
Q0444:Does DNA polymerase ever look back?
it may seem that DNA polymerase has no regrets and
unaffectedly carries on in his 5'->3' direction. in truth; though;
he is very aware of his past mistakes & corrects them in the
3'->5' direction with exonucleases.
Q0445:What does DNA polymerase I have against RNA
primer?
she has always hated that tart; RNA primer. thus she uses her
exonucleases to degrade RNA primer at any given chance &
fills in the gaps w/DNA (she is much against
interrelationships b/w RNA & DNA (she's a deoxyribose
supremicist).
Q0446:What keeps us from getting xeroderma pigmentosa?
we have endonucleases that kick out messed up nucleotides.
Q0447:Are all bases created equal?
NO! we make a lot of messed up bases that glycolases remove
by cutting the base out at a pyramimidic site.
Q0448:What happens when there're irreconsilable differences
b/w nucleotides?
if counseling doesn't work; then your body may hire a
mismatch repair attorney. people w/hereditary nonpolyposis
colon cancer lack access to litigation.
Q0449:What are the different kinds of RNA polymerases in
eukaryotes?
I=rRNA;II=mRNA;III=tRNA
Q0450:Which RNA polymerase helps DNA open up?
II=mRNA poly. her nemesis is her mother-in-law alpha-
amantin (she thwarts all efforts of mRNA poly by serving her
death cap mushrooms at all family get togethers).
Q0451:When does transcription begin?
in AUG just like school. codes for methionine
Q0452:When does transcription end (and don't say JUN)?
think U Go Away; U Are Away; U Are Gone (geeze; mRNA
can take a hint!).
Q0453:What is a promoter?
well DNA likes to be recognized for its contributions too. it
takes a lot of work to make protein & this should be
rewarded. DNA doesn't care about money; having a trust fund
an all; DNA is rewarded for its efforts by being somewhat
relieved of duty by RNA poly & other transcription factors.
this only takes place after DNA has done most of the work &
has reached a TATA or CAAT box.
Q0454:What is an enhancer?
area of DNA that attracts transcription factors that enhance
gene expression.
Q0455:What is an operator?
area of DNA that attracts transcription factors that repress
gene expression.
Q0456:What's the difference b/w an intron & exon?
exons are what contribute to your growth while introns are
just interuptions along the way (kind of like your first
boyfriends); introns remain in the nucleus.
Q0457:How does hnRNA become whole in spite of all of her
intron baggage?
well differences are the "splice" of life; so hnRNA discovers
new meaning by redefining herself via new experiences. she
decides to move on w/the aide of her snRP friends; forming a
spliceosome alliance. They help her release her intron baggage;
thereby allowing her to persue healthy relationships w/exons.
happy w/the exons; hnRNA agrees to seal the deal by capping
& polyadenylation (huge commitment). she is now referred to
as mRNA (she's old fashioned & conceeded to the name
change).
Q0458:What is Pellegra?
think 3D: diarrhea; dermatitis; dementia; caused by niacin
(B3) def or a tryptophan def; B3 comes from tryptophan.
Q0459:Name the Vitamin D forms.
D2= ergocalciferol (drink milk); D3= cholecalciferol (get some
sun); 25-OH D3= storage; 1;25 (OH)2 D3= active form->
intestinal absorption of calcium & phosphate.
Q0460:What is tRNA?
transfer RNA is the pre-aminoacid. the amino acid is
covalently attached to its 3' end.
Q0461:What does tRNA look like?
cloverleaf shape; CCA at 3'end.
Q0462:what is aminoacyl-tRNA synthetase?
it's the enzyme that makes the amino acid; there's 1 for every
kind of amino acid; it is also a proof reader for its amino acid;
it requires ATP to make a peptide bond but will read the
transcript w/o it.
Q0463:why does tRNA wobble?
b/c it need only accurately read the first two nucleotides; then
it can just insert whatever (hopefully a nucleotide that codes
for the proper amino acid).
Q0464:How is protein synthesis initiated?
a 30S ribosome unit/initiator tRNA are hooked up w/the
assistance of initiation factors
Q0465:what happens during elongation?
1. aminoacyl tRNA binds to A site 2. peptidyltransferase
makes a peptide bond & transfers growing polypeptide chain
to A site 3. ribosome cruises 3 nucleotides toward 3' RNA
while moving peptidyl RNA to P site.
Q0466:how is protein synthesis terminated?
protein is released from ribosome.
Q0467:what is the E site of the ribosome?
where tRNA is held while exiting.
Q0468:what is trimming?
post-translational modification; removal of N or C terminal
from a zymogen.
Q0469:what is a covalent modification?
post-translational phosphor/glycos/hydroxylation.
Q0470:what does ubiquitin do?
it is the scarlet letter to be worn by defective proteins.
Q0471:how are cell cycles regulated?
by checkpoints that control the cell phases; regulators include
cyclins; cdks; & tumor suppressors.
Q0472:what's included in mitosis?
PMAT; this is the shortest phase.
Q0473:what's included in interphase?
G1; S ; G2
Q0474:what's Go? Hint: it's not that lame movie.
it's where permanent cells stay if you refrain from dropping
acid. neurons; skeletal mm; RBCs; cardiac mm.
Q0475:what are stable cells?
although they are compliant w/their Prosac tx; they are also
capable of entering G1 if stimulated; otherwise they'll stay in
G1; hepatocytes; lymphocytes
Q0476:what are labile cells?
they never Go; they are always movin' rapidly; though;
marrow; gut epithelium; hair
Q0477:What takes place in the rough ER?
synthesis of exported secretory proteins & N-linked
oligosaccharide addition (eg goblet cells & plasma cells are rich
w/rough ER).
Q0478:What tkaes place in the smooth ER?
site of steriod synthesis & detox.
Q0479:What does the golgi do?
processing & packaging of proteins & lipids from ER to
plasma membrane; modifies N-oligosaccharides on asparagine;
adds O-oligisaccharides to serine & threonine; adds mannose-
6-P; assembles & sulfates proteoglycans & tyrosine.
Q0480:What is I-cell disease?
when mannose-6-P addition by the golgi doesn't target
lysosome proteins to lysosome; coarse face; clouded corneas;
restricted jiont movement; high plasma lysosomal enzymes;
fatal in childhood.
Q0481:What is COPI?
vesicular trafficking protein; golgi -> ER (retrograde).
Q0482:What is COPII?
vesicular trafficking protein; RER -> cis-golgi (anterograde).
Q0483:What is clathrin?
vesicular trafficking protein; trans-golgi -> lysosomes; plasma
membrane -> endosomes.
Q0484:What are microtubules?
polymerized dimers of alpha/beta-tubulin; 2GTP
bound/dimer; part of flagella; cilia; & spindles.
Q0485:Name 5 drugs that act on microtubules?
mebendazole; taxol; griseofulvin; vincristine; colchicine.
Q0486:What is Chediak-Higashi syndrome?
microtubule polymerization defect.
Q0487:What are cilia made of?
microtubule doublets (9 +2)linked by dynein ATPase.
Q0488:What is Kartagener's syndrome?
defective dynein resulting in defective cilia.
Q0489:What is the plasma membrane made of?
cholesterol; phospholipids; sphingolipids; glycolipids;
proteins.
Q0490:What is phosphatidylcholine(aka lecithin)?
component of RBC membrane; myelin; bile; & surfactant;
esterfies cholesterol (eg LCAT).
Q0491:Name 2 drugs that inhibit the sodium pump?
ouabain binds K+ site; cardiac glycosides inhibit
Na+/K+ATPase.
Q0492:Name the 4 types of collagen.
type 1= bone; skin; tendon; cornea; type II= catilage; type III
= reticulin; type VI= basement membrane.
Q0493:What cells make collagen?
fibroblasts.
Q0494:How is collagen made?
preprocollagen synthesized in RER-> hydroxylation (req
Vitamin C)-> glycosylation in golgi & synthesis of
procollagen; exocytosis; proteolysis into tropocollagen;
crosslinking forms collagen fibrils.
Q0495:What is Ehlers-Danlos syndrome?
defective collagen synthesis; hyperextendible skin; bruising;
hypermobile joints; assoc w/berry aneurysms; inherited.
Q0496:What is osteogenesis imperfecta?
abnormal type I collagen synthesis; autosomal dominant;
fractures; blue sclerae; hearing loss; dental problems.
Q0497:What is Marfan's syndrome?
defective fibrillin.
Q0498:What is elastin made of?
non-hydroxylated proline & lysine; elastin= tropoelastin +
fibrillin scaffolding; elastase allows relaxed form; alpha1-
antitrypsin inhibits elastase.
Q0499:What happens in the mitochondria?
beta-oxidation; acetyl-CoA production; Kreb's cycle.
Q0500:What happens in the cytoplasm?
glycolysis; fatty acid synthesis; TTP shunt; protein synthesis
(RER); steroid synthesis (SER).
Q0501:What happens in both the mitochondria & the
cytoplasm?
gluconeogenesis (hepatocytes); urea cycle; heme
Q0502:What is S-adenosyl-methionine (aka SAM)?
ATP + methionine; transfers methyl units; relies on B12.
Q0503:What is NADPH?
electron acceptor used in anabolic processes (eg steroid
synthesis); respiratory burst; & P-450; comes from the TPP
shunt.
Q0504:What is chronic granulomatous disease?
deficit of NADPH oxidase (makes bleach out of O2);
neutrophils can't kill bugs
Q0505:What blood problem is commonly assoc. w/a
glycolytic enzyme deficiency?
hemolytic anemia b/c RBC's rely on glycolysis for energy.
Q0506:What is the pyruvate dehydrogenous complex?
the enzyme + vitamins B1;2;3;5; + lipoic acid; makes
pyruvate into acetyl-CoA; activated by excercise.
Q0507:What happens when you have a pyruvate
dehydrogenase deficiency?
lactic acidosis; neurologic defects; tx w/ketogenic nutrients.
Q0508:How many ATP does 1 NADH make per turn?
3
Q0509:How many ATP does 1 FADH2 make per turn?
2
Q0510:Name 8 ox-phos poisons.
rotenone; CN-; antimysin A; CO (e- transport inhibitors);
oligomycin (ATPase inhibitor); UCP; 2;4-DNP; aspirin
(uncouplers).
Q0511:What happens when you have a glucose-6-P
deficiency?
cannot generate G6PD that is required to reduce glutathionine
that detoxifies the free rads & peroxides; RBC's are especially
susceptible to oxidizing agents & will form hemoglobin
precipitates (Heinz bodies); Blacks; X-linked recessive.
Q0512:What happens when you have an aldolase B
deficiency?
recessive; fructose accumulation; inhibition of glycogenolysis
& gluconeogenesis; hypoglycemia; jaundice; cirrhosis;
vomiting.
Q0513:What is essential fructosuria?
deficient fructokinase; benign.
Q0514:What is galactosemia?
autosomal recessive; absence of galactose-1-P
uridyltransferase; accumulation of toxins (eg galactitol);
cataracts ; hepatosplenomegaly; mental retardation
Q0515:Name the essential aminoacids;Hint: PriVaTe TIM
HALL.
phe; val; thr; trp; ile; met; his; arg; leu; lys
Q0516:What is hyperammonemia?
can be acquired (eg liver damage) or hereditary (eg ornithine
transcarbamoylase def); excess NH4+ -> inhibition of Kreb's
cycle; tremor; slurring; vomiting; cerebral edema; blurred
vision; somnolence.
Q0517:Why do we need insulin?
allows entrance of glucose into adipose & muscle cells.
Q0518:What does insulin inhibit?
glucagon release by alpha pancreas cells.
Q0519:What does insulin do?
increases: glucose transport; glycogen synthesis/storage; TG
synthesis/storage;Na+ retention; protein synthesis (muscles).
Q0520:What cells don't require glucose? Hint: BRICK L
brain; RBCs; intestine; cornea; kidney; liver.
Q0521:What role does adrenaline (aka epinephrine) play in
glycogenensis & glycogenolysis?
glycogenesis = (-);glycogenolysis = (+)for both muscle & liver
glycogen stores.
Q0522:What's the difference in glycogen response for muscle
vs liver?
muscle metabolizes glucose fast; the liver acts to maintain
blood sugar levels.
Q0523:How do you synthesize fat?
acetyl-CoA (mitochondria)-> citrate shuttle (matrix)-> acetyl-
CoA + biotin (cytoplasm)-> malonyl CoA-> FA
Q0524:How do you burn fat?
FA + CoA-> acyl-CoA (cytoplasm)-> carnitine shuttle
(matrix)-> acyl-CoA which is beta-oxidized into acetyl-CoA
groups.
Q0525:What are ketone bodies?
FA + aminoacids in the liver -> acetoacetate + beta-
hydroxybutyrate. these products can be used in leiu of
glucose during fasting & diabetes for the brain & muscle;
fruity breath.
Q0526:How do you make cholesterol?
HMG-CoA reductase is the rate limiting step; converts
HMG-CoA to mevalonate; most cholesterol gets esterfied by
LCAT.
Q0527:What drug inhibits cholesterol synthesis?
lovestatin inhibits HMG-CoA reductase.
Q0528:What are the essential fatty acids?
linoeic & linolenic acid. eicosanoids rely on these babies!
Q0529:What does pancreatic lipase do?
degrades TG in small intestine.
Q0530:What does lipoprotein lipase do?
degrades TG in chylomicrons & VLDLs.
Q0531:What does hepatic TG lipase do?
degrades TG in IDL.
Q0532:What does hormone sensitive lipase do?
degrades TG in adipocytes.
Q0533:What does LCAT do?
esterfies cholesterol.
Q0534:What does CEPT do?
transfers cholesterol esters to other lipoproteins.
Q0535:What does A1 do?
activates LCAT.
Q0536:What does B-100 do?
binds to LDL receptor & mediates VLDL secretion.
Q0537:What does CII do?
it's a cofactor for lipoprotein lipase.
Q0538:What does B-48 do?
mediates chylomicron secretion.
Q0539:What does E do?
mediates extra remnant uptake.
Q0540:What are lipopriteins made of?
cholesterol; TG; phospholipids.
Q0541:What do chylomicrons do?
takes TG from intestine to peripheral tissues & cholesterol to
liver.
Q0542:Which lipoproteins do chylomicrons need?
B-48; A;C;E.
Q0543:What does VLDL do?
takes liver TGs to peripheral tissues.
Q0544:Which lipoproteins do VLDLs need?
B-100; C-II; E
Q0545:What does IDL do?
comes from VLDL degradation. takes TGs & cholesterol to
liver to process into LDL.
Q0546:What lipoproteins does IDL need?
B-100; E.
Q0547:What does LDL do?
takes liver cholesterol to peripheral tissues; formed from
VLDL via lipoprotein lipase in peripheral tissue.
Q0548:What lipoproteins does LDL need?
B-100.
Q0549:What does HDL do?
takes peripheral cholesterol to liver; also a storage for apoC &
apoE for chylomicron & VLDL metabolism; secreted by liver
& intestine.
Q0550:How is heme broken down?
heme-> biliverdin-> bilirubin -> liver -> bile.
Q0551:What is heme made of?
2 alpha + 2 beta polypetide subunits.
Q0552:Explain R vs T forms of heme.
T = low O2 affinity; R= high O2 affinity. T unloads!
Q0553:What favors T form over R form heme?
increased: Cl-; H+; CO2; 2;3-BPG; temperature favor O2
unloading; shifts curve right.
Q0554:What happens to CO2?
travels as bicarbonate in blood to lungs; binds to globin (not
heme); favors T form of heme.
Q0555:What is methemoglobin?
this is oxidized hemoglobin (Fe3+) that prefers CN- over O2;
push nitrates!
Q0556:What is carboxyhemoglobin?
hemoglobin has a fettish for CO.
Q0557:What are the four irreversible enzymes in glycolysis?
1. hexokinase/glucokinase;2. phosphofructokinase-1;3.
pyruvate kinase;4. pyruvate dehydrogenase
Q0558:What is the rate-limiting step in glycolysis?
Conversion of Fructose-6-phosphate into Fructose-1;6 BP via
phosphofructokinase 1
Q0559:What factor negatively inhibits hexokinase in
glycolysis?
Glucose-6-Phosphate
Q0560:What factors (2) negatively inhibit PFK-1 in
glycolysis?
1. ATP;2. citrate
Q0561:What factors (2) positivcely affect PFK-1 in
glycolysis?
1. AMP;2. fructose-2;6-BP
Q0562:What factors (2) NEGATIVELY inhibit pyruvate
kinase in glycolysis?
1. ATP;2. alanine
Q0563:What factor positively affects pyruvate kinase in
glycolysis?
fructose-1;6 BP
Q0564:What factors (3) negatively inhibit pyruvate
dehydrogenase in glycolysis?
1. ATP;2. NADH;3. acetyl-Coa
Q0565:What enzymes (2) CONVERT D-glucose into
Glucose-6-phosphate in glycolysis?
1. hexokinase;2. gLucokinase (liver only)
Q0566:What enzyme CONVERTS PEP into pyruvate?
pyruvate kinase
Q0567:What enzyme CONVERTS pyruvate into Acetyl-
CoA
pyruvate dehydrogenase
Q0568:What enzyme converts Fructose-6-P into Fructose-
1;6-BP?
Phosphofructokinase (rate-limiting step)
Q0569:What glycolytic enzyme deficiencies result in
hemolytic anemia? (7)
1. hexokinase;2. glucose phosphate isomerase;3. aldolase;4.
triosephosphate isomerase;5. phosphate glycerate kinase;6.
enolase;7. pyruvate kinase
Q0570:Do RBCs possess mitochondria?
no: metabolize glucose anaerobically and thus depend solely
on glycolysis
Q0571:Order of enzymes in a phagolysosome that destroy
bacteria in oxygen-dependent respiratory burst?
1. NADPH OXIDASE;2. SOD;3. MYELOPEROXIDASE
Q0572:What enzyme converts O2 into its free radical?
NADPH OXIDASE; using NADPH
Q0573:What enzyme converts an O2 free radical into H2O2?
SOD
Q0574:What enzyme converts H2O2 into HOCl free radical
myeloperoxidase; using a chloride anion
Q0575:What enzyme converts GSH into GSSG?
catalase; via oxidation using H2O2
Q0576:What enzyme converts NADPH into NADP+ using
GSSG?
glutathione reductase; resulting in GSH and NADP+
Q0577:What enzyme restores NADPH by converting G6P
into 6-phosphogluconolactone?
Glucose-6-phosphate dehydrogenase
Q0578:A deficiency in what enzyme can cause chronic
granulomatous disease?
NADPH OXIDASE DEFICIENCY --> CGD
Q0579:How many enzymes does the Pyruvate
Dehydrogenase Complex contain?
3 enzymes
Q0580:What are the 5 co-factors for the Pyruvate
Dehydrogenase Complex?
1. Pyrophosphate;2. FAD;3. NAD;4. CoA;5. Lipoic
acid;(First 4 B vitamins plus lipoic acid)
Q0581:From what is PyroPhosphate derived in the PDH
complex?
1. Vitamin B1 (thiamine);2. TPP
Q0582:From what is FAD derived in the PDH complex?
Vitamin B2 (riboflavin)
Q0583:From what is NAD derived in the PDH complex?
Vitamin B3 (niacin)
Q0584:From what is CoA derived in the PDH complex?
Vitamin B5 (pantothenate)
Q0585:What is the overall reaction in the PDH complex?
pyruvate + NAD+ + CoA --> acetyl-CoA + CO2 + NADH
Q0586:What three factors activate PDH during exercise?
1. increase in NAD+/NADH ratio;2. increase in ADP ratio;3.
increase in Ca2+
Q0587:PDH complex is similar to what other complex by
having the same cofactors; similar substrate; and similar
action?
PDH is similar to alpha-KG DH complex
Q0588:What enzyme deficiency cause cause lactic acidosis?
PDH complex deficiency from a backup of pyruvate and
alanine
Q0589:Alcoholism with a Vitamin B1 deficiency can also
cause what (besides Wernicke-Korsakoffe)?
PDH deficiency (B1 is a co-factor)
Q0590:What are the findings in PDH complex deficiency?
neurologic deficits
Q0591:What is the treatment for PDH complex deficiency?
1. increase intake of KETOGENIC nutrients (high fat
content);2. increase intake of LEUCINE and LYSINE
Q0592:What four items can pyruvate be converted into?
1. alanine;2. oxaloacetate;3. acetyl-Coa;4. lactate
Q0593:How can OAA be used after it is converted from
pyruvate?
1. replenish TCA cycle;2. gluconeogenesis
Q0594:What enzyme converts pyruvate into alanine?
ALT
Q0595:What enzyme converts pyruvate into OAA?
pyruvate carboxylase (using CO2 + ATP)
Q0596:What enzyme converts pyruvate into Acetyl-CoA?
PDH (using NAD+ and releasing CO2)
Q0597:What enzyme converts pyruvate into lactate in the
cytosol?
LDH (using NADH)
Q0598:What is the purpose of the Cori cycle?
Cori cycle transfers excess reducing equivalents from RBCs
and muscle --> liver; allowing muscle to function
anaerobically.
Q0599:In the TCA cycle; what are the products per one
acetyl CoA?
1. 3 NADH;2. 1 FADH2;3. 2 CO2;4. 1 GTP
Q0600:How many ATP are produced from a single acetyl-
Coa in the TCA cycle?
12 ATP/acetyl-Coa in the TCA cycle
Q0601:How many ATP are produced from a single glucose
molecule in the TCA cycle?
24 ATP
Q0602:In the TCA cycle; what are the products per one
glucose molecule?
1. 6 NADH;2. 2 FADH2;3. 4 CO2;4. 2 GTP
Q0603:What enzyme converts pyruvate into Acetyl-Coa?
PDH in glycolysis
Q0604:What three factors inhibit PDH?
1. ATP;2. Acetyl-Coa;3. NADH
Q0605:What factor inhibits Citrate synthase?
ATP
Q0606:wWhat enzyme converts Acetyl-CoA + OAA -->
citrate?
citrate synthase in the TCA cycle
Q0607:What enzyme converts Isocitrate into alpha-KG?
Isocitrate dehydrogenase
Q0608:What 2 factors negatively inhibit Isocitrate DH?
1. ATP;2. NADH
Q0609:What factor positively affects Isocitrate DH?
ADP
Q0610:What two molecules are released in the conversion of
Isocitrate into alpha-KG?
1. CO2;2. NADH
Q0611:what enzyme converts alpha-KG into Succinyl-CoA
alpha-KG DH
Q0612:What two molecules are released in the conversion of
alpha-KG into Succinyl CoA?
1. CO2;2. NADH
Q0613:What 3 factors negatively inhibit alpha-KG?
1. Succinyl-CoA;2. NADH;3. ATP
Q0614:What 2 molecules are released in the conversion of
Succinyl-CoA --> Succinate?
1. GTP;2. CoA
Q0615:What molecule is released in the conversion of
Succinate --> Fumarate?
FADH2
Q0616:What molecule is released in the conversion of malate
into OAA?
NADH
Q0617:1 NADH yields how many ATP?
3 ATP per 1 NADH
Q0618:1 FADH2 yields how many ATP?
2 ATP per 1 FADH2
Q0619:Name 4 electron transport inhibitors:
1. rotenone;2. antimycin A;3. CN-;4. CO
Q0620:What is the end result of electron transport inhibition?
1. decrease in proton gradient;2. block of ATP synthesis
Q0621:What is an example of a mitochondrial ATPase
inhibitor?
Oligomycin
Q0622:The enzymes for gluconeogenesis are located in what
organs only?
1. liver;2. kidney;3. intestinal epithelium
Q0623:Can muscle participate in gluconeogenesis?
NO
Q0624:The pentose phosphate pathway (HMP Shunt)
produces [;] from G6P for nucleotide synthesis
ribose-5-P
Q0625:The Pentose Phosphate Pathway (HMP Shunt)
produces [;] from [;] for FA and steroid biosynthesis and for
maintaining reduced glutathione inside RBCs.
NADPH from NADP+
Q0626:All rxns in the HMP Shunt ocur in the [;].
cytoplasm
Q0627:[;] ATP is used or produced in the HMP Shunt.
NO
Q0628:What are the organs involved in the HMP Shunt
(Pentose Phosphate Pathway)?
1. lactating mammary glands;2. liver;3. adrenal cortex;4. all
sites of FA or steroid synthesis
Q0629:[;] is the rate-limiting enzyme in the HMP shunt
G6PD
Q0630:Hemolytic anemia is caused by a decrease in [;] in
RBCs due to poor RBC defense against oxidizing agents.
NADPH
Q0631:What are the oxidizing agents involved in hemolytic
anemia due to a G6PD deficiency?
1. fava beans;2. sulfonamide;3. primaquine;4. Anti-TB drugs
Q0632:What are Heinz bodies?
altered H.emoglobin precipitates within RBCs
Q0633:What is the inheritance pattern of G6PDH deficiency?
X-linked recessive
Q0634:Glucose-6-Phosphate DH converts G6P and NADP+
into what?
1. 6-PG;2. NADPH
Q0635:Glutathion reductase converts NADPH and oxidized
GS-SG into what?
1. NADP+;2. 2 GSH (reduced)
Q0636:Hydrogen peroxide reacts with what to produc GS-SG
(oxidized) + 2 H2O?
2 GSH (reduced)
Q0637:What enzyme is associated with Essential fructosuria?
Fructokinase
Q0638:What enzyme is associated with Fructose intolerance?
Aldolase B
Q0639:What is the end result of Fructose intolerance?
1. Fructose-1-phosphate accumulates;2. DECREASE in
available phosphate;3. INHIBITION of
GLYCOGENOLYSIS and GLUCONEOGENSIS
Q0640:What are the symptoms of hereditary aldolase B
deficiency (Fructose intolerance)?
1. hypoglycemia;2. jaundice;3. cirrhosis;4. vomiting
Q0641:What is the treatment for Fructose intolerance?
1. DECREASE intake of fructose;2. DECREASE intake of
sucrose (glucose + FRUCTOSE)
Q0642:Aldolase B converts Fructose-1-P into what 2
products?
1. DHAP;2. glyceraldehyde
Q0643:What enzyme converts Glyceraldehyde into
Glyceraldehyde-3-P?
Triose kinase
Q0644:What enzyme converts Galactose-1-P to Glucose-1-P?
Galactose-1-phosphate uridyltransferase
Q0645:Galactosemia is caused by the absence of what
enzyme?
Galactose-1-phosphate uridyl transferase
Q0646:What are the symptoms of galactosemia?
1. cataracts;2. hepatosplenomegaly;3. mental retardation
Q0647:What is the treatment of galactolsemia?
1. EXCLUDE galactose;2. EXCLUDE LACTOSE (galactose
+ glucose) from diet
Q0648:What causes the symptoms of galactosemia?
accumulation of toxic substances (galactitol)
Q0649:What enzyme converts UDP-galactose back into
UDP-glucose?
4-epimerase
Q0650:What is the mnemonic for all essential amino acids?
P.riV.aT.e T.I.M. H.A.L.L.
Q0651:What are the glucogenic/ketogenic essential amino
acids?
1. P.henylalanine;2. I.le;3. T.ryptophan;"Gluco/ketogenic is
the P.I.T.s"
Q0652:What are the Glucogenic essential amino acids?
1. M.ethionine;2. T.hreonine;3. V.aline;4. A.rginine;5.
H.istidine;"MTV? AH!"
Q0653:What essential amino acids are required during growth?
1. Arginine;2. Histidine;both increase GH
Q0654:What basic amino acid has no net charge at body pH?
Histidine
Q0655:What is the most basic AA?
Arginine
Q0656:What 2 amino acids are found in histones?
1. Arginine;2. Lysine;(both have an extra NH3 group)
Q0657:What is formed in the conversion of glutamate -->
alpha-KG?
NADPH
Q0658:The Urea Cycle degrades [;] into amino groups.
amino acids
Q0659:What accounts for 90% of nitrogen in the urine?
Urea Cycle
Q0660:In what organ does the Urea Cycle occur?
liver
Q0661:In what organelle does carbamoyl phosphate
incorporation occur?
mitochondria
Q0662:Where do the remaining steps of the Urea Cycle occur;
besides the mitochondria?
cytosol
Q0663:What is released in the conversion of Arginine -->
Ornithine?
Urea
Q0664:Tryptophan is used to form what 3 things?
1. Niacin;2. Serotonin;3. Melatonin
Q0665:Glycine is used to form what?
glycine --> porphyrin --> heme
Q0666:Arginine is used to form what?
1. Creatine;2. Urea;3. Nitric oxide
Q0667:In PKU; what constituents(2) are deficient?
1. phenylalanine hydroxylase;2. tetrahydrobiopterin cofactor
Q0668:What are the findings (5) in PKU?
1. MR;2. growth retardation;3. fair skin;4. eczema;5. musty
body odor
Q0669:What is the R(x) for PKU?
1. DECREASE Phe;2. INCREASE Tyr in diet
Q0670:What are the 3 phenyllactones that accumulate in
PKU?
1. phenylacetate;2. phenyllactate;3. phenylpyruvate
Q0671:What is the incidence of PKU?
1/10;000
Q0672:What enzyme converts Phe --> Tyr?
Phenylalanine hydroxylase
Q0673:What enzyme converts DHB --> THB and restores
NADP+?
dihydropterin reductase
Q0674:What are the 2 possible causes of albinism?
1. deficiency of TYROSINASE (inability to synthesize
malanin from tyrosine);2. Defective tyrosine transporters
(DECREASE amounts of tyrosine and thus melanin)
Q0675:[;] can result from a lack of migration of neural crest
cells
Albinism
Q0676:Full-term neonate of uneventful delivery becomes
mentally retarded and hyperactive and has a musty odor.
What is the D(x)?
PKU
Q0677:Stressed executive comes home from work; consumes
7 or 8 martinis in rapid succession before dinner; and becomes
hypoglycemic. What is the mechanism?
NADH increase prevents gluconeogenesis by shunting
pyruvate and OAA to lactate and malate.
Q0678:2-year-old girl has an increase in abdominal girth;
failure to thrive; and skin and hair depigmentation. What is the
D(x)?
Kwashiorkor
Q0679:Alcoholic develops a rash; diarrhea; and altered mental
status. What is the vitamin deficiency?
Vitamin B3 (pellagra)
Q0680:51-year-old man has black spots in his sclera and has
noted that his urine turns black uon standing. What is the
D(x)?
Akaptonuria
Q0681:25-year-old male complains of severe chest pain and
has xanthomas of his Achilles tendon. What is the disease; and
where is the defect?
Familial hypercholesterolemia; LDL receptor.
Q0682:What is the definition of UNAMBIGUOUS when
describing the genetic code?
each codon specifies only 1 AA
Q0683:What is the definition of Degenerate when describing
the genetic code?
more than 1 codon may code for the same AA
Q0684:Why organism does NOT have a commaless;
nonoverlapping genetic code?
viruses
Q0685:What are the EXCEPTIONS to a universal genetic
code?
1. mitochondria;2. archaeobacteria;3. Mycoplasma;4. yeasts
(some)
Q0686:[;] makes an RNA primer on which DNA polymerase
III can initiate replication in PROKARYOTIC DNA
replication.
Primase
Q0687:[;] degrades the RNA primer in PROKARYOTIC
DNA replication.
DNA polymerase I
Q0688:DNA polymerase III has [;] synthesis and proofreads
with [;] exonuclease
5'--> 3' synthesis; 3' --> 5' exonuclease (DNA polymerase III
for PROKARYOTES)
Q0689:In PROKARYOTIC DNA replication; DNA
polymerase I excises the RNA primer with a [;] exonuclease
5' --> 3'
Q0690:Where does replication begin for Eurkaryotic DNA
polymerases?
consensus sequences of AT base pairs.
Q0691:What is the function of Eukaryotic DNA polymerase
alpha?
synthesize RNA PRIMERS
Q0692:What is the function of Eukaryotic DNA polymerase
beta?
LEADING-strand DNA
Q0693:What is the function of Eukaryotic DNA polymerase
gamma?
LAGGING-strand DNA
Q0694:What is the function of Eukaryotic DNA polymerase
delta?
MITOCHONDRIAL DNA
Q0695:What is the function of Eukaryotic DNA polymerase
epsilon?
DNA repair
Q0696:X-rays can damage DNA; and a repair defect can cause
what?
ataxia-telangiectasia
Q0697:Radiation can damage DNA; and a repair defect can
cause what?
Bloom's syndrome
Q0698:Cross-linking agents can damage DNA; and a repair
defect can cause what?
Fanconi's anemia
Q0699:DNA; RNA; and protein are all synthesized in what
direction?
5' --> 3'
Q0700:AA's are linked [;] to [;]
N --> C
Q0701:What are the types of RNA polymerases for
EUKARYOTES?
1. RNA POLYMERASE I;2. RNA POLYMERASE II;3.
RNA POLYMERASE III
Q0702:Do RNA polymerases have proofreading function?
NO
Q0703:Alpha-amantin inhibits which RNA polymerase?
RNA polymerase II
Q0704:Where does RNA polymerase II bind?
promotor site of DNA
Q0705:In Prokaryotes; does RNA polymerase make all 3
kinds of RNA?
yes
Q0706:What binds to a PROMOTOR site?
1. RNA polymerase;2. transcription factors;(UPSTREAM
FROM THE GENE)
Q0707:What binds to an ENHANCER site?
transcription factors
Q0708:What binds to an OPERATOR?
repressors (a repressive operator)
Q0709:Only [;] RNA is transported out of the nucleus
processed
Q0710:The [;] the Km; the higher the affinity.
lower
Q0711:The S phase of the cell cycle involves what?
Synthesis of DNA
Q0712:The G0 phase in the cell cycle is a quiescent [;] phase
G1 phase
Q0713:In the cell cycle; [;] is the shortest phase
mitosis
Q0714:Most cells are in what phase?
Go
Q0715:RER does what 2 things?f
1. synthesis of secretory (exported) proteins;2. N-linked
oligosaccharide addition to many proteins
Q0716:What are the major functions of the Golgi?
1. MODIFIES N-oligosaccharides on asparagiNe;2. ADDS O-
oligosaccharides to serine and threOnine;3. sulfation of sugars
on proteoglycans;4. sulfation of Tyrosine;5. ADDITION of
mannose-6-phosphate to lysosomal proteins; which targets
the protein to the lysosome.
Q0717:What are the symptoms of I-cell disease?
1. coarse facial features;2. restricted joint movement
Q0718:What are the 3 key features of microtubules?
1. helical;2. alpha + beta tubulin dimers (2 GTP bound
each);3. forms flagella; cilia; and mitotic spindles
Q0719:What are 5 drugs that act on microtubules?
1. Mebendazole/thiabendazole;2. Taxol;3. Griseofulvin;4.
Vincristine/vinblastine;5. Colchicine
Q0720:Chediak-Higashi syndrome is due to a microtubule
polymerization defect; resulting in a DECREASE in [;]
phagocytosis
Q0721:What are the 2 key features of Cilia?
1. 9 + 2 arrangment of microtubules (9 doublets);2. doublets
linked by Dynein; an ATPase
Q0722:Kartagener's syndrome is due to a dynein arm defect;
resulting in [;] cilia.
immotile cilia
Q0723:What 2 components in the plasma cell membrane can
INCREASE the melting temperature?
1. cholestrol;2. long saturated fatty acids
Q0724:Name 5 functions of Phosphatidylcholine:
1. RBCs;2. myelin;3. bile;4. surfactant (DiPalmitoyl
Phosphatidyl Choline);5. esterification of cholesterol (LCAT)
Q0725:Ouabain INHIBITS the Na+/K+ pump by binding to
what?
K+ site
Q0726:What is the most abundant protein in the human
body?
collagen
Q0727:What are the components of Type I collagen?
1. B.one;2. tendon;3. skin;4. dentin;5. fascia;6. cornea;7. late-
wound repair
Q0728:What are the components of Type II collagen?
1. C.artilage ("Type II: carTWOlage"); hyaline too;2. vitreous
body;3. nucleus pulposus
Q0729:What are the components of Type III collagen?
1. R.eticulin;2. skin;3. blood vessels;4. uterus;5. fetal tissue;6.
granulation tissue
Q0730:What are the components of Type IV collagen?
1. B.asement membrane;2. basal lamina "Type IV: under the
FLOOR (basement membrane)"
Q0731:What is the component of Type X collagen
epiphyseal plate
Q0732:What is the mnemonic for the first four collagen types
(I-IV)?
"B.e C.ool; R.ead B.ooks"
Q0733:What is the 1st step in collagen synthesis INSIDE
fibroblasts?
collagen alpha chains (PREPROCOLLAGEN) translated on
RER--usually Gly-X-Y polypeptide (X and Y are proline;
hydroxyproline; or hydroxylysine)
Q0734:What is the 2nd step in collagen synthesis INSIDE
fibroblasts?
ER--> hydroxylation of specific proline and lysine residues
(requires vitamin C)
Q0735:What is the 3rd step in collagen synthesis INSIDE
fibroblasts?
Golgi --> glycosylation of pro-alpha-chain lysine residues and
formation of PROCOLLAGEN(triple helix of 3 collagen alpha
chains)
Q0736:What is the 4th step in collagen synthesis INSIDE
fibroblasts?
PROCOLLAGEN molecules are exocytosed into the
extracellular space
Q0737:What is the 5th step in collagen synthesis OUTSIDE
fibroblasts?
PROCOLLAGEN peptidases cleave terminal regionals of
PROCOLLAGEN; transforming PROCOLLAGEN into
insoluble TROPOCOLLAGEN
Q0738:What is the 6th and last step in colagen synthesis
OUTSIDE fibroblasts?
staggered TROPOCOLLAGEN molecules are reinforced by
covalent lysine-hydroxylysine cross-linkage (by lysyl
oxidase) to make COLLAGEN FIBRILS
Q0739:What are the 8 major points concerning Ehlers-Danlos
syndrome?
1. faulty collagen synthesis;2. hyper-extensible skin;3. easy
bleeding/brusing;4. hypermobile joints;5. berry aneurysms;6.
type III collagen (reticulin: blood vessels; skin);7. mitral valve
prolapse;8. CAN'T make COLLAGEN FIBRILS from
TROPOCOLLAGEN!
Q0740:What are the 9 major points concerning
OSTEOGENESIS IMPERFECTA?
1. AUTOSOMAL DOMINANT (UNIQUE);2. faulty
collagen synthesis;3. brittle bone disease;4. translucency of
CT over choroid (blue sclerae);5. hearing loss: abnormal
middle ear bones;6. lack of dentition;7. Type II OI: fatal;8.
Indicence of OI: 1/10;000;9. CAN'T make PROCOLLAGEN
from PREPROCOLLAGEN
Q0741:What three metabolic processes occur in the
mitochondria?
1. B.eta-oxidation;2. A.cetyl-CoA production;3. K.rebs cycle
Q0742:What five metabolic processes occur in the
cytoplasm?
1. glycolysis;2. FA synthesis;3. protein synthesis;4. steroid
synthesis;5. HMP shunt
Q0743:What 2 metabolic processes occur in BOTH the
mitochondria and cytoplasm?
1. H.eme synthesis;2. U.rea cycle;3.
G.luconeogenesis;"H.U.G. both the mitochondria and
cytoplasm for their metabolism."
Q0744:A deficiency of what enzyme causes MILD
galactosemia?
Galactokinase
Q0745:A deficiency of what enzyme causes SEVERE
galactosemia?
Galactose-1-phosphate uridyltransferase
Q0746:Galactose-1-phosphate --> Glucose-1-phosphate by
what enzyme?
Galactose-1-phosphate uridyltransferase
Q0747:A deficiency of what enzyme causes Von Gierke's
disease?
Glucose-6-phosphatase
Q0748:Glucose-6-phosphate --> 6-phosphogluconolactone
by what enzyme?
Glucose-6-phosphate dehydrogenase (G6PD)
Q0749:Hemolytic anemia is caused by a deficiency of what
enzyme?
G6PD
Q0750:Ribulose-5-phosphate --> fructose-6-phosphate by
what enzyme?
transketolase
Q0751:A deficiency of what enzyme causes ESSENTIAL
fructosuria?
fructokinase
Q0752:A deficiency of what enzyme causes fructose
INTOLERANCE?
Aldolase B
Q0753:F1P --> DHAP + Glyceraldehyde. What enzyme?
aldolase B
Q0754:PEP --> pyruvate. What enzyme?
pyruvate kinase
Q0755:Pyruvate --> Acetyl-CoA. What enzyme?
pyruvate dehydrogenase
Q0756:Acetyl-CoA --> Malonyl-CoA. What cofactor?
biotin to tranfer CO2
Q0757:HMG CoA --> mevalonate. What enzyme?
HMG-CoA reductase
Q0758:pyruvate --> OAA. What enzyme?
pyruvate carboxylase
Q0759:OAA --> PEP. What enzyme?
PEP carboxykinase
Q0760:Acetyl-CoA + OAA --> citrate. What enzyme?
citrate synthase
Q0761:alpha-KG --> Succinyl-CoA. What enzyme?
alpha-ketoglutarate dehydrogenase
Q0762:Ornithine + Carbamoyl phosphate --> citrulline. What
enzyme?
ornithine transcarbamylase
Q0763:Aerobic metabolism of glucose --> 38 ATP via [;]
malate shuttle
Q0764:Aerobic metabolism of glucose --> 36 ATP via [;]
G3P shuttle
Q0765:What are 2 activated ACYL carriers?
1. coenzyme A;2. lipoamide
Q0766:What is an activated CO2 carrier?
biotin
Q0767:What is an activated 1-carbon unit carrier?
tetrahydrofolate
Q0768:What is an activated carrier of aldehydes?
TPP
Q0769:What is an activated carrier of choline?
CDP-choline
Q0770:ATP + methionine --> SAM. using what cofactor?
B12
Q0771:NADPH used in 3 processes:
1. anabolic processes;2. respiratory burst;3. p-450
Q0772:What enzymes involve NADPH in respiratory burst?
1. NADPH oxidase;2. glutathione reductase;3. Glucose-6-
Phosphate dehydrogenase
Q0773:Glucose -> G6P; an irreversible regulatory step in
glycolysis is catalyzed by which enyzme?
glucokinase/hexokinase
Q0774:Fructose 6-phosphate -> F1;6BP; an irreversible
regulatory step in glycolysis is catalyzed by which enyzme?
Phosphofructokinase (PFK)
Q0775:Phosphoenolpyruvate (PEP)->pyruvate; an
irreversible regulatory step in glycolysis is catalyzed by
which enyzme?
pyruvate kinase
Q0776:Acetyl CoA --> Citrate; an irreversible regulatory step
in the TCA cycle is catalyzed by which enyzme?
citrate synthase
Q0777:a-ketoglutarate -> succinate; an irreversible regulatory
step in the TCA cycle is catalyzed by which enyzme?
a-ketoglutarate dehydrogenase
Q0778:How many ATP are produced from one glucose
molecule in anaerobic glycolysis?
2 ATP produced
Q0779:How many ATP are produced from one glucose
molecule in aerobic metabolism?
38ATP from malate shuttle;36 ATP from Glucose 3
phosphate shuttle
Q0780:What is the product of the hexose monophospate
shunt (HMP)?
NADPH used in anabolic processes (steroid and fatty acid
synthesis) and ribose 5-phosphate for nucleotide synthesis
Q0781:What molecules are universal electron acceptors?
Nicotinamides (NAD; NADP);Flavin nucleotides (FAD)
Q0782:Where is hexokinase found?
ubiquitous
Q0783:What are the kinetic characteristics of hexokinase in
relation to glucose?
high affinity; low capacity
Q0784:What product inhibits hexokinase?
Glucose 6-Phosphate
Q0785:Where does one find glucokinase?
in the liver
Q0786:What are the kinetic characteristics of glucokinase in
relation to glucose?
low affinity; high capacity
Q0787:Where does glycolysis occur in the cell?
Cytoplasm
Q0788:Which enzyme is activated in the fasting state
converting fructose-6-phosphate to fructose 2;6-
bisphosphate?
PFK2
Q0789:Which enzyme is activated in the fed state converting
fructose-2;6-bisphosphatase to fructose 6-phosphatate?
fructose bisphosphatate-2
Q0790:What molecule is the most potent activator of
phosphofructokinase; converting fructose-6-phosphate to
fructose 1;6-phosphate
Fructose 2;6 BP
Q0791:A deficiency of which glycolytic enzyme is associated
with hemolytic anemia?
pyruvate kinase;G6PDH is not part of glycolysis; it is part of
the HMP shunt
Q0792:What are the only two purely ketogenic amino acids?
Lysine and Leucine
Q0793:What factors increase the activation of the pyruvate
dehydrogenase complex?
Low ATP/ADP ratio (exercise);high NAD/NADH ratio;high
IC [Ca]
Q0794:How many ATP equivalents are needed to generate
glucose from pyruvate?
6 ATP equivalents
Q0795:What are the 4 fates for pyruvate at the end of
glycolysis?
1. Alanine;2. oxaloacetate;3. Acetyl CoA;4. Lactate
Q0796:What is the function of the Cori cycle?
transfers excess reducing equivalents from RBCs and muscle
to the liver; shifts the metabolic burden to the liver
Q0797:What are the steps in the TCA cycle?
Citrate > isocitrate > a-ketoglutarate > succinyl CoA >
succinate > fumate > malate > OAA
Q0798:What cofactors are required for the a-ketoglutarate
dehydrogenase complex?
B1; B2; B3; B5; lipoic acid
Q0799:Which complexes bring protons across the inner
mitochondrial membrane?
Complexes I; III; IV
Q0800:In oxidative phosphorylation; how many ATP are
produced from 1 NADH?
3 ATP
Q0801:In oxidative phosphorylation; how many ATP are
produced from 1 FADH2?
2 ATP
Q0802:What are the three categories of oxidative
phosphorylation poisons?
1. e- transport inhibitors;2. ATPase inhibitors;3. Uncoupling
agents
Q0803:What are the 4 irreversible enzymes of
gluconeogenesis and where are they located?
1. Pyruvate carboxylase (mitochondria);2. PEP carboxykinase
(PEPCK; cytosol);3. Fructose 1;6-bisphosphatase
(cytosol);4. Glucose 6-phophotase (ER)
Q0804:What tissues contain the irreversible enzymes of
gluconeogenesis?
liver; kidney; intestinal epithelium;muscle does not contain
G6Ptase and cannot participate in gluconeogenesis
Q0805:Deficiency of key gluconeogenic enzymes causes what
symptoms?
hypoglycemia
Q0806:The HMP shunt occurs in what parts of the body?
lactating mammary glands; liver; adrenal cortex
Q0807:In what part of the cell does the HMP shunt occur?
cytoplasm
Q0808:What enzyme is required for the irreversible reaction
of the HMP shunt producing NADPH?
Glucose-6-phosphate dehydrogenase
Q0809:What is the product of the reversible reaction of HMP
shunt?
Ribose-5-phosphate (for nucleotide synthesis) and
Glyceraldehyde 3-phosphate; fructose 6-
phosphate(intermediate of gyloslysis)
Q0810:GLUT2 receptors are found in which cells?
b-cells in the pancreas; Liver; kidney
Q0811:GLUT4 receptors are found in which cells?
Muscles and Fat
Q0812:What is the general function of insulin?
-moves glucose into cells;-inhibits glucagon secretion from a-
cells in pancreas
Q0813:Which organs do not require insulin for glucose
uptake?
Brain;RBCs;Intestine;Cornea;Kidney;Liver
Q0814:What are the anabolic effects of insulin?
increased glucose transport;increased glycogen synthesis and
storage;increased triglyceride synthesis and storage;increased
Na retention;increased protein synthesis
Q0815:What is the role of glycogen in skeletal muscle?
rapidly metabolize glucose during exercise
Q0816:What is the role of glycogen in hepatocytes?
storage depot to maintain blood sugar at appropriate levels.
Q0817:What are the main reactions of
glycogenesis/degradation?
G6P > G1P > UDP-glucose > branched version > limit
dextran > debranched glycogen
Q0818:What are the 4 glycogen storage diseases?
Von Gierke's Dz (Type I);Pompe's Dz (Type II);Cori's Dz
(Type III);McArdle's Dz (Type V)
Q0819:What are the findings of Von Gierke's Dz?
severe fasting hypoglycemia; high glycogen in the liver;
increased blood lactate; hepatomegaly
Q0820:What is the deficient enzyme in Von Gierke's Dz?
Glucose-6-phosphate
Q0821:What are the findings of Pompe's dz?
cardiomegaly and systemic findings leading to early death
Q0822:What is the deficient enzyme in Pompe's Dz?
Lysosomal a-1;4-glucosidase (acid maltase)
Q0823:What are the findings of Cori's disease?
Milder form of Van Gierke's (Type I) with normal blood
lactate levels
Q0824:What is the deficient enzyme in Cori's Dz?
debranching enzyme (a-1;6-glucosidase
Q0825:What are the findings of McArdle's dz (Type V)?
increased glycogen in muscle but cannot break it down ->
painful cramps and myoglobinuria with strenuous exercise
Q0826:What is the deficient enzyme in McArdle's Dz?
skeletal muscle phosphorylase
Q0827:A full-term neonate of uneventful delivery becomes
mentally retarded and hyperactive and has a musty odor.
What is the dx?
PKU
Q0828:A stressed executive comes home from work;
consumes 7 or 8 martinis in rapid succession before dinner;
and becomes hypoglycemic. What is the mechanism?
Increase in NADH prevents gluconeogenesis by shunting
pyruvate and oxaloacetate to lactate and malate
Q0829:A 2 year-old girl has an increase in abdominal girth;
failure to thrive; and skin and hair depigmentation. What is the
dx?
Kwashiorkor
Q0830:Alcoholic develops a rash; diarrhea; and altered mental
status. What is the Vitamin Deficiency?
Vitamin B3 (pellagra)
Q0831:A 51-year-old man has black spots in his sclera and
has noted that his urine turns black upon standing. What is his
dx?
Alkaptonuria
Q0832:A 25-year-old male complains of severe chest pain and
has xanthomas of his Achilles tendons. What is the dz and
where is the defect?
Familial hypercholesterolemia; LDL receptor
Q0833:A woman complains of intense muscle cramps and
darkened urine after exercise. What is the dx?
McArdle's Dz
Q0834:Two parents with albinismhave a sone who is normal.
Why is the son not affected?
Locus heterogeneity
Q0835:A 40-year-old man has chronic pancreatitis with
pancreatic insufficiency. What vitamins are likely deficient?
A;D;E;K
Q0836:What are the fat soluble vitamins?
A;D;E;K
Q0837:What two organs contribute most to the absorption of
fat-soluble vitamins?
gut (ileum) and pancreas
Q0838:What dzs can cause fat soluble vitamin deficiencies?
Malabsorption syndromes such as CF; celiac sprue; miner oil
intake can also cause deficiencies
Q0839:Which vitamins are water soluble?
B1; B2;B3;B5;B6;B12;C;Biotin;Folate
Q0840:Which water soluble vitamin does NOT wash out of
the body easily and why?
Vit B12 because it is stored in the liver
Q0841:What are some common symptoms of B-complex
deficiencies?
dermatitis; glossitis; and diarrhea
Q0842:What is another name for Vitamin A?
Retinol
Q0843:A deficiency in Vitamin A causes what symptoms?
night blindness; dry skin
Q0844:What is the function of Vitamin A?
constituent of visual pigments
Q0845:Excess of Vitamin A causes what symptoms?
arthralgias; fatigue; headaches; skin changes; sore throat;
alopecia
Q0846:What is another name for Vitamin B1?
thiamine
Q0847:A deficiency in Vitamin B1 causes what symptoms?
BeriBeri and Wernike-Korsakoff syndrome
Q0848:What is the function of Vitamin B1?
a cofactor for oxidative decarboxylation of a-keto acids and a
cofactor in the HMP shunt
Q0849:What is another name for Vitamin B2?
riboflavin
Q0850:A deficiency in Vitamin B2 causes what symptoms?
angular stomatitis; Cheilosis; corneal vascularization
Q0851:What is the function of Vitamin B2?
Cofactor for oxidation and reduction (e.g; FADH2)
Q0852:What is another name for Vitamin B3?
niacin
Q0853:A deficiency in Vitamin B3 causes what symptoms?
Pellagra: diarrhea; dermatitis; dementia
Q0854:What is the function of Vitamin B3?
Constituent of NAD; NADP (redox rxns); derived from
tryptophan
Q0855:What is another name for Vitamin B5?
pantothenate
Q0856:A deficiency in Vitamin B5 causes what symptoms?
dermatitis; enteritis; alopecia; adrenal insufficiency
Q0857:What is the function of Vitamin B5?
Constituent of CoA and component of FA synthase
Q0858:A deficiency in Vitamin C causes what symptoms?
Scurvy - swollen gums; bruising; anemia; poor wound healing
Q0859:What is the function of Vitamin C?
needed for the hydroxylation of proline and lysine in collagen
synthesis; keeps Fe+2 in a reduced state increaseinf Fe
absorption; cofactor for DA->NE
Q0860:A deficiency in Vitamin A causes what symptoms?
night blindness; dry skin
Q0861:What is the function of Vitamin A?
constituent of visual pigments
Q0862:Excess of Vitamin A causes what symptoms?
arthralgias; fatigue; headaches; skin changes; sore throat;
alopecia
Q0863:What is another name for Vitamin B1?
thiamine
Q0864:What is the function of Vitamin B1?
a cofactor for oxidative decarboxylation of a-keto acids and a
cofactor in the HMP shunt
Q0865:What is another name for Vitamin B2?
riboflavin
Q0866:A deficiency in Vitamin B2 causes what symptoms?
angular stomatitis; Cheilosis; corneal vascularization
Q0867:What is the function of Vitamin B2?
Cofactor for oxidation and reduction (e.g; FADH2)
Q0868:What is another name for Vitamin B3?
niacin
Q0869:A deficiency in Vitamin B3 causes what symptoms?
Pellagra: diarrhea; dermatitis; dementia
Q0870:What is the function of Vitamin B3?
Constituent of NAD; NADP (redox rxns); derived from
tryptophan
Q0871:What is another name for Vitamin B5?
pantothenate
Q0872:A deficiency in Vitamin B5 causes what symptoms?
dermatitis; enteritis; alopecia; adrenal insufficiency
Q0873:What is the function of Vitamin B5?
Constituent of CoA and component of FA synthase
Q0874:What is another name for Vitamin B6?
pyridoxine
Q0875:A deficiency in Vitamin B6 causes what symptoms?
convulsions; hyperirritability; peripheral neuropathy
Q0876:What is the function of Vitamin B6?
converted to pyridoxal phosphate; a cofactor in
transanimation; decarboxylation and heme synthesis
Q0877:What is another name for Vitamin B12?
cobalamin
Q0878:A deficiency in Vitamin B12 causes what symptoms?
macrocytic; megaloblastic anemia; neurologic symptoms;
glossitis
Q0879:What is the function of Vitamin B12?
cofactor in homocysteine methylation and methylmalonyl Co-
A handlining
Q0880:B12 is found in what types of foods?
Only animal products
Q0881:What test is used to detect a B12 deficiency?
Schilling Test
Q0882:What are the three main causes of a B12 deficiency?
Malabsorption; lack of intrinsic factor (pernicious anemia) or
absence of terminal ileum (chron's dz)
Q0883:A deficiency in folic acid causes what symptoms?
macrocytic megaloblastic anemia w/o neuro symptoms (unlike
B12)
Q0884:What is the function of Folic acid?
coenzyme for 1-carbon transfers (methylation rxns); needed
for the synthesis of nitrogenous bases in DNA and RNA
Q0885:Supplemental folic acid is given in pregnancy to
prevent what defects?
neural tube
Q0886:What is the folic acid precursor in bacteria?
PABA
Q0887:A deficiency in biotin causes what symptoms?
dermatitis; enteritis
Q0888:What is the function of biotin?
cofactor for carboxylation rxns: pyruvate -> oxaloacetate;
Acetyl CoA -> malonyl CoA; Proprionyl CoA ->
methylmalonyl CoA
Q0889:What is another name for Vitamin C?
Ascorbic acid
Q0890:A deficiency in Vitamin C causes what symptoms?
Scurvy - swollen gums; bruising; anemia; poor wound healing
Q0891:What is the function of Vitamin C?
needed for the hydroxylation of proline and lysine in collagen
synthesis; keeps Fe+2 in a reduced state increaseinf Fe
absorption; cofactor for DA->NE
Q0892:A deficiency in Vitamin D causes what symptoms?
Rickets in children and osteomalacia in adults (improper bone
mineralization); hypocalcemic tetany
Q0893:What is the function of Vitamin D?
Increased absorption of Ca and P in the gut
Q0894:What is the mechanism by which Vitamin D
deficiency causes tetany?
less D -> less Ca -> lowering the membrane potential of a cell
-> making it easier to get to threshold for AP
Q0895:Excess of Vitamin D causes what symptoms?
Hypercalcemia; stupor; lossof appetite
Q0896:A deficiency in Vitamin E causes what symptoms?
Increased fragility of erythrocytes; neurodysfunction
Q0897:What is the function of Vitamin E?
Antioxidant: protects erythrocytes from hemolysis
Q0898:A deficiency in Vitamin K causes what symptoms?
Neonatal hemorrhage with increased PT and PTT; but normal
bleeding time (neonates unable to synthesize Vit K)
Q0899:What is the function of Vitamin K?
Catalyzes gamma-carboxylation of glutamic acid residues on
various proteins concerned with clotting; synthesized by
intestinal flora
Q0900:What can cause a Vitamin K deficiency?
braod spectrum antibiotics (killing intestinal flora)
Q0901:What are the Vitamin K dependent clotting factors?
II; VII; IX; X; protein C and S
Q0902:Which drug is a Vitamin K antagonist?
Warfarin
Q0903:A deficiency in zinc causes what symptoms?
delayed wound healing; hypogonadism; decreased adult hair
Q0904:Outline the pathway of ethanol metabolism.
ethanol ->[alcohol dehydrogenase] -> Acetaldehyde -
>[acetaldehyde dehydrogenase]-> acetate
Q0905:What is the limiting reagent in the ethanol metabolism
pathway?
NAD+
Q0906:What are the pharmacokinetics of alcohol
dehydrogenase?
zero-order
Q0907:Which drug inhibits acetaldehyde dehydrogenase
allowing for the accumulation of acetaldehyde and increasing
hangover symptoms?
Disulfiram (anabuse)
Q0908:Describe the mechainsm for ethanol hypoglycemia in
chronic alcoholics.
Ethanol metabolism -> increased NADH/NAD+ ratio in liver
-> pyruvate diverts to lactate and OAA diverts to malate ->
inhibition of gluconeogenesis
Q0909:Kwashiorkor is malnutrition resulting from what
deficiency?
protein
Q0910:What does a pt with Kwashiorkor look like?
small child with a swollen belly
Q0911:Marasmus is a malnutrition syndrome resulting from
what deficiency?
calories/energy
Q0912:What does a pt with marasmus look like?
small child with tissue and muscle wasting
Q0913:Describe the structure of chromatin.
(-)charged DNA loops choice around nucleosome core to form
a nucleosome bead; H1 ties the nucleosomes together in a
string
Q0914:Which is the only histone that is not in the
nucleosome core?
H1
Q0915:Which form of chromatin is transcriptionally inactive?
Active?
Inactive: heterochromatin Active: Euchromatin
Q0916:Which amino acids are necessary for purine synthesis?
Glycine; Aspartate; Glutamine
Q0917:Which nucleotide bonds are stronger and what is the
consequence of this?
G-C bonds are stronger (3 H-bonds) resulting in a higher
melting temperature
Q0918:In regards to nucleotides; what is transition?
substitution of a purine for a purine or pyrimidine for
pyrimidine
Q0919:In regards to nucleotides; what is transversion?
substituting purine for pyrimidine or vice versa
Q0920:What are the four main features of the genetic code?
unambiguous; degenerate; nonoverlapping; universal
Q0921:What does it mean to say that the genetic code is
degenerate?
more than one codon may code for the same amino acid
Q0922:What is the mechanism of base excision repair?
Glycosylases remove damaged bases; endonuclease cuts DNA
at apyrimidinic site; sugar is removed; gap is filled and
resealed
Q0923:What is the mechanism of mismatch repair?
unmethylated; newly synthesized string is recognized;
mismatched nucleotides are removed; gap is filled and resealed
Q0924:Which DNA repair mechanism is mutated in
hereditary nonpolyposis colon cancer?
mismatch repair
Q0925:What is the mechanism of nonhomologous end joining?
bringing together two ends of DNA fragments
Q0926:What is the direction of DNA/RNA/protein
synthesis?
5' -> 3'
Q0927:How are amino acids joined?
N to C
Q0928:What are the three types of RNA?
mRNA; tRNA; rRNA
Q0929:Which type of RNA is the most abundant?
rRNA
Q0930:Which type of RNA is the largest?
mRNA
Q0931:Which type of RNA is the smallest?
tRNA
Q0932:What is the function of RNA pol-I?
makes rRNA
Q0933:What is the function of RNA pol-II?
Makes mRNA
Q0934:What is the function of RNA pol-III?
makes tRNA
Q0935:What substance; found in death cap mushrooms;
inhibits RNA pol-II?
a-amanitin
Q0936:Which codon codes for methionine; thus initiating
mRNA?
AUG
Q0937:What are the three stop codons?
UAA; UAG; UGA
Q0938:Which phase of the cell cycle is the shortest?
Mitosis
Q0939:In which phase is new DNA synthesized?
S phase
Q0940:What type of cells remain in Go and are regenerated
from stem cells?
Permanent cells such as neurons; skeletal and cardiac muscle;
RBCs
Q0941:What type of cells enter G1 from Go when
stimulated?
Stable cells such as lymphocytes and hepatocytes
Q0942:What type of cells never go to Go and divide rapidly
with a short G1?
Labile cells such as bone marrow; gut epithelium; skin; and
hair follicles
Q0943:What is the function of the rough ER (RER)?
synthesis of secretory (exported) proteins and N-linked
oligosaccharide addition to many proteins
Q0944:What type of cells are rich in RER?
Mucus-secreting goblet cells of the small intestine and
antibody-secreting plasma cells
Q0945:What is the function of Nissl bodies in neurons?
synthesize enzymes (e.g; ChAT) and peptide
neurotransmitters
Q0946:What is the function of the smooth ER (SER)?
site of steroid synthesis and detoxification of drugs and
poisons
Q0947:What type of cells are rich in SER?
liver hepatocytes and steroid hormone-producing cells of the
adrenal cortex
Q0948:What are the 6 main functions of the Golgi apparatus?
taking protwins and lipids from the ER to the
PM/lysosomes/secretory vesicles; 2. Modifies N-oligosac. On
asparagine; 3. adds O-oligosac to Ser and Thr; 4. addd
mannose-6P to lysosomal proteins (targeting to lysosome); 5.
prtoeoglycan assembly; 6. sulfation of sugar on proteoglycans
Q0949:What is the pathophys of I-cell disease and what are
the consequences?
mannose-6P cannot be added to the lysosomal proteins so
enzymes are secreted out of the cell instead of being targeted
to the lysosome
Q0950:What are the characteristics of I-cell disease?
coarse facial features; clouded corneas; restricted joint
movement; high plasma levels of lysosomal enzymes. Can be
fatal in childhood
Q0951:What are the three main cvesicular trafficking proteins
and where do they go?
1. COP-I: retrograde; Golgi -> ER; 2. COP-II anterograde;
RER -> cis-Golgi; 3.Clathrin: trans-Golgi->lysosomes; plasma
membrane->endosomes (receptor mediated endocytosis)
Q0952:In what cellular structures would one find
microtubules?
flagella; cilia; mitotic spindles
Q0953:What are the four main drugs that act on microtubules
and for what dz?
1. Mebendazole/thiabendazole (antihelminthic); 2. Taxol (anti-
breast ca); 3. Grisofulvin (antifungal); 4.
Vineristine/vinblastine (anti-ca); 5. Colchicine (anti-gout)
Q0954:What syndrome is caused by a defect in microtubule
polymerization resulting in decreased phagocytosis?
Chediak-Higashi
Q0955:Describe the structure of cilia.
9+2 arrangement of microtubules; doublets linked by dynein
ATPase and allows for the bending of cilia
Q0956:Which protein is responsible for retrograde motion of
cilia? Anterograde?
retrograde = dynein; anterograde = kinesin
Q0957:What is Kartagener's syndrome?
immobile cilia due to a dynein arm defect
Q0958:What are the symptoms of Kartagener's syndrome?
infertility in both males and females; bronchiectasis; recurrent
sinitus (any place where cilia are moving things around)
Q0959:What are the two most abundant components of the
plasma membrane?
cholesterol and phospholipids
Q0960:What is the major component of RBC membranes;
myelin; bile and surfactant?
phosphatidylcholine (lecithin)
Q0961:What is the NA/ K exchange ratio in a Na/K ATPase
pump?
3 Na out:2 K in
Q0962:Explain when in the exchange is the Na/K ATPase
pump phosphorylated/dephos?
Phosphorylated to let Na out (ATP->ADP) and
dephosphorylated to let K in
Q0963:What is the most abundant protein in the human
body?
collagen
Q0964:90% of all collagen is of what type?
Type I
Q0965:Type I collagen provides support for which
organs/functions?
Bone; Skin; Tendon; dentin; fascia; cornea; late wound repair
Q0966:Type II collagen provides support for which
organs/functions?
Cartilage; vitreous body; nucleus pulposus
Q0967:Type III collagen provides support for which
organs/functions?
Reticulin: skin; blood vessels; uterus; fetal tissue; granulation
tissue
Q0968:Type IV collagen provides support for which
organs/functions?
Basement membrane or basa lamina
Q0969:What are the four phases of collagen synthesis in the
fibroblasts and where do they take place?
1. synthesis (RER); 2. hydroxylation (ER); 3. glycosylation
(Golgi); 4. exocytosis as procollagen
Q0970:What are the two phases of collagen synthesis that
occur outside of the fibroblasts?
1. proteolytic processing (procollagen ->tropocollagen); 2.
cross-linking by covalent Lys-hydroxylysine = collagen fibrils
Q0971:What stage of collagen synthesis requires Vitamin C?
hydroxylation in the ER
Q0972:What are the three signs of Ehlers-Danlos syndrome?
1. hyperextensible skin; 2. tendency to bleed; 3. hypermobile
joints (faulty collagen synthesis)
Q0973:What type of collagen is most affected by Ehlers-
Danlo syndrome?
Type III (blood vessel instability)
Q0974:What is the most common form of osteogenesis
imperfecta?
AD inheritance with abnl Type I synthesis
Q0975:What are the key clinical features of osteogenesis
imperfecta?
1. multiple fractures from minimal trauma (brittle bone); 2.
blue sclerae (translucency of connective tissue over the
choroid); 3. hearing loss; 4. dental imperfections
Q0976:What may osteogenesis imperfecta be confused with
upon examination?
child abuse
Q0977:For the following cell type; state the
immunohistochemical stain used to see it: connective tissue
Vimentin
Q0978:For the following cell type; state the
immunohistochemical stain used to see it: Muscle
Desmin
Q0979:For the following cell type; state the
immunohistochemical stain used to see it: Epithelial cells
cytokeratin
Q0980:For the following cell type; state the
immunohistochemical stain used to see it: Neuroglia
glial fibrillary acid proteins (GFAP)
Q0981:For the following cell type; state the
immunohistochemical stain used to see it: neurons
neurofilaments
Q0982:In what structures does one find elastin?
lungs; large arteries; elastic ligaments
Q0983:Elastin is rich in which two amino acids?
Proline and lysine
Q0984:Marfan syndrome is caused by a defect in which
component of elastin?
fibrillin
Q0985:Elastase is inhibited in which disease? (Hint: it also
causes early-onset emphysema)
a1-antitrypsin deficiency
Q0986:What syndrome is caused by a defect in microtubule
polymerization resulting in decreased phagocytosis?
Chediak-Higashi
Q0987:Fructose intolerance is a hereditary deficiency of what
enzyme?
Aldolase B
Q0988:What is the pathophys of fructose intolerance?
fructose-1-P accumulates causing decreased available
phosphate resulting in inhibition of glycogenolysis and
gluconeogenesis
Q0989:What is the tx for fructose intolerance?
decrease intake of both fructose and sucrose (glucose and
fructose)
Q0990:Essential fructosuria is a defect in which enzyme?
fructokinase
Q0991:Galactosemia results from the absence in what
enzyme?
galactose-1-phosphate uridyltransferase
Q0992:What is the pattern of inheritance in galactosemia?
AR
Q0993:What are the symptoms of galactosemia?
cataracts; hepatosplenomegaly; mental retardation
Q0994:Lactase deficiency is a hereditary lactose intolerance
due to a loss of what type of enzyme?
Brush border enzyme
Q0995:What demographic is most susceptible to lactase
deficiency and what are the sx?
blacks; Asians; bloating; cramps; osmotic diarrhea
Q0996:What are the essential amino acids?
Phe; Val; Trp; Thr; Ile; Met; His; Arg; Leu; Lys
Q0997:what is the pathophys of hyperammonemia?
excess NH4 depletes a-ketoglutarate; leading to inhibition of
TCA cycle
Q0998:What are the sx of ammonia intoxication?
tremor; slurring of speech; somnolence; vomiting; cerebral
edema; blurring vision
Q0999:What are the three main derivatives of phenylalanine?
Dopamine; NE; Epi
Q1000:What are the three main derivatives of tryptophan?
Niacin; serotonin; melatonin
Q1001:What causes the musty odor of PKU?
disorder of excess aromatic amino acids
Q1002:Variable inheritance of albinism is due to what genetic
mechanism?
locus heterozygosity
Q1003:Albinism is a congential deficiency one of which two
items?
1. Tyrosine (auto. Recess.) or 2. defective tyrosine
transporters
Q1004:All forms of homocystinuria have wha inheritance
pattern?
autosomal recessive
Q1005:What are the sx of homocystinuria?
mental retardation; osteoporosis; tall stature; kyphosis; lens
subluxation; atherosclerosis (stroke and MI)
Q1006:The conversion of methionine to cysteine is dependent
on which two vitamins?
B6 (homocysteine to cystothionine) and B12 (homocysteine
back to Met)
Q1007:Maple syrup urine disease is due to what defect?
Blocked degradation of branced amino acid (Ile; Val; Leu) due
to decreased a-ketoacid dehydrogenase
Q1008:What are the sx of maple syrup urine disease?
CNS defecs; mental retardation and death
Q1009:Adenosine deanimase deficiency can cause what other
major immunologic complication?
SCID
Q1010:For the following lysosomal storage dz state (1) main
findings; (2) deficient enzyme; (3)accumulated substrate and
(4) inheritance pattern: Fabry's Dz
1. peripheral neuropathy of hands/feet; CV/renal dz;2. a-
galactosidase A;3. ceramide trihexoside;4. XR
Q1011:For the following lysosomal storage dz state (1) main
findings; (2) deficient enzyme; (3)accumulated substrate and
(4) inheritance pattern: Gaucher's Dz
1. hepatosplenomegally; aseptic necrosis of femur; bone
crises; Gaucher cells (macrophages);2. B-glucocerebrosidase;3.
glucocerebroside;4. AR;Most common lysosomal storage dz
Q1012:For the following lysosomal storage dz state (1) main
findings; (2) deficient enzyme; (3)accumulated substrate and
(4) inheritance pattern: Neimann-Pick
1. progressive neurodegeneration; hepatosplenomegally;
cherry red spot (on macula);2. Sphingomyelinase;3.
Shingomyelin;4. AR
Q1013:For the following lysosomal storage dz state (1) main
findings; (2) deficient enzyme; (3)accumulated substrate and
(4) inheritance pattern: Tay-Sachs
1. progressive neurodegeneration; developmental delay;
lysozymes with onion skin;2. Hexosaminidase A;3. GM2
ganglioside;4. AR
Q1014:For the following lysosomal storage dz state (1) main
findings; (2) deficient enzyme; (3)accumulated substrate and
(4) inheritance pattern: Krabbe's dz
1. peripheral neuropathy; developmental delay; optic
atrophy;2. B-galactosidase;3. Galactocerebroside;4. AR
Q1015:For the following lysosomal storage dz state (1) main
findings; (2) deficient enzyme; (3)accumulated substrate and
(4) inheritance pattern: Metachromic leukodystrophy
1. central and peripheral demyelination with ataxia and
dementia;2. Arylsulfatase A;3. Cerebroside sulfate;4. AR
Q1016:In fatty acid synthesis; the conversion of Acetyl-CoA
to Malonyl CoA requires what cofactor?
Biotin
Q1017:In fatty acid synthesis; what mechanism carries acetyl
CoA across the inner mitochondril membrane into the
cytoplasm?
Citrate shuttle
Q1018:In fatty acid degradation; what mechanism carries acyl
CoA across the inner mitochondrial membrane into the
mitochondria?
carnitine shuttle
Q1019:What is the rate limiting step in fatty acid degradation?
Carnitine shuttle
Q1020:Under what physiological conditions will one see
ketone bodies?
DKA or prolonged starvation
Q1021:The rate limiting step in cholesterol synthesis is
catalyzed by which enzyme?
HMG-CoA reductase
Q1022:What is the classification of the drug Lovastatin?
HMG-CoA reductase inhibitor
Q1023:What are the two essential fatty acids?
Linoeic acid and linolenic acid
Q1024:What is the function of pancreatic lipase?
degradation of TG in sm intestine
Q1025:What is the function of lipoprotein lipase?
degradation of TG circulating in chylomicrons and VLDLs
Q1026:What is the function of Hepatic TG lipase?
degradation of TG remaining in IDL
Q1027:What is the function of hormone-sensitive lipase?
Degradation of TG stored in adipocytes
Q1028:What is the funtion of lecithin-cholesterol
acyltransferase (LCAT)?
catalyzes the esterification of cholesterol
Q1029:what is the function of cholesterol ester transfer
protein (CETP)?
mediated the transfer of cholesterol esters to other lipoprotein
particles
Q1030:What is the role of apolipoprotein A-I?
activates LCAT
Q1031:What is the role of apolipoprotein B-100?
binds to LDL receptor; mediates VLDL secretion
Q1032:What is the role of apolipoprotein C-II?
cofactor for lipoprotein lipase
Q1033:What is the role of apolipoprotein B-48?
mediates chylomicrom secretion
Q1034:What is the role of apolipoprotein E?
Mediates remnant uptake (Extra uptake)
Q1035:What is the function of the following lipoprotein:
Chylomicron
delivers dietary TGs to peripheral tissues and cholesterol to
the liver
Q1036:What apo's are associated with chylomicrons?
B-48; A;C; and E
Q1037:What is the function of the following lipoprotein:
VLDL
delivers hepatic TGs to peripheral tissues
Q1038:what apos are associated with VLDL?
B-100; C-II and E
Q1039:What is the function of the following lipoprotein: IDL
Delivers TGs and cholesterol to the liver to be degraded to
LDL
Q1040:What apos are associated with IDL?
B-100 and E
Q1041:What is the function of the following lipoprotein: LDL
delivers hepatic cholesterol to peripheral tissues
Q1042:What apos are associated with LDL?
B-100
Q1043:What is the function of the following lipoprotein:
HDL
mediates centripital transport of cholesterol (periphery to
liver); repository for apoC and apoE
Q1044:Hypercholesterolemia has which Family type? what is
increased?
Type IIa;Increased LDL
Q1045:Combined hypercholesterolemia has which familial
type? what is increased?
Type IIb;LDL; VLDL both increased
Q1046:Hypertriglyceridemia has which familial type? what is
increased?
Type IV;VLDL increased
Q1047:Type IIa hypercholesterolemia has what
pathophysiology?
decreased number of LDL receptors
Q1048:What metabolic processes occur solely in the
mitochondria?
FA oxidation (b-oxidation); acetyl CoA production; Krebs
Q1049:What metabolic processes occur solely in the
cytoplasm?
glycolysis; FA synthesis; HMP shunt; protein synthesis
(RER); steroid synthesis (SER)
Q1050:What metabolic processes occur in both the
mitochondria and the cytoplasm?
Gluconeogenesis; urea cycle; heme synthesis
Q1051:Regeneration of methionine (and thus S-adenosyl-
methionine/SAM) is dependent on what factor?
B12
Q1052:What three enzymes are associated with respiratory
burst in the phagolysosome?
NADPH oxidase;Superoxide dismutase;Myeloperoxidase
Q1053:What three enzymes are associated with oxidative
burst in the neutrophil?
Catalase/glutathione peroxidase;Glutathione reductase;G6PD
Q1054:Fructose intolerance is an inherent deficiency of what
enzyme?
aldolase B
Q1055:What metabolic processes are inhibited from a fructose
deficiency?
glyogenolysis and gluconeogenesis
Q1056:What are the symptoms of fructose intolerance?
hypoglycemia; jaundice; cirrhosis; vomiting
Q1057:What is the treatment for fructose intolerance?
decrease intake of fructose and sucrose (glucose and fructose)
Q1058:the appearance of fructose in the blood or urine is due
to a defect in what enzyme? What is the px?
fructokinase; the condition in benign and asymptomatic
Q1059:Galactosemia is caused by the absence of what
enzyme?
galactose-1-phosphate uridyltransferase
Q1060:What is the inheritance pattern and sx of galactosemia?
AR; cataracts; hepatosplenomegally; mental retardation
Q1061:Lactase deficiency is due to a loss of the enzyme from
what area of the body?
Brush border
Q1062:What are the symptoms of lactase deficiency?
bloating; cramps; osmotic diarrhea
Q1063:Which are the essential amino acids?
Leu; Lys; Ile; Phe; Trp; Met; Thr; Val; Arg; His
Q1064:Which two essential amino acids are increased in
histones which bind negatively charged DNA?
Arg; Lys
Q1065:What is the direct effect of excess NH4
(hyperammonemia) on metabolism?
depletes a-ketoglutarate -> inhibition of the TCA cycle
Q1066:What is the treatment for hyperammonenmia?
Arginine
Q1067:What are the signs of ammonia intoxication?
tremor; slurring of speech; somnolence; vomiting; cerebral
edema; blurring of vision
Q1068:What is the fnxn of the urea cycle?
to degrade amino acids into amino groups
Q1069:What are the 8 main players in the urea cycle?
Ornithine; Carbamoyl; Citruline; Aspartate;
Argininosuccinate; Fumarate; Arginine; Urea;Ordinarily
Careless; Crappers Are Also Frivolous About Urination
Q1070:What are the main derivatives of Phenylalanine?
Tyrosine;Dopamine;NE ;EPI
Q1071:What are the main derivatives of Tryptophan?
Niacin (NAD/NADP);Serotonin;Melatonin
Q1072:What are the main derivatives of Histidine?
Histamine
Q1073:What are the main derivatives of Glycine?
Heme
Q1074:What are the main derivatives of Arginine?
Creatine;Urea;NO
Q1075:What are the main derivatives of glutamate?
GABA
Q1076:What enzyme is decreased in PKU?
phenylalanine hydroxylase (or tetrahydrobiopterin cofactor)
Q1077:What is the treatment for PKU?
Decrease phenylalanine and increase tyrosine
Q1078:Alkaptonuria is caused by a deficiency of what
enzyme?
homogentisic acid oxidase (alkapton bodies cause urine to turn
black when standing)
Q1079:Abinism is a congenital deficiency in what two
factors?
Either tyrosinase or defective tyrosine transporters
Q1080:Albinism exhibits variable inheritance due to what
mechanism?
locus heterozygosity
Q1081:What is the inheritance pattern for all three forms of
homocystinuria?
AR
Q1082:What are the three forms of homocystinuria?
1. cystathionine synthase deficiency;2. decreased affinity of
cystathionine synthase for pyridoxal phosphate;3. methionine
synthase deficiency
Q1083:Cystinuria is a common inherited defect of;
the renal tubular amino acid transporter for cystine; ornithine;
lysine; and arginine
Q1084:What can be a sequellae of excess cystine in the urine?
cystine kidney stones
Q1085:What is the treatment for cystinuria?
acetazolamide (alkalinize the urine)
Q1086:Maple syrup urine disease is caused by a decrease in
which enzyme?
a-ketoacid dehydrogenase
Q1087:Which amino acids canno be degraded in maple syrup
urine disease?
Ile; Leu; Val;I Love Vermont maple syrup
Q1088:What is codominance?
Neither of two alleles is dominant (e.g. blood groups)
Q1089:What is variable expression?
nature and severity of the phenotype varies from 1 individual
to another
Q1090:What is incomplete penetrance?
Not all individuals with a mutant genotype show the mutant
phenotype
Q1091:What is pleiotropy?
1 gene has > 1 effect on an individual's phenotype
Q1092:What is imprinting?
At a single locus; only one allele is active; the other is inactive
(methylation).Differneces in phenotype depend on whether
the mutation is of maternal or paternal origin (e.g. Prader-Willi
= paternal)
Q1093:What is anticipation?
Severity of disease worsens or age of onset of disease is earlier
in succeeding generations (e.g. Huntington's)
Q1094:What is loss of heterozygosity?
If a patient inherits or develops a mutation in a tumor
suppressor gene; the complementary allele must be
deleted/mutated before the cancer develops (NOT true of
oncogenes)
Q1095:What is a dominant negative mutation?
exerts a dominant effect. A heterozygote produces a
nonfunctional altered protein that also prevents the normal
gene product from functioning
Q1096:What is linkage disequilibrium?
tendency for certain alleles at 2 linked loci to occur together
more often than expected by chance. Measured in a
population; not in a family; and often varies in different
populations
Q1097:What is mosacism?
when cells in the body have different genetic makeup
(e.g.lyonization--random X inactivation in females)
Q1098:What is locus heterogeneity?
Mutations at different loci can produce the same phenotype
Q1099:If a population is in Hardy-weinberg equilibrium; how
do you measure allele prevalence?
p^2 + 2pq + q^2 = 1; p and q are separate alleles and 2pq is
the prevalence of heterozygotes
Q1100:If a population is in Hardy-weinberg equilibrium; how
do you measure allele prevalence?
p+q=1
Q1101:What are the 4 assumptions of Hardy-Weinberg law?
1. no mutation; 2. no selection for any of the genotypes at a
locus; 3. Random mating; 4. no migration into or out of a
population
Q1102:In Prader-Willi; whose normally active allele is deleted
(maternal or paternal)?
Paternal (opposite for angelman's)
Q1103:What are some of the features of autosomal dominant
inheritance?
many generations affected; both male and female; often
pleiotropic; present after puberty
Q1104:What are some of the features of autosomal recessive
inheritance?
25% of offspring from 2 carrier parents affected; due to
enzyme deficiencies; usually only seen in 1 generation; more
severe than AD; presents in childhood
Q1105:What are some of the features of X-linked recessive
inheritance?
Sons of heterozygous mothers have a 50% chance; commonly
more severe in males but heterozygous females may be
affected
Q1106:What are some of the features of X-linked dominant
inheritance?
transmitted through both parents; ALL female offspring of an
affected father will be diseased (either male of female from
mother); hypophosphatemic rickets
Q1107:What are some of the features of mitochondrial
inheritance?
transmitted only through mother; all offspring of affected
mother show disease; Leber's hereditary optic neuropathy
Q1108:Mode of inheritance and major features: APKD
AD; always bilateral; mutation in APKD1 (Chr 16); berry
aneurysms; mitral valve prolapse (juvie form is recessive)
Q1109:Mode of inheritance and major features: familial
hypercholesterolemia
AD; elevated LDL from a defective or absent LDL receptor;
severe atherosclerosis disease at young age; tendon xanthomas
(achilles); may have MI before age 20
Q1110:Mode of inheritance and major features: Marfan's
Syndrome
AD; fibrilin gene mutation = connective tissue disorders; tall;
aortic incomptenece and dissecting aortic aneurysms; floppy
mitral valve
Q1111:Mode of inheritance and major features:
Neurofibromatosis; type 1
AD long arm of chr. 17; café-au-lait spots; neural tumors;
Lisch nodules (pigmented iris hamartomas); pheo's
Q1112:Mode of inheritance and major features:
Neurofibromatosis; type 2
AD; bilateral acoustic neuroma; juvenile cataracts; NF2 gene
on chr 22 (type 2 = 22)
Q1113:Mode of inheritance and major features: tuberous
sclerosis
AD; facial lesions (adenoma sebaceum); hypopigmented "ash
leaf spots" on skin; cardiac rhabdomyomas; incomplete
penetrance/variable presentation
Q1114:Mode of inheritance and major features: von Hippel-
Lindau dz (VHL)
AD; hemangioblastomas of the retina/cerebellum/medulla;
50% develop multiple bilateral renal cell carcinomas; deletion
of VHL gene on Chr 3 (three words for Chr 3)
Q1115:Mode of inheritance and major features: Huntington's
AD; triplet repeat disorder on chr 4; depression; progressive
dementia; choreiform movements; caudate atrophy; decreased
levels of GABA and ACh in brain
Q1116:Mode of inheritance and major features: Familial
adenomatus polyposis
AD; progresses to colon cancer unless resected; Deletion on
chr. 5 (5 letters in polyp)
Q1117:Mode of inheritance and major features: Hereditary
spherocytosis
AD; spheroid erythrocytes; hemolytic anemia; increases
MCHC; splenectomy is curative
Q1118:Mode of inheritance and major features:
Achondroplasia
AD; cell-signalling defect of fibroblast growth factor (FGF)
receptor 3; dwarfism (short limbs but head and truck are
normal size); associated with advanced PATERNAL age
Q1119:What gene is defective in cystic fibrosis?
CFTR on chromosome 7
Q1120:What are the typical manifestations of a pt with CF?
recurrent pulmonary infections; infertility in males (absent
vas deferens); fat soluble vitamin deficiencies
Q1121:What is the treatment for CF?
n-acetylcysteine to loosen mucous plugs
Q1122:What are the typical X-linked recessive disorders?
Bruton's agammaglobulinemia; Fragile X; G6PD deficiency;
ocular albinism; Lesch-Nyhan; Duchenne's; Hemophilia A and
B; Fabry's dz; Hunter's syndrome
Q1123:Duchenne's MD is caused by what type of genetic
mutation?
Frame shift leading to deletion of the dystrophin gene
Q1124:What movement is a diagnostic sign of Duchenne's
MD?
Gower's maneuver
Q1125:What is the less severe form of duchenne's MD?
Becker's
Q1126:What is the 2nd most common cause of genetic mental
retardation after Down's?
Fragile X syndrome
Q1127:What are the clinical features of Fragile X syndrome?
macro-orchidism; long face with a large jaw; large everted ears;
autism
Q1128:What type of disorder is Fragile X?
Triple repeat (CGG) - may show anticipation
Q1129:What are the trinucleotide repeat diseass?
Huntington's; myotonic dystrophy; Friedreich's ataxia; Fragile
X
Q1130:What are the three autosomal trisomies and which
chr's are affected?
Down's (21); Edward's (18); Patau (13)
Q1131:What are the prenatal signs of Down's?
decreased a-fetoprotein; increased b-hCG; increased nuchal
translucency
Q1132:Pts with Down's are at risk of developing what
neurological disorder?
early-onset Alzheimer's
Q1133:95% of Down's cases are due to what problem in
meiosis? What is the associated parental "problem"?
meiotic nondisjunction of homologous chromosomes;
associated with advanced maternal age
Q1134:Cri-du-chat is associated with what chromosomal
abnormality?
congenital deletion of short arm of chr 5
Q1135:22q11 syndromes are associted with what
signs/symptoms?
cleft papate; abnormal facies; thymic aplasia (t-cell
deficiency); cardiac defects; hypocalcemia 2' to parathyroid
aplasia; variable presentation as DiGeorge
Q1136:When are the risks for fetal alcohol syndrome the
greatest?
3-8 weeks
Q1137:What is the #1 cause of congenital malformations in
the US?
Fetal Alcohol Syndrome
Q1138:DNA polymerase III vs I
III--5'->3' synthesis; 3'->5' exonuclease (proofread); I--
degrades RNA primer (5'->3' exonuclease); fills in gap
w/DNA
Q1139:nucleotide excision repair vs base excision repair
nuc--releases damaged OLIGOnucleotides (ex; in XP); base--
specific glycosylases recognize and remove damaged BASES
Q1140:alpha-amanitin
poison from a mushroom; inhibits euk RNA pol II (mRNA);
initially GI problems; rapidly results in death
Q1141:different RNAs of the euk RNA polymerases
RNA pol I--rRNA; RNA pol ii--mRNA; RNA pol III--tRNA
Q1142:mRNA stop codons
UGA; UAG; UAA
Q1143:site where negative gene expression regulators bind
operator
Q1144:amino acid binds to which end of tRNA?
3' end (CCA end)
Q1145:I cell dz
failure of addition of mannose-6-phosphate to lysosome
enzymes-->enzymes secreted outside cell; get coarse facial
features; clouded corneas; restricted jnt movement; high
plasma lysosomal enzymes; often fatal in childhood
Q1146:vesicular trafficking protein--COPI-->
retrograde; Golgi->ER
Q1147:vesicular trafficking protein COPII--> ?
anterograde; RER->cis-Golgi
Q1148:vesicular trafficking protein clathrin--> ?
trans-Golig-> lysosome; plasma membrane-> endosomes
Q1149:type III collagen
reticulin; in skin; blood vessels; uterus; fetal tissue;
granulation tissue. Most common type of Ehrlos-Danlers
involves this
Q1150:type IV collagen found where?
basement membrane; basal lamina
Q1151:steps of collagen synthesis
1. synthesis of preprocollagen (rER); 2. hydroxylation (ER;
requires vit C); 3. glycosylation (Golgi); 4. exocytosis (still as
procollagen); 5. proteolytic processing (tropocollagen); 6.
crosslinking (lysyl oxidase; collagen fibrils)
Q1152:which type of osteogenesis imperfecta is fatal in utero
or in neonate?
type II
Q1153:vimentin stains?
connective tissue
Q1154:desmin stains?
muscle
Q1155:cytokeratin stains?
epithelial cells
Q1156:all enzymes except 1 of TCA are WHERE and
where/what is the exception?
all but succinate DH are in mitochondrial MATRIX; succinate
DH is in inner mitochondrial membrane (along with ETC)
Q1157:irreversible enzymes in gluconeogenesis (4)
pyruvate carboxylase (mit; pyruvate->oxaloacetate); PEP
carboxykinase (cyt; oxalo->PEP); F1;6BPase (cyt; F1;6BP-
>F6P); G6Pase (er; G6P->glucose)
Q1158:most potent activator of PFK? (rate limiter of
glycolysis)
F2;6BP (overrides inhibition by ATP; citrate)
Q1159:what are the only purely ketogenic amino acids?
lysine; leucine
Q1160:how much ATP produced per acetyl CoA in TCA
cycle?
12 (3/NADH; 2/FADH2; 1/GTP)
Q1161:what is rotenone
fish poison that complexes with NADH DH (complex I of
ETC); NADH accumulates; but can still get electrons into
ETC from FADH2
Q1162:what is antimycin A?
abx that blocks passage of electrons through cytochrome b-c1
complex (ETC)
Q1163:why is cyanide poisonous?
combines with cytochrome oxidase and blocks electrons->O2
in ETC; (CO does this also)
Q1164:what is dinitrophenol (2;4DNP)?
ionophore that uncouples ETC
Q1165:what is the enzyme of the irrerversible/oxidative
reaction of PPP (HMP shunt)?
glucose phosphate DH
Q1166:what is the enzyme of the reversible/nonoxidative
reaction in PPP (HMP shunt)?
transketolase (requires thiamine)
Q1167:essential fructosuria
defect in fructokinase (F->F1P); however; this is benign and
asymptomatic; only get fructose in blood and urine
Q1168:aldolase B deficiency--> ?
(AR) fructose intolerance; F1P accumulates and decreases
available phosphate. This inhibits glycogenolysis;
gluconeogenesis; get hypoglycemia; jaundice; cirrhosis;
vomiting
Q1169:which 5 essential amino acids are glucogenic?
met; thr; val; arg; his;(Argh; This Here Mnemonic is Vile)
Q1170:which essential amino acids are glucogenic and
ketogenic?
Ile; Phe; Trp
Q1171:Phe is precursor for?
tyrosine; thyroxine; DOPA; melanin; DA; NE; Epi
Q1172:Tryptophan is precursor for?
niacin; serotonin; melatonin
Q1173:glycine is precursos for?
porphyrin; (succinyl CoA+ glycine-> ALA; via ALA
synthetase)
Q1174:arginine is precursor for?
creatine; NO; urea
Q1175:glutamate is precursor for?
GABA; your mom
Q1176:cystinuria is inherited defect for renal transport of
which amino acids?
cystine; ornithine; lysine; arginine
Q1177:which GLUT is insulin responsive?
GLUT 4 (adipose; sk mm)
Q1178:which GLUT is in RBCs; brain?
GLUT 1
Q1179:GLUT 2 is where?
beta islet cells; liver; kidney
Q1180:which enzyme affected in a)Pb poisoning b) AIP c)
PCT
a)ferrochelatase; ALA dehydrase; b) porphobilinogen
deaminase (formerly known as: uroporphyrinogen I
synthase); c) uroporphyrinogen decarboxylase
Q1181:Full term neonate of uneventful delivery becomes
mentally retarded and hyperactive and has musty odor;WHat
is the diagnosis
PKU
Q1182:Stressed executive comes home from work consumes 7
or 8 martinis in rapid succession before dinner and becomes
hypoglycemic;WHat is the mechanism
NADH increase prevents gluconeogenesis by shunting
pyruvate and oxaloacetate to lactate and malate
Q1183:2 year old girl has increase in abdominal girth; failure
to thrive and skin and hair depigmentation - what is the
diagnosis
Kwashiorkor
Q1184:Alcoholic develops rash; diarrhea and altered mental
status;What is the vitamin defficiency
B3 (pellagra)
Q1185:20 year old male presents with idiopathic
hyperbillirubinemia;WHat is the most common cause
Gilberts syndrome
Q1186:51 year old man has black spots on his sclera and has
noted that urine turns black when he is standing
Alkaptonuria
Q1187:25 year old complains of chest pains and has
xanthoma of Achilles tendon;What is his disease and where is
the defect
Familial hypercholesterolemia; lDL receptor
Q1188:Condensed by negatively charged DNA looped twice
around positively charged H2a; H2b; H3 and H4 histones
(nucleosome bead)
Chromatin
Q1189:_ ties nucleosomes together in a string (30 nm fiber)
H1 histone
Q1190:Condensed; transcriptionally inactive chromatin
Heterochromatin
Q1191:Less condensed; transcriptionally active chromatin
Euchromatin
Q1192:Name purines
A; G;PURe As Gold
Q1193:Name pyrimidines
C; T; U;CUT Pye
Q1194:Which nucleotides have two rings?
Purines
Q1195:WHich nucleotides have one ring?
Pyrimidines
Q1196:Which nucleotide has a ketone
guanine
Q1197:Which nucleotide has methyl
thymine
Q1198:Deamination of cytosine makes _
Uracyl
Q1199:Uracil is found in _ ;Thymine is found in _
Uracil - RNA;Thymine - DNA
Q1200:Which bond is stronger GC or AT
GC (3 H bonds); AT is weaker (2 H bonds)
Q1201:If GC content is increased what happens to melting
temperature
Increases
Q1202:Nucleotides are linked by _
3-5 phosphodiesterase bond
Q1203:Substituting purine for purine or pyrimidine for
pyrimidine is called ?
TransItion (identical)
Q1204:Substituting purine for pyrimidine or vice versa
TransVersion (conVersion between types)
Q1205:4 features of genetic code
Unambiguous (each codon for only one amino
acid);Degenerate (more then one codon can code for same
amino acid);COmmaless; nonoverlapping;Universal
Q1206:Name type of mutation - same amino acid; often base
change in 3d position of codon tRNA wobble)
Silent mutation
Q1207:Name type of mutation - changed amino acid
(conservative - new amino acid is similar in chemical
structure)
Missence mutation
Q1208:Name type of mutation - change resulting in early stop
codon
Nonsense mutation
Q1209:Name type of mutation - change resulting in
misreading of all nucleotides downstream; usually resulting in
a truncated protein
Frameshift mutation
Q1210:In prokaryotic replication; is there single or multiple
origins of replication
Single origin of replication - continuous DNA synthesis on
leading strand and discontinuous (okazaki fragments) on
lagging strand
Q1211:What is the role of primase in prokaryotic replication
Primase makes RNA PRIMER on which DNA polymerase
III can initiate replication
Q1212:Elongates the chain by adding deoxynucleotides to the
3 end until it reaches primer of preceding fragment
DNA polymerase III
Q1213:Name enzyme that degrades RNA primer
5'-3' exonuclease activity of DNA polymerase I
Q1214:Has 5'-3' synthesis and proofreads with 3'-5'
exonuclease
DNA polymerase III
Q1215:Create a nick in the helix to relieve supercoils
DNA topoisomerases
Q1216:Name DNA repair defects
Xeroderma pigmentosum (skin sensitivity to UV
light);Ataxia-telangiectasia (x rays);Blooms syndrome
(radiation);Fanconis anemia (cross linking agents)
Q1217:Defective excision repair such as uvr ABC
endonuclease. Results in inability to repair thymidine dimers;
which form in DNA when exposed to UV light;Associated
with dry skin and with melanoma and other
cancers;Inheritance pattern
Xeroderma pigmentosum;Autosomal recessive
Q1218:In which direction is Dna and RNA synthesized
5'-->3'
Q1219:Chromatin Structure
negatively charged DNA wrapped around a histone (H2A;
H2B; H3; H4); connected by H1;Condensed = hetero-;
inactive;Less condensed = eu-; active
Q1220:Nucleotides
Purines (A; G) large rings;Pyrimidines (C; U; T) small
rings;PURe As Gold;CUT the PY (pie)
Q1221:Eukaryotic DNA polymerases
alpha - replicates lagging strand; synthesizes RNA
primer;beta - repairs DNA;gamma - replicates mitochondrial
DNA;delta - replicates leading strand;epsilon - repairs DNA
Q1222:DNA repair process
endonuclease cleaves strand upstream;exonuclease clease
strand downstream;DNA polymerase Beta fills gap
Q1223:DNA repair defects lead to disease
Xeroderma Pigmentosum (UV light);Ataxia-Telangiectasia (X-
rays) ;Bloom's syndrome (radiation);Fanconi's anemia
(crosslinking agents)
Q1224:Eukaryotic RNA polymerases
polymerase I - rRNA;polymerase II - mRNA;
snRNPs;polymerase III - tRNA;no proofreading;alpha-
amanitin inhibits poly II
Q1225:Start and Stop Codons
Start;AUG (Are U Going?);Stop;UGA (U Go Away);UAA
(U Are Away);UAG (U Are Gone)
Q1226:Regulation of Gene Expression
Promoter - where RNA polymerase/transcription factors bind
upstream;TATA (25 bp upstream);CAAT (70 bp
upstream);Enchancer - where transcription factors bind to
increase expression
Q1227:RNA processing
1. 5' 7-methyl-guanine cap;stability; mediates translation;2. 3'
Polyadenylation;stability; mediates nuclear export;3. Splicing
out introns;for fun?
Q1228:tRNA Structure
75-90 nucleotides;anticodon end is opposite 3' aminoacyl ;3'
CCA sequence;chemically modified bases
Q1229:tRNA charging
aminoacyl-tRNA synthetase ;adds 1 aa to 3' end; using 1
ATP;proofreading capability
Q1230:tRNA wobble
allows many codons to match one tRNA with only the first
two bases of it's anticodon
Q1231:PCR
ligate/denature DNA;add premade specific probes;add heat-
stable DNA polymerase;repeat until DNA sequence is
amplified
Q1232:Molecular Biology Techniques
Southern - DNA probe to find DNA;Northern - DNA probe
to find RNA;Western - Ab probe to find
protein;Southwestern - DNA probe for TFs
Q1233:ELISA
labeled Ab/Ag to Ag/Ab in pt sample;used in HIV;Sn = Sp =
~100%
Q1234:Inheritance Modes
Auto Dom - structural genes;M/F affected equally;presents
after puberty;Auto Rec - 25% offspring of carriers;enzyme
deficiencies;present in childhood;X-link Rec - 50% sons of
hetero mom;X-link Dom - all F kids of sick dad ;M/F kids of
sick mom;hypophosphatemic rickets;Mitochondrial -
transmitted by mom;all kids may show dz;leber's hereditary
optic neuropathy;mitchondrial myopathies
Q1235:Variable expression
nature and severity of phenotype varies from one pt to
another
Q1236:Incomplete penetrance
Not all individuals with mutant genotype show dz phenotype
Q1237:Pleiotropy
one gene has greater than one effect on phenotype
Q1238:Imprinting
Differences in phenotype depend on whether the mutation is
of maternal or paternal origin;Angelmans - maternal
transmission;Prader-Willi - paternal transmission
Q1239:Anticipation
Severity of disease worsens or age of onset decreases in
successive generations
Q1240:Loss of heterozygosity
if a pt inherits or develops a mutation in a tumor suppressor
gene; the complementary allele must be deleted/mutated
before cancer develops (not true of oncogenes)
Q1241:Dominant negative mutation
exerts a dominant effect because the body cannot produce
enough of the normal gene product with only one functioning
allele or presence of the altered gene product inhibits the
normal product
Q1242:Linkage disequilibrium
two alleles at linked loci occur together more often than
probability would suggest.
Q1243:Hardy Weinberg population genetics
assumes no migration; no mutation; no natural selection; no
mating preferences
Q1244:Down Syndrome
mental and growth retardation;trisomy 21;tested with
karyotyping;1:800;increased risk with maternal age;decreased
AFP in amniotic fluid;polyhydramnios
Q1245:Fragile X
mental retardation;characteristic facial features;large testes;X-
linked;failure to express RNA binding protein
Q1246:Sickle cell anemia
recurrent painful crises;autosplenectomy -> imm
def;autosomal recessive;1 missense mutation in beta
globin;1:400 Af-Am
Q1247:Cystic fibrosis
recurrent pulmonary infections;exocrine pancreas
insufficiency;infertility;autosomal recessive;mutated CFTR
(Cl- channel);1/2000 whites
Q1248:Neurofibromatosis
cafe-au-lait
spots;neurofibromas;pheochromocytomas;autosomal
dominant;signaling molec loss-of-fxn mutations
Q1249:Duchenne's Muscular Dystrophy
muscular weakness and degeneration;X-linked
recessive;dystrophin gene deletion;Dx DNA
test;pseudohypertrophy of calf
Q1250:Osteogenesis Imperfecta
increased bone fx;blue sclera - translucent CT over
choroid;many mutations - abn collagen synth;1:10000
Q1251:Phenylketonuria
autosomal recessive;phenylalanine hydroxylase
deficiency;tetrahydrobiopterin cofactor deficiency;tyrosine
becomes essential;mental retardation;fair skin (decreased
melanin);eczema;musty body odor;decrease phenylalanine in
diet;increase tyrosine in diet
Q1252:Fabry's disease
X-linked recessive;alpha galactosidase deficiency;ceramide
trihexoside accumulation;renal failure;peripheral neuropathy
hands/feet;CV disease
Q1253:Krabbe's disease
Autosomal recessive;Bgalactosidase def;galactocerebroside
accumulation;optic atrophy;peripheral
neuropathy/spasticity;developmental delay
Q1254:Gaucher's disease
glucocerebrosidase def;glucocerebroside
accumulation;hepatosplenomegaly;aseptic necrosis of femoral
head;bone crises;Gaucher's cells (macrophages)
Q1255:Niemann-Pick disease
Autosomal recessive;sphingomyelinase def;sphingomyelin
accum in reticuloendothelial cells/parenchyma;leading to
organomegaly and progressive neurodegeneration;cherry red
spot on macula;No Man PICKs his nose w/ a SPHINGer
Q1256:Tay-Sachs disease
Autosomal recessive;Absence of hexosaminidase A;GM2
ganglioside accumulation;Askenazi Jews (carriers =
1/30);death by age 3;cherry red spot on macula
Q1257:Metachromatic Leukodystrophy
Autosomal recessive;arylsulfatase A deficiency;demyelination
with w/ ataxia; dementia;Cerebroside sulfate in brain; liver;
kidney; PNS
Q1258:Hurler's syndrome
Autosomal recessive;alpha-L-iduronidase deficiency;corneal
clouding;gargoylism;developmental delay;Hurlers (shot put)
do more damage than hunters (arrows)
Q1259:Hunter's syndrome
X-linked recessive;iduronate sulfatase deficiency;mild form of
Hurler's (mild retardation);with aggressive behavior;no corneal
clouding
Q1260:Lineweaver Burke plots
noncompetitive inhibitors change the Vmax (the y-intercept
will increase);competitive inhibitors change the Km (the x-
intercept will increase)
Q1261:Sodium Potassium Pump
BL membrane;moves 3 Na out; 2 K in; uses 1 ATP;inhibitied
by oubain; digoxin
Q1262:Collagen synthesis and structure
hydoxylation of proline/lysine residues in RER requires Vit
C;procollagen exocytosed into ECM;peptidases cleave
terminal portion;self-assembly into collagen fibrils;crosslinked
by lysyl oxydase
Q1263:Ehlers-Danlos Syndrome
faulty collagen synthesis causes;hyperextensible
skin;tendency to bleed;hypermobile joints
Q1264:S-adenosyl methionine
ATP + methionine = SAM;methyl group donor;makes
phosphocreatine;regeneration with B12
Q1265:NAD+/NADPH
NAD+ catabolic electron acceptor;NADPH anabolic electron
donor;product of HMP shunt;makes superoxide;regenerates
GSH;p450
Q1266:Glycolysis: irreversible reactions
glu to G6P;G6P inhibits hexokinase;F6P to
F1;6BP;ATP/citrate inhib PFK ;F2;6BP/AMP upreg
PFK;PEP to pyruvate;ATP/Ala inhib Pyr Kinase;F1;6BP
upreg Pyr Kinase;Pyr to AcetylCoA;ATP/NADH/AcylCoA
inhib Pyr DeH
Q1267:Pyruvate Dehydrogenase Complex
pyr + NAD+ + CoA -> ;AcylCoA + CO2 +NADH;3
enzymes;5 cofactors;B1 thiamine;B2 FAD;B3 NAD;B5
CoA;lipoic acid
Q1268:Pyruvate Dehydrogenase Deficiency
buildup of pyruvate and alanine;reduced to lactate ->
acidosis;seen in alcoholics in B1 deficiency;Rx: high
fat/ketogenic nutrients
Q1269:Cori cycle
shuttles lactate from muscle to liver for regeneration to
pyruvate;allows muscles to fxn anaerobically
Q1270:TCA cycle
12 ATP/Acyl CoA;24 ATP/glu molec;1st four enzymes are
inhib by ATP/NADH;Cindy Is Kind So She's Friendly More
Often
Q1271:Gluconeogenesis: irreversible reactions
Pyr to oxaloacetate;Pyr carb req
ATP/AcylCoA/biotin;Oxaloacetate to PEP;PEP carbK req
GTP;F1;6BP to F6P;F1;6BPase;G6P to glu;G6Pase;enzymes
in liver; kidney; intestine;hypogly with G6Pase def (von
Gierke's)
Q1272:Glucose 6 Phosphate Dehydrogenase Deficiency
rate limiting enzyme of HMP shunt;necessary for RBCs to
produce NADPH for GSH regeneration;loss leads to
hemolytic anemia;triggered w/ oxidizing agents: sulfas;
primaquine; fava beans;Heinz bodies - Hb
precipitates;prevalent in Af-Am;X-linked recessive
Q1273:Fructose intolerance
aldolase B deficiency;all phosphate accum in F1P;inhib
glycogenolysis/gluconeogenesis;hypoglycemia; jaundice;
cirrhosis;Rx: decrease fructose/sucrose
Q1274:Galactose intolerance
galactose-1P uridyltransferase def;accum of toxic
metabolites;cataracts;hepatosplenomegaly;mental
retardation;Rx: decrease galactose/lactose
Q1275:Essential Amino Acids
PVT TIM HALL;phe;val;trp;thr;ile;met;his;arg;leu;lys
Q1276:Urea Cycle
C from CO2 (mitochondria);N from NH4 (mitochondria);N
from aspartate (cytosol)
Q1277:Phenylalanine derivatives
Tyrosine; Dopa; DA; NE; Epi; Melanin; Thyroxine
Q1278:Tryptophan derivatives
Niacin (NAD; NADP);Serotonin;Melatonin
Q1279:Histidine derivatives
Histamine
Q1280:Glycine derivatives
Porphyrin/Heme
Q1281:Arginine derivatives
Creatine;Urea;Nitric Oxide
Q1282:Homocytinuria
defective cystathionine synthase or;defective methionine
synthase;cysteine become essential;mental
retardation;osteoporosis;lens subluxation;tall stature;kyphosis
Q1283:Maple Syrup Urine Disease
alpha ketoacid dehydrogenase def blocked degradation of
branched aa (Ile; Leu; Val = I Love Vt maple syrup);CNS
defects; mental retardation; death
Q1284:Adenosine Deaminase Definiciency
SCID;Excess dATP prevents production of other deoxyribose
nucleotides via ribonucleotide reductase;-> lymphopenia
Q1285:Lesch-Nyhan Syndrome
X-linked recessive;LNS (Lacks Nucleotide Salvage);HGPRT
deficiency -> dec IMP/GMP prod -> inc uric acid
excr;retardation;self-mutilation;aggression;hyperuricemia;
gout; choreoathetosis
Q1286:Fatty Acid Metabolism
Synthesis = cytosol;enters via citrate shuttle;degradation =
mitochondria;enters via carnitine shuttle
Q1287:von Gierke's disease
Glycogen storage disease type I;glucose-6Pase
deficiency;(liver becomes like muscle);severe fasting
hypoglycemia;glycogen accum in liver;Very
Q1288:Pompe's disease
Glycogen storage disease type II (trashes the pump);lysomal
alpha-1;4-glucosidase def;cardiomegaly;early death;Poor
Q1289:Cori's Disease
Glycogen storage disease type III;deficiency of debranching
enzyme;alpha-1;6-glucosidase;Carbohydrate
Q1290:Glycogen storage disease type V
McArdle's disease;muscle glycogen phosphorylase
def;glycogen in musc -> painful cramps;myoglobinuria w/
strenuous exercise;Metabolism
Q1291:Ketone bodies
acetoacetate and betahydroxybutyrate;made in liver from
HMG-CoA;excr in urine (test for acetoacetate);elevated in
starvation/DM ketoacidosis;fruity breath;converted to 2
AcetylCoA in brain
Q1292:Insulin
from pancreatic beta cells;inc glu uptake in
musc/liver/fat;GLUT2 R in beta cells;GLUT4 in
periphery;inhib glucagon from alpha cells;C-peptide cleaved -
> activation
Q1293:Glucagon vs. Insulin
Glucagon phosphorylates;turns OFF glycogen synthase;turns
ON glycogen phosphorylase;Insulin dephosphorylates;turns
ON glycagen synthase;turns OFF glycagen phosphorylase
Q1294:Cholesterol synthesis and esterification
HMG-CoA reductase is the rate limiting enzyme in
synthesis;inhib by Lovastatin ;Esterification via LCAT
Q1295:Chylomicrons
dietary TGs to peripheral tissues; dietary chol to liver;travel
in lymphatics to thoracic duct to blood;excess ->
xanthomas;Apo B48 mediates excretion;Apo CII for
lipoprotein lipase;Apo E mediates liver uptake
Q1296:VLDL
hepatic TGs to periphery;excess causes pancreatitis;apo
B100 mediates secr;apo CII for lipoportein lipase;apo E
mediates liver uptake
Q1297:LDL
produced via VLDL modification;hepatic cholesterol to
periphery;uptake via R-med endocytosis (Apo B100);excess
causes ATH; xanthomas;Bad for you
Q1298:HDL
periphery cholesterol to liver;repository for Apo C/E;Apo A-
1 for LCAT & chol-esters;Good for you
Q1299:Familial Hypercholesterolemia
Increased Cholesterol/LDL;Auto Dom defect in LDL
R;xanthomas;MI before 30y in homozygous pt
Q1300:Familial Hypertriglyceridemia
Increased TGs/VLDL;Hepatic overproduction of VLDL
Q1301:Heme Synthesis
Rate limiting step: glycine + succinyl CoA -> ALA via ALA
synthase;occurs w/in mitochondria;inhibition ->
porphyrias;Pb inhib other enzymes ->
microcytic/hypochromic anemia and porphyria
Q1302:Heme catabolism
scavenged from RBCs;heme -> biliverdin -> bilirubin;bilirubin
excr in bile;converted to urobilinogen;excreted as urobilin in
urine
Q1303:Methemoglobinemia
Fe in ferric (oxidized) state;low O2 affinity
Q1304:Hb structure/affinity
Cl; H; CO2; DPG; heat favor the T (taut) form over R
(relaxed);causes decreased O2 affinity
Q1305:Vitamin A
Retinol; in retinal pigment;deficiency: night blindness; dry
skin
Q1306:Vitamin B1
Thiamine; cofactor in pyruvate carboxylase ;deficiency:
Beriberi and Wernicke's encephalopathy
Q1307:Vitamin B2
Riboflavin; FAD/FADH2;deficiency: corneal vascularization;
cheilosis
Q1308:Vitamin B3
Niacin; NAD/NADH (from Trp);deficiency: Pellagra;caused
by carcinoid syndrome; INH; Hartnup Dz;Diarrhea;
Dermatitis; Dememtia
Q1309:Vitamin B5
Pantothenate -> Coenzyme A;FA synth; Kreb's
Cycle;deficiency: Dermatitis; Enteritis; Alopecia; adrenal
insufficiency
Q1310:Vitamin B6
Pyridoxine;Converted to Pyridoxal Phosphate;cofactor in
transamination (ALT/AST)
Q1311:Biotin
cofactor for carboxylation;1. pyruvate -> oxaloacetate;2.
acetyl CoA -> malonyl CoA;3. proprionyl CoA ->
methylmalonyl CoA
Q1312:Folic Acid
Coenzyme for 1-C transfer;methylation rxns for nitrogenous
bases;most common vitamin deficiency in US: macrocytic;
megaloblastic anemia
Q1313:Vitamin B12
Cobalamin;Cofactor for homocyteine methylation &
methylmalyonyl handling;Stored in liver;deficiency caused by:
malabsorption (sprue; enteritis; diphyllobothrium latum);
pernicious anemia; ileectomy;Dx: Schilling test
Q1314:Vitamin C
Ascorbic Acid;cofactor for hydroxylation of proline/lysine in
collagen;facilitates Fe adsorption by keeping it
reduced;Deficiency: scurvy = swollen gums; bruising; poor
healing
Q1315:Vitamin D
D2 absorbed from gut;D3 formed in skin;25OH D3 storage
form;1;25OH D3 active form;increases Ca/PO4
absorption;deficiency: rickets; osteomalacia; hypoCa
tetany;excess: hyperCa; stupor;caused by sarcoidosis
mphages producing active D3
Q1316:Vitamin E
antioxidant protects RBCs against hemolysis;increased
fragility of RBCs
Q1317:Vitamin K
synthesis of clotting factors II; VII; IX; X and Proteins
C/S;synth'd by intestinal flora;deficiency seen in broad-
spectrum ABx; warfarin use;inc PT; PTT; INR;Neonatal
hemorrhage
Q1318:Ethanol Metabolism
alcohol and acetaldehyde dehydrogenase produce NADH and
acetate;excess NADH shunts pyruvate away from
gluconeogenesis to lactate;leads to hypoglycemia and FA
synth (fatty liver)
Q1319:Where does calcitonin work?
Osteoclast inhibits bone reabsorption
Q1320:What receptor does PTH hook on?
Osteoblast
Q1321:What does PTH releases?
IL-1 Osteoclast activating factor
Q1322:What keeps a check on IL-1?
Testosteron and Estrogen
Q1323:Why do women get osteoporosis?
Menopausal Women; breaking bone down since IL-1 is not
checked
Q1324:What enzyme is in the S.E.R. when you have increase
P-450?
Gamma-glutamyl transferase;- key tests for alcoholics
Q1325:Why does renal dz causes vitamin D deficiency?
Caused by Diabetes Mellitus
no alpha-1-hydroxylase
Q1326:Vitamin D from the store; what happens to it before it
becomes activate?
25-OH D activated in the liver;- 1;25 OH D in kidneys by
alpha-hydroxylase
Q1327:Hypervitaminosis D? What happens?
Increase Calcium (hypercalcemia); more Calcium in urine
causing Stones.
Q1328:Type I Vitamin D is what?
Missing alpah-1-hydroxylase
Q1329:What is wrong with Type II Vitamin D deficiency?
Bad receptors
Q1330:What is vitamin E main fuction?
- Prevent lipid peroxidation of cell membranes;- protect
membrane from breaking down by phospholipid A;-
neutralizes oxidis LDL (makes it less injurious); i.e.
cardioprotective
Q1331:Who gets Vitamine E deficiency?
Cystic Fibrosis Patients
Q1332:Does vitamin E deficiency cause hemolytic anemia?
Yes! Susceptible to membrane damage (radical)
Q1333:Does vitamin E help myelin?
Yes! Problems neurologicly since they disrupt the membranes
in the brain. Spinalcerebellar Dz
Q1334:What vitamin enhances the activity of warfarin?
Vitamin E excess!
Q1335:What changes k2(inactive) to k1?
epoxide reductase ;k1 gamma carboxylates activates factors II;
VII; IX; X;hydroxylates proline and lisine;activates them so
they are fucntional
Q1336:Warfarin does what?
Blocks epoxide reductase; all vitamin K is K2 (inactive)
Q1337:Vitamine K deficiency?
Prolong Antibiotics;Poor Diets;New Borns
Q1338:What amino acids stimulate Growth Hormone (GH)?
Arganine and Histidine(Ornithine)
Q1339:Symptoms of hypocortisolism?
fasting hypoglycemia and fatigue;ACTH low;Corisol Low
Q1340:Central Diabetes Insipidus;Causes?
Car accident
Q1341:Where is it made?
Superoptic/Paraventricular Nucleus of hypothalamus
Q1342:Where is ADH stored?
Vassopressin (ADH) is stored in the posterior hypofisis
Q1343:Signs and Symptoms of SIADH?
-Thrist (polydypsia);- polyuria
Q1344:Mechanisms of polyuria in DM?
Osmotic Diuresis
Q1345:Where is HMG CoA Synthase seen?
Ketogenesis;Acetyl CoA ----> HMG CoA
Q1346:What is involved in ketogenolysis (extrahepatic)?
Hydroxybutyrate ----> Acetoacetate ---> Acetoacetyl CoA --
--> Acetyl CoA ----> Citric Acid Cycle
Q1347:What is perceived as fruity odor?
Acetone in the blood;Seen in prolonged fasting
Q1348:What amino acid is used in starvation?
Alanine
Q1349:How long does glycogen storages last when you are
fasting?
24 hours
Q1350:When does protein degradation start?
after 12-24 hours
Q1351:How many days does it take for fat to become the
predominant source of glucose?
1 week is the breaking point
Q1352:When is the highest fat source for glucose? Week?
When are ketones at the highest level?
After the 3rd week
Q1353:G6PDH;Reactant and product
Reactant: Glucose-6-P;Product: 6-phosphogluconate
Q1354:G6PDH;cofactor
NADP;(rxn: NADP ---> NADPH)
Q1355:G6PDH;part of what pathway?
Hexose monophosphate shunt
Q1356:G6PDH;induced by
insulin
Q1357:G6PDH;activated by
NADP (decrease in NADPH/NADP ratio)
Q1358:G6PDH deficiency;inheritence
XLR
Q1359:G6PDH;major function
Generate NADPH for anabolic purposes (EG: FA synthesis);
antimicrobial killing and protection of cells from reactive
oxygen species
Q1360:Describe the role of NADPH in protecting against
ROS (particularly in RBCs)
NADPH reduces oxidized Glutathionine (G-S-S-G) back to
its reduced form (2 GSH). GSH allows the enzyme
Glutathionine peroxidase to breakdown H202
Q1361:G6PDH deficiency;3 key features
-immunodeficiency (in severe disease);- Heinz bodies;-
hemolytic anemia
Q1362:G6PDH deficiency;stressors that can acute hemolytic
anemia
-ifn;-drugs (sulfas; chloroquine);-fava beans
Q1363:G6PDH deficiency;Why can this condition lead to
immunodeficiency?
NAPDH oxidase generates bactericidal superoxide. NADPH
deficiency inhibits this function.
Q1364:CGD/NADPH deficiency;How can the diagnosis be
confirmed
a NEGATIVE nitroblue tetrazolium test
Q1365:HMG CoA reductase;location
ER
Q1366:HMG CoA reductase;activated by?
insulin
Q1367:HMG CoA reductase;;enzyme ACTIVITY is
inhibited by
-glucagon;-statins
Q1368:HMG CoA reductase;reaction
HMG-CoA is converted to mevalonate
Q1369:HMG CoA reductase;cholesterol effect on the enzyme
Increased hepatic cholesterol represses expression and
enhances degradation
Q1370:HMG CoA reductase;How does inhibition by statin
drugs decrease cholesterol levels?
Inhibition by statins both decreases de novo synthesis and
enhances hepatic clearence of serum cholesterol by increased
LDLR expression
Q1371:HMG CoA reductase;statins are _________ inhibitors
competitive
Q1372:DHFR;reaction it catalyzes
Folate---->DHF--->THF
Q1373:DHFR;eukaryotic inhibitior
methotrexate
Q1374:DHFR;prokaryotic inhibitors (2)
-Trimethoprim;-pyrimethamine
Q1375:DHFR;Most important downstream consequence of
inhibition?
A block of DHFR function ultimately prevent synthesis of
thymidylate (thymidylate synthase is folate dependent)
Q1376:how can ornithine transcarbamylase deficiency be
distinguished from orotic aciduria
OTCD has hyperammonia and low BUN;Orotic aciduria has a
normal BUN
Q1377:Orotic Aciduria;enzymatic causes
- orotic acid phosphoribosyltransferase deficiency;OR;- OMP
decarboxylase deficiency
Q1378:UDP-Glucuronyl transferase;key reaction
Bilirubin + glucoronide ----> bilirubin-diglucoronide
Q1379:UDP-Glucuronyl transferase;significance in neonates
UDP-Glucuronyl transferase is the last enzyme expressed in
infants. Thus; neonates have increased susceptibility to
jaundice and kernicterus
Q1380:Dubin-Johnson syndrome;characterized by
black pigment in the liver due to impaired excretion of direct
bilirubin
Q1381:Crigler-Najjar syndrome;define
a severe UDP-Glucuronyl transferase deficiency
Q1382:Gilbert Syndrome;define
a benign UDP-Glucuronyl transferase deficiency
Q1383:gamma-glutamyl carboxylase;rxn:
glu -----> gamma gultamic acid
Q1384:gamma-glutamyl carboxylase;function and substrate of
this rxn:
gamma carboxylation of factors II; VII; IX; X and Protein
C&S generates Ca binding sites.
Q1385:gamma-glutamyl carboxylase;dependent on?
vit K
Q1386:gamma-glutamyl carboxylase;inhibited by
warfarin and dicoumarol
Q1387:Warfarin;does this drug inhibit in vitro clotting?
NO; warfarin's effect (vit K reductase inhibition) prvents in
vivo clotting by blocking clotting factor synthesis but has no
effect on existant factors.
Q1388:gamma-glutamyl carboxylase;this reaction catalyzes
what type of modification
cotranslational modification of clotting factors
Q1389:Homocysteine methyltranferase;rxn:
homocysteine ---> methionine
Q1390:Homocysteine methyltranferase;cofactors:
-N5-methyl THF;-methylcobalamin (B12)
Q1391:mild homocysteinuria is associated with deficiencies is
what vitamins
folate;B12;B6 (cystathionine synthase rxn)
Q1392:mild homocysteinuria;symptoms
-DVT;-stroke;-atherosclerosis
Q1393:how can B12 deficiency be distinguished from folate
deficiency?
B12 deficiency is associated with methylmalonic aciduria
Q1394:name the 3 "ABC" carboxylases
(ATP; Biotin; Co2);-propionyl-CoA carboxylase;-acetyl-CoA
carboxylase;-pyruvate carboxylase
Q1395:Methylmalonyl-CoA mutase;rxn:
Methylmalonyl-CoA ---> succinyl-CoA
Q1396:Methylmalonyl-CoA mutase;cofactor:
adeonsylcobalamin (B12)
Q1397:Methylmalonyl-CoA mutase;deficiency results in:
Methylmalonylic aciduria and peripheral neuropathy
Q1398:Lesch-Nyhan Syndrome;inheritence:
XLR
Q1399:Lesch-Nyhan Syndrome;genetic cause:
HGPRT deficiency
Q1400:Lesch-Nyhan Syndrome;symptoms:
-spastic cerebral palsy;-self-mutilation;-hyperuricemia;-early
death
Q1401:6-Mercaptopurine is activated by
HGPRT
Q1402:Lesch-Nyhan Syndrome;pathophysiology
Loss of the salvage pathways results in shunting of
Hypoxanthine and guanine to the excretion
pathway;Furthermore; loss of feedback inhibition of PRPP
admidotransferase results in additional purine synthesis;Thus;
HGPRT deficiency leads to crippling excesses of urate
Q1403:HGRPT;rxn:
hypoxanthine/guanine -----> IMP/GMP
Q1404:Branched-chain Ketoacid DH;cofactors:
TPP;Lipoic acid;CoA;FAD;NAD
Q1405:Branched-chain Ketoacid DH;critical in the
metabolism of
Valine;Leucine;Isoleucine
Q1406:Branched-chain Ketoacid DH;associated disease:
maple serup urine disease
Q1407:maple serup urine disease;symptoms:
-urine has a maple odor;-mental retardation;-abnormal muscle
tone;-ketosis;-coma;death
Q1408:name the substances that enter the propionyl-CoA
pathway
(VOMIT);Valine;Odd chain
FA;Methionine;Isoleucine;Threonine
Q1409:Pyruvate DH;cofactors
TPP;Lipoic Acid;CoA;FAD;NAD
Q1410:Pyruvate DH;rxn:
pyruvate------> acetyl CoA
Q1411:Pyruvate DH;deficient in what population
(consequence)
alcoholics due to thiamine deficiency (results in lactic
acidosis)
Q1412:Pyruvate DH;inhibited by
acetyl-CoA
Q1413:PKU;genetic causes
phenylalanine hyxdroxylase
deficiency;OR;tetrahydrobiopterin deficiency
Q1414:Aspartame is contraindicated in what condition
PKU
Q1415:PKU;untreated symptoms
-pale skin and white hair;-mental retardation;-loss of motor
control;-musty; mousy odor
Q1416:PKU;-pathophysiology
elevated phenylalanine has a neurotoxic effect
Q1417:Phenylalanine hydroxylase;rxn
phe ---> tyrosine
Q1418:MCAD;function
oxidation of medium chain FA
Q1419:MCAD deficiency;symptoms
-fasting hypoglycemia;- NO KETONES;-C8-10 acyl
carnitines in blood;-DICARBOXYLIC ACIDEMIA
Q1420:Dicarboxylic acidemia is pathognomonic for
MCAD deficiency
Q1421:Topoisomerase II;inhibited by what drug class in
prokaryotes
#NAME?
Q1422:Topoisomerase II;function
relieves positive supercoiling during DNA replication by
introducing negative supercoils
Q1423:Topoisomerase II;target for what drugs in eukaryotes
etoposide;teniposide
Q1424:Excision endonuclease;function
removal of intrastrand thymine dimers
Q1425:Xerderma pigmentosum;defective gene
excision endonuclease
Q1426:Xerderma pigmentosum;symptoms
extreme UV sensitivity;excessive freckling;multiple skin
cancers;corneal ulcerations
Q1427:Carbamoyl-P Synthetase (CPS-I) deficiency
- urea cycle defect;- condition: type I hyperammonemia;-
metabolites accumulated: ammonia; glutamine; alanine
Q1428:Ornithine transcarbamoylase (OTC) deficiency
- urea cycle defect;- condition: type II hyperammonemia;-
metabolites accumulated: ammonia; glutamine; orotate
Q1429:Argininosuccinate synthetase deficiency
- urea cycle defect;- condition: citrullinemia;- metabolites
accumulated: citrulline
Q1430:Argininosuccinate lyase deficiency
- urea cycle defect;- condition: argininosuccinic aciduria;-
metabolites accumulated: argininosuccinate
Q1431:Arginase deficiency
- urea cycle defect;- condition: hyperargininemia;- metabolites
accumulated: arginine
Q1432:Maple Syrup Urine Disease
- AR defect in branched-chain ketoacid dehydrogenase;- high
plasma & urine levels of branched-chain AA (leucine; valine;
isoleucine) and their corresponding alpha-keto acids and
alpha-hydroxyacids;- urine odor of maple syrup or burnt
sugar;- brain damage
Q1433:strictly ketogenic AA
leu; lys;degraded tp acetyl-CoA or acetoacetyl-CoA --> both
converted to ketone bodies
Q1434:both ketogenic + glucogenic AA
ile; phe; tyr; trp
Q1435:strictly glucogenic AA
all others
Q1436:7 metabolic intermediates derived from AA
acetyl-CoA;acetoacetyl-CoA;oxaloacetate;fumarate;succinyl-
CoA;alpha-keto glutarate;propionyl-CoA (converted to
succinyl-CoA)
Q1437:AA that form pyruvate
glycine; alanine; cysteine; serine; threonine; tryptophan
Q1438:AA that form acetyl-CoA
leucine; isoleucine
Q1439:AA that form acetoacetyl-CoA
leucine; lysine;phenylalanine; tryptophan; tyrosine
Q1440:AA that form oxaloacetate
asparagine; aspartate
Q1441:AA that form fumarate
tyrosine; phenylalanine
Q1442:AA that form propionyl-CoA (then converted to
succinyl-CoA)
isoleucine; methionine; threonine; valine
Q1443:AA that form glutamate (then converted to alpha-
ketoglutarate)
proline; arginine; histidine; glutamine
Q1444:propionic aciduria
2/2 deficiency of biotin; propionyl-CoA carboxylase;
holocarboxylase synthase; or the enzyme that covalentloy
attaches biotin to all carboxylases (in last case; additional
organic acids accumulate)
Q1445:methylmalonic aciduria
2/2 deficiency in vitamin B12 or defect in methylmalonyl-
CoA mutase;some pts respond well to megadose of vit B12
Q1446:pyridoxal phosphate
derivative of pyridoxine (vit B6);acts as coenzyme for all
transaminases
Q1447:CPS-1 activation
high protein diet --> glutamate accumulation --> increase in
NAG --> CPS-1 activation
Q1448:arginase found only in
brain; liver; kidney
Q1449:amino groups in muscle
transferred to pyruvate to form alanine --> dumped into
circulation --> picked up by liver; where it is converted back
to pyruvate
Q1450:liver uses pyruvate for
gluconeogenesis
Q1451:liver uses amino groups for
urea synthesis
Q1452:phenylketonuria (PKU)
deficiency in phenylalanine hydroxylase or dihydrobiopterin
reductase;buildup of phenylalanine; phenylpyruvate;
phenylacetate; phenyllactate in blood and urine;**tyrosine
becomes an essential AA**
Q1453:PKU Sx
musty body odor;MR
Q1454:dihydrobiopterin reductase deficiency
PKU + impairment of catecholamine and serotonin synthesis
Q1455:PKU Tx
remove phenylalanine from diet (incl NutraSweet)
Q1456:NutraSweet
dipeptide containing phenylalanine + aspartic acid
Q1457:precursor for tyrosine
phenylalanine
Q1458:sulfur for cysteine synthesis comes from
methionine
Q1459:if phenylalanine deficient in diet
tyrosine becomes essential AA
Q1460:if methionine essential in diet
cysteine becomes essential AA
Q1461:elevated plasma homocysteine is risk factor for
coronary heart disease;independent of risk associated with
elevated cholesterol
Q1462:homocystinuria
large amts. homocystine in urine;acquired or inherited;most
often seen in children with FTT; lens displacement
Q1463:causes of homocystinuria
deficiency in pyridoxine; folate; or vitamin B12;OR;inherited
defect in either cystathionine synthase or methionine
synthase;all above result in accumulation of homocysteine;
which is readily oxidized to its disulfide form; homocystine
Q1464:homocystine
disulfide form of homocysteine
Q1465:cysthathionuria
2/2 deficiency in pyridoxine or from genetic defect in
cystathionase;large amts. cystathionine found in urine + blood
Q1466:Parkinson dz: cause
decreased dopamine in substantia nigra
Q1467:Parkinson prevalence
1% of pop > 55yrs
Q1468:Parkinson Sx
tremors; postural instability; rigidity; bradykinesia
Q1469:Parkinson Tx
L-dopa + carbidopa
Q1470:Carbidopa
decreases extra-CNS effects of L-dopa;selectively inhibits
aromatic acid decarboxylase outside CNS;does not cross BBB
so does not inhibit conversion of L-dopa --> dopamine
Q1471:carcinoid tumors
neoplastic transformation of enterochromaffin cells;secrete
excess serotonin;high levels of 5-HI in urine
Q1472:nitroglycerin and other angina tx
act in part to spontaneously generate nitric oxide
Q1473:porphyria
any abnormality in pathway of heme synthesis;block early in
pathway: intermediates buildup & are excreted in urine;block
late in pathway: excreted in urine + feces; accumulate in skin
Q1474:lead poisoning and heme synthesis
lead poisoning can be considered acquired porphyria b/c
inhibits ALA DEHYDRATASE and HEME SYNTHASE
(FERROCHELATASE)
Q1475:4 broad causes of hyperbilirubinemia
massive hemolysis;block in heme catabolism;bile
obstruction;liver damage;always jaundice
Q1476:anabolism
build stuff (need energy)
Q1477:What a.a. is the smallest?
Glycine
Q1478:What a.a. is involved in gluconeogenesis?
Alanine
Q1479:What a.a. are branched?
Valine;Leucine;Isoleucine;I Saw Lucy and Val(erie) Like syrup
Q1480:What a.a. bends proteins?
Proline
Q1481:What a.a. is converted to tyrosine?
Phenylalanine
Q1482:What is made with Tyrosine? 3 things;
1) Catecholamines (NE; Epi);2) Thyroid T3/T4;3) Melanin
Q1483:What causes Hartnup Dz?
Tryptophan ;tryp up the esophagus
Q1484:What coenzyme is needed to degrade tryptophan?
Niacin
Q1485:What deffect do you see when you have Niacin
defiency?
Pellagra
Q1486:What are the symptoms of pellagra?
Diarrhea;Dermatitis;Dementia
Q1487:What are the basic a.a.?
HAL;Histidine;Alaline;Lysine
Q1488:What two a.a. are acidic?
Aspartate;Glutamate;aspartic acid; glutamic acid
Q1489:What a.a. have OH in htem?
Serine;Threonine
Q1490:Valine is a branched a.a. what happens glutamate is
changed to val?;glutamate ----> valine
Sickle Cell Anemia;Missense mutation;anemia sickle
Q1491:What happens when there is a deletion of
phenylalanine at position 508?
Cystic Fibrosis;Chromosome seven;problem in folding
proteins
Q1492:Why is cysteine important?
Because it produces S-thiol and is the Tx for Acetaminophen
Toxicity;NO+ guanylate cyclase increasing cGMP
Q1493:Why is methionine important?
Makes AUG; SAM;initiate transcription;methylates in SAM
Q1494:WHat is asparagine famous for?
N.glycosylation in E.R. mannose to protein
Q1495:WHat is glutamine used for?
NH3 donor;Amonia
Q1496:WHat a.a. is need in positve nitrogen balance?
Arginine
Q1497:When does positive nitrogen balance occur?
Growth;Pregnancy;Recovery from injury or surgery;Recovery
from Negative Nitrogen Balance
Q1498:When does negative nitrogen balance?
Protein Malnutrition (Kwashiorkor);Starvation
(Marasmus);DM uncontrolled;Infection;Diet Defficient of
essential a.a.
Q1499:What is the enzyme deficiency in Von Gierke's disease
(Type I)?
Glucose-6-phosphatase
Q1500:What is the enzyme deficiency in Pompe's disease
(Type II)?
Lysosomal alpha-1;4-glucosidase
Q1501:What is the enzyme deficiency in Cori's disease (Type
III)?
Debranching enzyme alpha-1;6-glucosidase
Q1502:What is the enzyme deficiency in McArdle's disease
(Type V)?
Skeletal muscle glycogen phosphorylase
Q1503:What is the enzyme deficiency in Andersen's disease
(Type IV)?
Glycogen branching enzyme amylo-1;4-1;6-transglucosidase
Q1504:What is the enzyme deficiency in Tarui's disease
(Type VII)?
Skeletal muscle PFK-1
Q1505:What are the findings in Von Gierke's disease (Type I)
(6)?
Severe fasting hypoglycemia;Increased glycogen in liver;Thin
extremities; chubby facies;Fatty liver;Renal disease;Growth
retardation; delayed puberty
Q1506:What are the findings in Pompe's disease (Type II)
(4)?
Cardiomegaly;Early death;Normal blood glucose;"Trashes the
pump" (heart; liver; muscle)
Q1507:What are the findings in Cori's disease (Type III) (3)?
Hypoglycemia;Failure to thrive;Hepatomegaly
Q1508:What are the findings in McArdle's disease (Type V)
(3)?
Increased glycogen in muscle (can't break it down);Paimful
cramps;No rise in lactate w/ exercise
Q1509:What are the findings in Tarui's disease (Type VII)
(4)?
Like McArdle's;Nausea and vomiting;Acute exacerbation after
high-carb meal;Hyperuricemia and hyperbilirubinemia
Q1510:What are the findings in Andersen's disease (Type IV)
(4)?
Glycogen with unbranched chains in tissue;Resembles
amylopectin;Failure to thrive;Hepatosplenomegaly
Q1511:How do you treat Von Gierke's disease (Type I)?
Nocturnal glucose; uncooked corn starch
Q1512:What is/are the MAJOR REGULATORY ENZYME
of the citric acid cycle?
Citrate synthase
Q1513:What is/are the MAJOR REGULATORY ENZYME
of glycolysis?
Phosphofructokinase-1
Q1514:What is/are the MAJOR REGULATORY ENZYME
of pyruvate oxidation?
Pyruvate dehydrogenase
Q1515:What is/are the MAJOR REGULATORY ENZYMES
of gluconeogenesis (3)?
Pyruvate carboxylase;Phosphoenolpyruvate
carboxykinase;Fructose-1;6-bisphosphatase
Q1516:What is/are the MAJOR REGULATORY ENZYME
of glycogenesis?
Glycogen synthase
Q1517:What is/are the MAJOR REGULATORY ENZYME
of glycogenolysis?
Glycogen phosphorylase
Q1518:What is/are the MAJOR REGULATORY ENZYME
of the pentose phosphate pathway?
Glucose-6-phosphate dehydrogenase
Q1519:What is/are the MAJOR REGULATORY ENZYME
of cholesterol synthesis?
HMG-CoA reductase
Q1520:What is/are the MAJOR REGULATORY ENZYME
of lipogenesis?
Acetyl-CoA carboxylase
Q1521:What are the major ACTIVATORS of
phosphofructokinase-1 (3)?
AMP;Fructose-2;6-bisphosphate (liver);Fructose-1;6-
bisphosphate (muscle)
Q1522:What are the major ACTIVATORS of pyruvate
dehydrogenase (4)?
CoA;NAD;ADP;Pyruvate
Q1523:What is the major ACTIVATOR of pyruvate
carboxylase and phosphoenolpyruvate carboxykinase?
Acetyl-CoA
Q1524:What is the major ACTIVATOR of fructose-1;6-
bisphosphatase?
cAMP
Q1525:What are the major ACTIVATORS of glycogen
phosphorylase (2)?
cAMP;Ca2+ (muscle)
Q1526:What is the major ACTIVATOR of glucose-6-
phosphate dehydrogenase?
NADP+
Q1527:What is the major ACTIVATOR of acetyl-CoA
carboxylase
Citrate
Q1528:What are the INHIBITORS of citrate synthase (2)?
ATP;Long-chain acyl-CoA
Q1529:What are the INHIBITORS of phosphofructokinase-1
(3)?
Citrate (fatty acids; ketone bodies);ATP;cAMP
Q1530:What are the INHIBITORS of pyruvate
dehydrogenase (3)?
Acetyl-CoA;NADH;ATP (fatty acids; ketone bodies)
Q1531:What is the INHIBITOR of pyruvate carboxylase and
phosphoenolpyruvate carboxykinase?
ADP
Q1532:What are the INHIBITORS of fructose-1;6-
bisphosphatase (2)?
AMP;Fructose-2;6-bisphosphatase
Q1533:What are the INHIBITORS of glycogen synthase (3)?
Phosphorylase (liver);cAMP (muscle);Ca2+ (muscle)
Q1534:What is the INHIBITOR of glucose-6-phosphate
dehydrogenase?
NADPH
Q1535:What are the INHIBITORS of acetyl-CoA
carboxylase (2)?
Long-chain acyl-CoA;cAMP
Q1536:What are the INHIBITORS of HMG-CoA reductase
(2)?
Cholesterol;cAMP
Q1537:What two rate-limiting enzymes are INHIBITED by
long-chain acyl-CoA?
Citrate synthase;Acetyl-CoA carboxylase
Q1538:What two rate-limiting enzymes are ACTIVATED by
cAMP?
Fructose-1;6-bisphosphatase;Glycogen phosphorylase
Q1539:What is Type I Familial Dyslipidemia?
hyperchylomicronemia
Q1540:What is Type IIa Familial Dyslipidemia?
hypercholesterolemia
Q1541:What is Type IIb Familial Dyslipidemia?
combined hyperlipidemia
Q1542:What is Type III Familial Dyslipidemia?
dysbetalipoproteinemia
Q1543:What is Type IV Familial Dyslipidemia?
hypertriglyceridemia
Q1544:What is Type V Familial Dyslipidemia?
mixed hypertriglyceridemia
Q1545:What is the INHERITANCE of Type IIa Familial
dyslipidemia? (hypercholesterolemia)
autosomal dominant
Q1546:What is INCREASED in Type I Familial
Dyslipidemia?
chylomicrons
Q1547:What is INCREASED in Type IIa Familial
Dyslipidemia (hypercholesterolemia)?
LDL
Q1548:What is INCREASED in Type Iib Familial
Dyslipidemia (combined hyperlipidemia)?
LDL; VDL
Q1549:What is INCREASED in Type III Familial
Dyslipidemia (dysbetalipoproteinemia)?
IDL; VLDL
Q1550:What is INCREASED in Type IV Familial
Dyslipidemia (hypertriglyceridemia)?
VLDL
Q1551:What is INCREASED in Type V Familial
Dyslipidemia (mixed hypertriglyceridemia)?
VLDL; chylomicrons
Q1552:Type I hyperchylomicronemia ELEVATED BLOOD
LEVELS?
TG; cholesterol
Q1553:Type IIa hypercholesterolemia ELEVATED BLOOD
LEVELS?
cholesterol
Q1554:Type IIb combined hyperlipidemia ELEVATED
BLOOD LEVELS?
TG; cholesterol
Q1555:Type III dysbetalipoproteinemia ELEVATED
BLOOD LEVELS?
TG; cholesterol
Q1556:Type IV hypertriglyceridemia ELEVATED BLOOD
LEVELS?
TG
Q1557:Type V mixed hypertriglyceridemia ELEVATED
BLOOD LEVELS?
TG; cholesterol
Q1558:Type I hyperchylomicronemia
PATHOPHYSIOLOGY?
Lipoprotein lipase deficiency; or altered apolipoprotein C-II
(co-factor for lipoprotein lipase)
Q1559:Type IIa hypercholesterolemia
PATHOPHYSIOLOGY?
DECREASE LDL receptors
Q1560:Type IIb combined hyperlipidemia
PATHOPHYSIOLOGY?
hepatic OVERPRODUCTION of VLDL
Q1561:Type III dysbetalipoproteinemia
PATHOPHYSIOLOGY?
ALTERED apolipoprotein E
Q1562:Type IV hypertriglyceridemia
PATHOPHYSIOLOGY?
hepatic OVERPRODUCTION of VLDL
Q1563:Type V mixed hypertriglyceridemia
PATHOPHYSIOLOGY?
INCREASE production/DECREASE clearance of VLDL and
chylomicrons
Q1564:Vit A too much
arthralgias; fatigue; headache; skin changes; sore throat;
alopecia
Q1565:dry beriberi
polyneuritis; muscle wasting
Q1566:wet beriberi
dilated cardiomyopathy; edema
Q1567:B2 deficiency
angular stomatitis; cheilosis; corneal vascularization
Q1568:B3 deficiency
pellagra; diarrhea; dermatitis; dementia; beefy glossitis.
Q1569:causes of pellegra
hartnup disease (dec tryptophan absorbtion); malignant
carcinoid syndrome (increased trypophan metabolism); and
INH (decreased B6)
Q1570:B5 deficiency
dermatitis; enteritis; alopecia; adrenal insufficiency.
Q1571:B6 deficiency
convulsions; hyperirritability; peripheral neuropathy.
Q1572:Sources of B6 deficiency
INH; oral contraceptives.
Q1573:B12 function
Homocysteine + N-methyl THF (B12) to Methionine +
THF; Methylmalonyl-CoA (B12) to Succinyl-CoA
Q1574:causes of B12 defiency
malabsorption (sprue; enteritis; Diphyllobothrium latum);
lack of IF (pernicious anemia); or absence of terminal ileum
(chron's)
Q1575:folic acid precursor in bacteria and use
PABA - sulfa drugs and dapsone are PABA analogs.
Q1576:biotin deficiency
dermatitis; enteritis
Q1577:causes of biotin defiency
antibiotic use; ingestion of raw eggs
Q1578:vit c deficiency
scurvy - swollen gums; bruising; anemia; poor wound healing.
Q1579:vit c 3 mech
hydroxylation of proline and lysine in collagen synthesis;
facilitates iron absorption by keeping iron in Fe+2 reduced
state; necessary as a cofactor for Dopamine to NE
Q1580:Types of Vit D
D2 - ergocalciferol; in milk; D3 - cholecalciferol; sun exposed
skin; 25-OH D3 - storage form; 1;25 (OH)2 D3 active form.
Q1581:Vit D def
Rickets kids (bending bones); osteomalacia in adults (soft
bones) and hypocalcemic tenatny
Q1582:Vit D function
increases Ca and Phosphate aborption.
Q1583:Vit D excess
Hypercalcemia; loss of appetitie; stupor. Sarcoid - epitheliod
macrophages convert Vit D into its active form.
Q1584:Vit E def
increases fragility of EEErythrocytes; neurodysfunction
Q1585:Vit E function
antioxidant (protects erythrocytes from hemolysis.
Q1586:sxs and causes of Vit K def
neonatal hemorrhage with increased PT/aPTT but normal
bleeding times - sterile intestine cant make Vit K
Q1587:Vit K dependent factors
2;7;9;10
Q1588:Vit K antagonist
warfarin
Q1589:Vit K function
Catalyzes (gamma)-carboxylation of glutamic acid residues on
various proteins concerned with blood clotting.
Q1590:Zinc deficiency
Delayed wound healing; hypogonadism; dec adult hair; may
predispose to alcoholic cirrhosis.
Q1591:ETOH metabolism
ETOH (alcohol dehydrogenase) to acetaldehyde (acetaldehyde
dehydrogenase) to acetate. Both require NAD+ which goes to
NADH
Q1592:ETOH met rate limiting reagent
NAD+
Q1593:alcohol dehydrogenase kinetics
zero order
Q1594:Antabuse mech
inhibits acetaldehyde dehydrogenase
Q1595:EtOH hypoglycemia
ETOH metabolism increases NADH/NAD+ ratio in liver -
pyruvate to lactate and OAA to malate - inhibits
gluconeogensis and thus hypoglycemia - fatty acid synthesis -
hepatocellular steatosis (hepatic fatty change)
Q1596:Kwashiorkor
MEAL - malabsorption; edema; anemia; liver (fatty) - protein
malnutrition
Q1597:Marasmus
energy malnutrition - tissue and muscle wasting; loss of
subcut fat; variable edema
Q1598:Von Gierke's disease;Deficient enzyme
glucose-6-phosphatase
Q1599:Von Gierke's disease;Findings
Type I glycogen storage disease;Severe fasting hypoglycemia;
increased glycogen in liver; increased blood lactate;
hepatomegaly
Q1600:Pompe's disease;Deficient enzyme
Lysosomal alpha-1-4-glucosidase (acid maltase)
Q1601:Pompe's disease;Findings
Type II glycogen storage disease;Cardiomegaly and systemic
findings leading to early death;Pompe's trashes the Pump
(heart; liver and muscle)
Q1602:Cori's disease;Deficient enzyme
Debranching enzyme; alpha-1;6-glucosidase
Q1603:Cori's disease;Findings
milder form of type I (Von Gierke's disease) with normal
blood lactate levels;Gluconeogenesis is intact
Q1604:McArdle's disease;Deficient enzyme
Skeletal muscle glycogen phosphorylase
Q1605:McArdle's disease;Findings
increased glycogen in muscle; but cannot break it down;
leading to painful muscle cramps; myoglobinuria with
strenuous exercise;McArdles: think MUSCLE
Q1606:Fabry's disease;Deficient enzyme
Sphingolipidoses;alpha-galactosidase A;X-linked recessive!!!
Q1607:Fabry's disease;accumulated substrate
ceramide trihexoside;X-linked recessive!!!
Q1608:Fabry's disease;Findings
peripheral neuropathy of hands/feet; angiokeratomas;
cardiovascular/renal disease;X-linked recessive!!!
Q1609:Gaucher's disease;Deficient enzyme
beta-glucocerebrosidase!!;AR
Q1610:Gaucher's disease;Accumulated substrate
Glucocerebroside
Q1611:Gaucher's disease;Findings
AR!!;hepatosplenomegaly; aseptic necrosis of the femur;
bone crises; Gaucher's cells (macrophages that look like
crumpled paper)
Q1612:Niemann-Pick disease;Deficient enzyme
Sphingomyelinase
Q1613:Niemann-Pick disease;Accumulated substrate
Sphingomyelin;AR
Q1614:Niemann-Pick disease;Findings
progressive neurodegeneration; hepatosplenomegaly; cherry-
red-spot (on macula); foam cells;AR!
Q1615:Tay-Sachs disease;Deficient enzyme
hexosaminidase
Q1616:Tay-Sachs disease;Accumulated substrate
GM2 ganglioside;AR
Q1617:Tay-Sachs disease;Findings
progressive neurodegeneration; developmental delay; cherry-
red spot; lysosomes with onion skin!!
Q1618:Krabbe's disease;Deficient enzyme
Galactocerebrosidase;AR
Q1619:Krabbe's disease;Accumulated substrate
galactocerebroside
Q1620:Krabbe's disease;Findings
peripheral neuropathy; developmental delay; optic atrophy;
globoid cells
Q1621:Metachromic leukodystrophy;Deficient enzyme
Arylsulfatase A
Q1622:Metachromic leukodystrophy;Accumulated substrate
Cerebroside sulfate
Q1623:Metachromic leukodystrophy;Findings
Central and peripheral demyelination with ataxia; dementia
Q1624:Mucopolysaccharidoses
Hurler's syndrome and Hunter's syndrome
Q1625:Hurler's syndrome;Deficient enzyme
alpha-L-iduronidase
Q1626:Hurler's syndrome;accumulated substrate
heparan sulfate; dermatan sulfate
Q1627:Hurler's syndrome;Findings
developmental delay; gargoylism; airway obstruction; corneal
clouding; hepatosplenomegaly
Q1628:Hunter's syndrome;Deficient enzyme
Mucopolysaccharidoses;iduronate sulfatase
Q1629:Hunter's syndrome;Accumulated substrate
heparan sulfate; dermatan sulfate
Q1630:Hunter's syndrome;Findings
XR!!!;mild-Hurler's (developmental delay; gargoylism; airway
obstruction; corneal clouding; hepatosplenomegaly) with
aggressive behavior; NO corneal clouding
Q1631:What are the FINDINGS in Fabry's disease?
1. peripheral neuropathy of hands/feet;2. angiokeratomas;3.
cardiovascular/renal disease
Q1632:What are the FINDINGS in Gaucher's disease?
1. hepatosplenomegaly;2. asceptic necrosis of femur;3. bone
crises;4. Gaucher's cells (macrophages)
Q1633:What are the FINDINGS in Niemann-Pick disease?
1. progressive neurodegeneration;2. hepatosplenomegaly;3.
cherry red spot (on macula)
Q1634:What are the FINDINGS in Tay-Sachs disease?
1. progressive neurodegeneration;2. developmental delay;3.
cherry-red spot;4. lysozymes with onion skin
Q1635:What are the FINDINGS in Krabbe's disease?
1. peripheral neuropathy;2. developmental delay;3. optic
atrophy
Q1636:What are the FINDINGS in Metachromatic
leukodystrophy disease?
1. Central and peripheral demyelination;2. ataxia;3. dementia
Q1637:What are the FINDINGS in Hurler's Syndrome?
1. Developmental delay;2. gargoylism;3. airway obstruction;4.
corneal clouding;5. hepatosplenomegaly
Q1638:What are the FINDINGS in Hunter's Syndrome?
1. aggressive behavior;2. NO corneal clouding;3. Mild
Hurler's;4. developmental delay;5. gargoylism;6. airway
obstruction;7. hepatosplenomegaly
Q1639:What is the DEFICIENT ENZYME in Fabry's
disease?
alpha-galactosidase A
Q1640:What is the DEFICIENT ENZYME in Gaucher's
disease?
beta-glucocerebrosidase
Q1641:What is the DEFICIENT ENZYME in Niemann-Pick
disease?
sphingomyelinase;"NO MAN PICKS (NIEMANN-PICK)
his nose with his SPHINGER (SPHINGOMYELINASE)."
Q1642:What is the DEFICIENT ENZYME in Tay-Sach's
disease?
Hexosaminidase A;;"Tay-SaX (TAY-SACHS) lacks
heXosaminidase."
Q1643:What is the DEFICIENT ENZYME in Krabbe's
disease?
beta-galactosidase
Q1644:What is the DEFICIENT ENZYME in Metachromatic
Leukodystrophy disease?
Arylsulfatase A
Q1645:What is the DEFICIENT ENZYME in Hurler's
syndrome?
alpha-L-iduronidase
Q1646:What is the DEFICIENT ENZYME in Hunter's
syndrome?
Iduronate sulfatase
Q1647:What is the ACCUMULATED SUBSTRATE in
Fabry's disease?
Ceramide trihexoside
Q1648:What is the ACCUMULATED SUBSTRATE in
Gaucher's disease?
glucocerebroside
Q1649:What is the ACCUMULATED SUBSTRATE in
Niemann-Pick disease?
Sphingomyelin
Q1650:What is the ACCUMULATED SUBSTRATE in
Tay-Sachs disease?
GM2 ganglioside
Q1651:What is the ACCUMULATED SUBSTRATE in
Krabbe's disease?
Galactocerebroside
Q1652:What is the ACCUMULATED SUBSTRATE in
Metachromatic Leukodystrophy?
Cerebroside sulfate
Q1653:What is the ACCUMULATED SUBSTRATE in
Hurler's syndrome?
1. Heparan sulfate;2. Dermatan sulfate
Q1654:What is the ACCUMULATED SUBSTRATE in
Hunter's syndrome?
1. Heparan sulfate;2. Dermatan sulfate
Q1655:What is the INHERITANCE of Fabry's disease?
XLR
Q1656:What is the INHERITANCE of Gaucher's disease?
AR
Q1657:What is the INHERITANCE of Niemann-Pick
disease?
AR
Q1658:What is the INHERITANCE of Tay Sach's disease?
AR
Q1659:What is the INHERITANCE of Krabbe's disease?
AR
Q1660:What is the INHERITANCE of Metachromatic
Leukodystrophy disease?
AR
Q1661:What is the INHERITANCE of Hurler's syndrome?
AR
Q1662:What is the INHERITANCE of Hunter's syndrome?
XLR;;"HUNTERS aim for the X";(XLR)
Q1663:What lysosomal storage dz has renal failure?
Fabry
Q1664:What lysosomal dz has optic atrophy; spasticity and
early death?
Krabbe
Q1665:Lysosomal Dz that is compatible with a normal life
usually?
Gaucher's Dz
Q1666:Lysosomal Dz w/ increase in sphingomyelin and
cholesterol in reticuloendothelial and parenchymal cells?
Niemann-Pick Dz
Q1667:What lysosomal Dz has cherry-red spot on macula?
Tay-Sachs Dz;Take them in the Sack (the cherries)
Q1668:What lysosomal Dz has accumulation of sulfatide in
brain; kidney; liver and peripheral nerves?
Metachromatic Leukodystrophy
Q1669:What enzyme is deficient in Fabry's Dz?
alfa-galactosidase
Q1670:What accumulates in Fabry's Dz?
ceramide trihexoside
Q1671:What mode of inheritane is Fabry's Dz?
X-linked
Q1672:What enzyme is deficient in Krabbe's?
beta-galactosidase
Q1673:What accumulates in Krabbe?
beta-galactocerebroside
Q1674:What enzyme is deficient in Krabbe?
beta-galatosidase
Q1675:What enzyme is deficient in Gaucher's Dz?
beta-Glucocerebrosidase
Q1676:What accumulates in Gaucher's?
glucocerebroside
Q1677:What enzyme is deficient in Niemann-Pick?
Sphyngomyelinase
Q1678:What accumulates in Neimann-Pick's?
sphingomyelin and cholesterol
Q1679:What enzyme is deficient in Tay-Sachs?
Hexosaminidase A
Q1680:WHat accumulates in Tay-Sach's Dz?
GM2 ganglioside
Q1681:What enzyme is deficient in Metachromatic
Leukodystrophy?
ArylSulfatase A
Q1682:What accumulates in Metachromatic
Leukodystrophy?
Sulfatide
Q1683:What enzyme is deficient in Hurler's Sx?
alpha-L-iduronidase
Q1684:What accumulates in Hurler's Sx?
Increase in dermatan sulfate
Q1685:What enzyme is deficient in Hunter's?
iduronate sulfatase
Q1686:What accumulates in Hunter's?
Heparan Sulfate
Q1687:What are the two Lysosomal Storage Dz that are X-
linked?
Fabry's X and Hunter's X
Q1688:What lysosomal Dz has corneal clouding and mental
retardation?
Hurler's Sx
Q1689:What lysosomal Dz has mild mental retardation?
Hunter's Sx;the hunter needs to see what he is shooting; So
NO corneal clouding;
Q1690:What lysosomal Dz has gargoyle facies?
Hurler's Dz
Q1691:What lysosomal dz has flaring of the distal femur?
(Like Erlehnmeyer Flask)
Gaucher's Dz
Q1692:What two lysosomal Dz are associated with Jews?
Tay-Sach's and Gaucher's Dz
Q1693:Hormones dived into?
Water soluble;Lipid Soluble
Q1694:Water solubles have receptor where?
Membrane Receptor
Q1695:WHere is the receptor in lipid soluble hormones?
Inside the cell
Q1696:What hormones require phosphorylation?
Water Soluble
Q1697:How is gene expression controlled in Water Soluble?
cAMP response element binding (CREB) protein
Q1698:What proteins are used in Water Soluble Hormones?
Leucine Zipper
Q1699:What protein is used for Lipid Soluble Hormones?
Zinc Finger Protein
Q1700:What are the water soluble hormones?
Insulin ;Glucagon;Catecholamines (NE; EPi)
Q1701:What are examples of lipid soluble?
Steroids;Calcitriol ---> Vit D;Thyroxines (thinks is
steroid);Retinoic Acid ----> Vit A.
Q1702:What do you think of with watersoluble hormones?
Male;Receptor Outside (penis);Zipper (Leucine Zipper)
Q1703:WHat do you think of with Lipid Soluble hormones?
Female;Receptor Inside;Zinc Finger Protein
Q1704:What receptors use Glucagon and Epinephrine?
cAMP pathway
Q1705:What are all the messenger involved in cAMP?
Gs Adenylate Cyclase ----> Protein Kinase ;Gi alpha2 beta2
2MAD
Q1706:What are pathway is used in Vasopressin and
Epinephrine (alpha 1)?
PIP2;PIMP
Q1707:Gq involves?
HAMMV (hummer);Gq magazine;Think C or
K;Phospholipase C;Protein kinase C;DAG; IP3; Ca+
Q1708:What pathway does Atrial Natriuretic Factor (ANF)
and Nitric Oxide (NO) use?
cGMP
Q1709:What pathway does Insulin use?
INsulin; growth factors ;via Tyrosine Kinase
Q1710:What do you see in the PIMP system? (PIP2)
Gq magazine;Cicis; C C C;Phospholipase C;Protein Kinase
C;Ca+
Q1711:Who activates the Ca+ release in the Endoplasmic
Reticulum E.R.?
IP3 activates Ca+ release
Q1712:What does Ca+ activates what in the PIP2 system?
Protein Kinase C
Q1713:What membrane enzyme is used in the ANF or NO?
Guanylate Cyclase
Q1714:If you want to;
Use Guanylate Cyclase
Q1715:Guanylate cyclase activates what?
cGMP ;G is nice to get some;
Q1716:cGMP activates what?
Protein Kinase G
Q1717:What does Protein Kinase G do?
Relaxes Smooth Muscle;;Relax and ENjoy the RIDE!
Q1718:Where is nitrous oxide found?
Heme membrane
Q1719:HOw does insulin activate hormone receptors?
Via Tyrosine Kinase
Q1720:WHat do you find in the membrane for Insulin?
Two beta subunits;cross membrane;;2 membrane helix
span;unlike ANF that has 1 membrane helix span
Q1721:What is unique about the cAMP and PIP2 system?
THey both have a 7 membrane helix span receptor
Q1722:What enzyme of the hormone receptor has only 1
membrane span?
ANF;guanylate cyclase
Q1723:What hormone receptor has 2 membrane helix span?
INsulin;Tyrosine Kinase
Q1724:Where does Nitrous Oxide (NO) come from? a.a.?
Arginine
Q1725:What are some drugs that increase NO?
nitroprusside;Nitroglycerine;Isosorbide dinitrate;Viagra
(Sildenafil);Agina
Q1726:Where is glut 4 found?
Adipose;Muscle;Not Liver Glut-1
Q1727:What happens if there is a mutation that increases G
protein?
Oncogenic;activation of ras (p21 monomeric);gsp (G2 alpha)
Q1728:What is involved in the p21ras oncogene?
Colon;Lung;Breast;Bladder ;ALL TUMORS!!!;liked to
Tyrosine Kinase;G protein!!!
Q1729:WHat is the mechanism for glucocorticods to cause
DM?
They increase PEPCK activity via response elements causing
increase gluconeogenesis;increase in glucose---> DM;via Zinc
Finger Proteins
Q1730:Which enzyme does Insulin activate in glucose related
metabolism?
Glycogen Synthase;glucose is stored;glycogen is made;you are
in a well-fed state;just ate
Q1731:Which enzyme is activated when glucagon is present?
Glycogen Phosphorylase is activated;degrages glycogen --->
glucose;increase the release of glucose;you are starving
Q1732:TCA cycle intermediates
Can I Keep Selling Sex For Money;
Officer?;Citrate;Isocitrate;alpha-Ketoglutarate;Succinyl-
CoA;Succinate;Fumarate;Malate;Oxaloacetate
Q1733:Regulated glycolytic enzymes
Hexokinase (-G6P);Glucokinase (+insulin);PFK1 (-citrate -
ATP +AMP +F-2;6-BP);Pyruvate kinase (-Ala; -ATP; +F-
1;6-BP);Pyruvate dehydrogenase (-ATP; -NADH; -Acetyl-
CoA)
Q1734:Regulated TCA cycle steps
Citrate synthetase (-ATP);Isocitrate dehydrogenase (+ADP; -
ATP; -NADH);Alpha-KG dehydrogenase (-NADH; -ATP; -
succinyl CoA)
Q1735:Lesch-Nyhan syndrome
HGPRT deficiency; can'd do purine salvage pathway; get uric
aciduria. X-linked
Q1736:I cell disease
Lack of mannose-6-phosphate transfer enzyme in golgi
network means can't tag lysosomal enzymes for traffic to
lysosome. Get secreted instead->coarse facies; early death
Q1737:Energy from TCA cycle per acetyl CoA
3 NADH --> 9 ATP;1 FADH2 --> 2 ATP;1 GTP --> 1
ATP;12 ATP/cycle via oxidative phosphorylation
Q1738:Galactosemia
Mild: Galactokinase deficiency->galactitol->childhood
cataracts;Severe: Gal-1P uridyl transferase deficiency-> very
high galactitol->liver damage; galactosemia; galacturia;
cataracts; mental retardation ;Tx both w/glucose & lactose free
diet
Q1739:Fructosuria
Fructokinase deficiency: benign fructosuria;Fructose
intolerance: Lack of aldolase B to convert F1P to DHAP and
glyceraldehyde->vomiting w/fructose load; mental retardation;
etc.
Q1740:Cofactors for PDH and a-KGDH
Vitamin B1->thiamine->TPP;Vitamin B2->riboflavin-
>FAD+;Vitamin B3->niacin->NAD+;Vitamin B5-
>pentothenate->CoA;Lipoic acid
Q1741:Cori cycle
1) Anaerobic glycolysis in muscle to pyruvate;2)Pyruvate ->
lactate to regenerate NAD+;3) Lactate to liver via blood;4)
Lacate converted back to pyruvate and then to glucose by
gluconeogenesis (6 ATP);5) Glucose sent back to muscle in
blood
Q1742:Energy yield of anaerobic glycolysis
2 ATP;Reducing equivalents in NADH used to convert
pyruvate to lactate via LDH to regenerate NAD+ to keep
running glycolysis
Q1743:Hexokinase vs glucokinase
Hexokinase: all cells; inhib'd by G6P;Glucokinase: liver & islet
cells; stim'd by insulin with lower Km but higher Vmax-
>glucose storage and
Q1744:SAM
S-adenosyl methionine;ATP+Met-->SAM->-
>Homocysteine;Need B12 & folate to regenerate methionine
from ATP;Regeneration of methionine is how B12 converts
dietary folate into form usable by purine synth and
thymidylate synthase
Q1745:GPCRs that signal via Gs
Gs stims cAMP synth;B1->inotrope/chronotrope;B2->SMC
relaxation;H2->stomach;V2->aquaporin insertion in
kidney;D1
Q1746:GPCRs that signal via Gq
Gq > PLC > DAG+IP3 > PKC & Ca2+;H1->allergy;a1-
>vasoconstrict;V1->vasoconstrict;M1;M3
Q1747:GPCRs that signal via Gi
Gi inhibs cAMP synth;M2;a2;D2
Q1748:Collagen types
Type 1: classic (bone; skin);Type 2: cartilage/joints;
hyaline;Type 3: Reticulin (skin etc); granulation tissue;Type
4: basement membranes
Q1749:Electron transport chain
NADH > e- > ;Complex I > H+ ;CoQ ;FADH2 > e- >
Complex II >CoQ;complex III > H+;Cyt C;Complex IV > H+
+ O2;H+ > ATP synthase > ATP
Q1750:Oligomycin
Inhibs ATP synthase > can't dissipate H+ gradient > ETC
machinery gets backed up and stopped > ROS
Q1751:2;4-dinitrophenol
Allows H+ to leak out of mitochondrial matrix > uncouples
electron transport from ATP synthesis gradient
Q1752:Rotenone
Inhibs e- transport > stops ETC > reduces proton gradient
Q1753:Cyanide
Inhibs e- transport > stops ETC > reduces proton gradient
Q1754:Irreversible gluconeogenesis enzymes
Pyruvate carboxylase (mitochondria;PEP
carboxykinase;Fructose-2;6-bisphosphatase;Glucose-6-
phosphatase (liver only)
Q1755:von Gierke's disease
Type I glycogen storage disease;Glucose-6-phosphatase
deficiency;Liver can't export glucose;-Glycogen accum-
>hepatomegaly;-Hypoglycemia;-Lactic acidosis
Q1756:Essential amino acids
PVT TIM
HALL;Phenylalanine;Valine;Tryptophan;Threonine;Isoleucin
e;Methionine;Histidine;Arginine;Leucine;Lysine
Q1757:PVT TIM HALL
Phenylalanine;Valine;Tryptophan;Threonine;Isoleucine;Methi
onine;Histidine;Arginine;Lysine;Leucine
Q1758:Purely ketogenic amino acids
Leucine & lysine only. Both are also essential
Q1759:Purely glucogenic essential AA:
V-MATH;Valine; methionine; arginine; threonine; histidine
Q1760:Urea cycle
Ordinarily; Careless Crappers Are Also Frivolous About
Urination;Ornithine;Carbamoyl
phosphate;Citruline;Asparatate;Arginosuccinate;Fumarate;Ar
ginine;Urea
Q1761:Black urine
Alkaptonuria: can't break down homogentisic acid; a
metabolite of tyrosine
Q1762:Musty odor; pale skin; mental retardation
Phenylketonuria: can't convert phenylalanine (musty and
retarded) to tyrosine (pale)
Q1763:Cystinuria frequency
4.902777778
Q1764:Cystinuria defect
COLA;Defect of AA transporter responsible for resorption
of Cysteine; Ornithine; Lysine & Arginine from proximal
tubule
Q1765:Components of sucrose
Fructose + glucose
Q1766:Components of lactose
GaLactose + glucose
Q1767:Function of ApoA1
Cofactor for LCAT
Q1768:Function of ApoB
Binds LDLR
Q1769:Function of ApoCII
Cofactor for lipoprotein lipase
Q1770:Function of ApoE
Cofactor for lipoprotein binding to receptor for uptake
Q1771:Cherry red spot
Tay-Sachs disease; deficiency of hexosaminidase; so can't
degrade GM2 ganglioside;Also Nieman Pick disease;
deficiency of sphingomyelinase
Q1772:fat soluble
ADEK; absorption dependent on gut (ileum) and
pancreas;toxicity more common because they accumulate in
fat;malabsorption can cause def
Q1773:water soluble
B1; B2; B3; B5; B6; B12; C; biotin; folate;all wash out easily
from body except B12 which is stored in liver
Q1774:A def
night blindness; dry skin
Q1775:A function; exceess
constituent of visual pigment; arthralgias; fatigue; headahce;
skin change; sore throat; alopecia;found in leafy veggies
Q1776:B1 (thiamine) def
Beriberi and WK syndrome; seen in alcoholism and
malnutrition;dry: polyneuritis;wet: high output CF
Q1777:B1 function
cofactor for oxidative decarboxy of a-ketoacids; cofactor for
transketolase in HMP shunt
Q1778:B2 (riboflavin) def
angular stomatitis; cheilosis; corneal vascularization
Q1779:B2 function
cofactor in ox-red;FMN; FAD
Q1780:B3 def
pellagra can be caused by Hartnup disease (decreased tryp
absorption); malignant carcinoid syndrome and INH;sxs:
diarrhea; dermatitis; dementia
Q1781:B3 function
constituent of NAD; NADP;derived from tryp using B6
Q1782:B6 (pyridoxine) def
convulsions; hyperirritability (def induced by INH and OCP);
peripheral neuropathy
Q1783:B5 function
coverted to pyridoxal phosphate - transaminatiors (ALT;
AST); decarbox; heme synthesis
Q1784:B12 (cobalamin) def
macrocytic; megaloblastic anemia; neuro sxs (optic
neuropathy; subacute combined degeneration; parasthesia);
glossitis;def caused by: malabsorption; lack of IF; or absence
of terminal ileum;Schilling test to detect def;abnormal myelin
seen
Q1785:B12 function
cofactor for homocysteine methylation (transfers CH3
groups);stored in liver;very large reserve;synthesized by
microorganisms
Q1786:Folic acid def
most common vitamin def; macrocytic; megaloblastic
anemia;no neuro sxs
Q1787:folic acid function
coenzyme (tetrahydrofolate) for 1 carbon transfer; involved in
Me reactions;important for synthesis of nitrogenous bases in
DNA and RNA
Q1788:Biotin def
dermitits; enteritis; caused by antiobiotic use; ingestion of raw
eggs
Q1789:Biotin function
cofactor for caboxylations;pyruvate-->OAA;ACOA--
>MCoA;PCOA-->MMCoA
Q1790:Vit C def
scurvy- swollen gums; bruising; anemia; poor wound healing
Q1791:Vit C funciton
necessary for hydroxylation of proline and lysine in collagen
synthesis;facilitates iron absorption by keeping iron in Fe2
reduced state;necessary cofactor for DA-->NE
Q1792:Vit D def
rickets in children (bending bones); osteomalacia in adults
(soft bones); hypocalcemic tetany
Q1793:function vit D
increase intestinal absorption of Ca and P
Q1794:vit D excess
hypercalcemia; loss of appetite; stupor;seen in sarcoidosis-
disease where epithelial macrophages convert vit D into active
form
Q1795:storage form of vitamin D
25-OH D3
Q1796:active form of vit D
1; 25 (OH)2 D3
Q1797:vit D from milk
ergocalciferol; consumed in milk D2
Q1798:vit D from sun skin
cholecalciferol D3
Q1799:Vit E def
increased fragility of erythrocytes; neurodysfunction
Q1800:vit E function
antioxidant (protects erythrocytes from hemolysis)
Q1801:vit K def
neonatal hemorrhage with increased PT and PTT but normal
bleeding time because neonates have sterile intestines and are
unable to synthesize vit K
Q1802:vit K function
catalyzes gamma carboxylation of glutamic acid residues on
various proteins concerned with blood clotting;synthesized
by intestinal flora
Q1803:K dependent clotting factors
II; VII; IX; X;protein C and S;warfarin is vitamin K antagonist
Q1804:Zinc deficiency
delayed wound healing; hypogonadism; decreased adult hair;
may predispose to alcoholic cirrhosis
Q1805:Clinical characteristics of WK syndrome
ocular distrubances; nystagmus;gait ataxia;mental dysfunction
(confusion; apathy; listlessness; disorientation);Korsakoff
psychosis- retrograde recall; inability ot acquire new info
Q1806:ntureint def assoc with cheilosis; glossitis; stomatitis
iron; riboflavin; niacin; folate; B12
Q1807:how does niacin help tx type IIb
hyperlipoproteinemia
inhibits lipolysis in adipose-->less circulating free fatty acids
--> less fatty acids to liver --> less VLDL --> less LDL
Q1808:INH leads to deficiency in
B6 and B3
Q1809:folic acid is involved in synthesis of;
purines (A and G) and thymine
Q1810:where is B12 absorbed
distal ileum;Crohns and sprue can cause absorption problems
Q1811:what organs help absorb B12
salivary glands; stomach; pancreas; distal ileum
Q1812:antioxidant vitamins
C; E and A
Q1813:how does vit D work at the cell
interacts with target cell DNA to selectively stimulate or
repress gene stimulation
Q1814:first vit D hydroxylation
25; in liver
Q1815:second vit D hydroxylation
1; in kidney
Q1816:What drug block DNA Topoisomerase II? Prok?
Eukar?
Prok: nalidixic acid/quinolones;Eukaryotes: etoposide ;and
teniposide
Q1817:Which cells contain telomerase? What are they linked
with?
embryonic; germ cells; stem cells except somatic cells;-
cancer/malignant cells have a high level of telomerase;They are
linked with apoptosis
Q1818:What is another name for topoisomerase II in PROK?
DNA gyrase
Q1819:What nucleic acid has the most methyl groups?
Cytosine
Q1820:What happens when you take a methyl out of
Cytosine?
It becomes demethylated to Uracil
Q1821:During what cell cycle does DNA repair occur?
G1 Phase
Q1822:When does mismatch repair occur? (phase)
G2 phase
Q1823:What does p53 gene encode for?
Protein that prevents a cell w/ damaged DNA from entering
the S phase
Q1824:What disease is associated with p53 gene?
Li Fraumeni Syndrome and many solid tumors
Q1825:What is ATM gene?
ATM encodes for a kinase needed for p53 to work
Q1826:What is ATM gene associated with?
ataxia telangiectasia
Q1827:What is ataxia telangiectasia?
hypersensitivity to X-rays;predisposition to lymphomas
Q1828:What is BRCA1 associated with?
Breast; Prostate and Ovarian Cancer
Q1829:What is BRCA 2 associated with?
Breast cancer
Q1830:What are BRCA1 and 2 associated with?
required for p53 activity
Q1831:What happens when UV light damages DNA? What
disease is prone to this damage? Why?
it crease thyamine dimers;- patients with Xeroderma
Pigementosa;- they lack excision endonuclease
Q1832:What does Xeroderma Pig. consists of?
- Extreme UV sensitivity;- excessive freckling;- multiple skin
cancers;- corneal ulcerations
Q1833:What are two diseases that are associated with DNA
repair?
Xeroderma and Hereditary Nonpolyposis Colorectal Cancer
(HNCC)
Q1834:What drug inhibits DNA dependent RNA
polymerase?
Rifampin
Q1835:What drug binds to DNA preventing its transcription?
Actinomycin D
Q1836:What drug inhibits RNA polymerase II?
amanitin (from mushrooms)
Q1837:Which RNA do RNA Pol 1;2 and 3 code for?
1 2 and 3 rhyme with R M T;respectively;1 rRNA 2 mRNA
3 tRNA
Q1838:What is similar to sigma factor in Eukaryotes?
TFIID; transcription factors II;they bind before RNA Pol;
just like Sigma factors
Q1839:How does RNA pol know where to start?
- sigma factor needs to find promoter region;- two consensus
sequences are recognized as TATA BOX
Q1840:How long does sigma stay bound to DNA?
As soon as transcription begins; sigma is released
Q1841:How does mRNA know when to stop trasncription?
Rho-independent termination occurs when newly formed
RNA folds on itself to form GC-rich hairpin loop
Q1842:How does Rho-dependet termination work?
Rho displaces RNA pol from the 3' end of the RNA once it
has paused at the termination site
Q1843:What binds to Shine-Dalgarno sequence?
Ribosomes
Q1844:Where are Shine-Dalgarno sequences located?
5' end
Q1845:What is unique about prokaryotic transcription and
translation?
They can both start at the same time with the help of Shine-
Dalgarno sequences which allow ribosomes to hook on and
start the translation before transcription is done
Q1846:What is Shine-Dalgarno?
Shine-Dalgarno sequences lets prokaryotes shine! They can
do 2 things at the same time! Transcribe and TRANSLATE!
Q1847:What are the three STOP codons?
UAG;UAA;UGA;U Are Gone;U Are Away;U Go Away
Q1848:What is the poly-A tail added for?
1) protect from rapid degradation;2) transport to cytoplasm
Q1849:What is added at the 5' end of the transcribed mRNA?
It is actually hnRNA and a methylguanosine cap Me-Gppp is
added to the 5' end
Q1850:What is the function of the methyl guanosine cap?
It helps protect the mRNA chain from degradation
Q1851:Where is the poly A tail added?
3' end
Q1852:What can you say about the length of the poly A tail?
The longer the more stable the mRNA (hnRNA)
Q1853:What is another name for spliceosome?
snRNP; SNURP
Q1854:What is the function of spliceosomes?
They excise introns and leave only exons to be expressed
Q1855:What disease has a problem in spliceosomes/snRNP?
B-thatlassemia since mutations interfere with the splicing of
Beta-Globin mRNA
Q1856:How are the introns degraded?
They are degraded in a lariat structure and excised by
spliceosomes
Q1857:How can you calculate how many introns you have?
I=E-1;If you have 4 exons;Then you have I=4-1;I=3; 3 introns
Q1858:Where is the activated amino acid in a tRNA?
at the 3' end
Q1859:How does tRNA accomplish its lariat shape
(loop/cloverleaf)?
it has weird bases like;D;T;Pseudouridine
Q1860:Where is the anticodon found in the tRNA?
in the middle of the loop;center loop in between 5' and 3' ends
Q1861:How do you know a protein is marked for
destruction?
It has been ubiquiniated by ubiquitin;Usually because of
misfolding
Q1862:Who translates proteins for cytoplasm and
mitochondria?
free cytoplasmic ribosomes
Q1863:Who transtalates proteins for secreted proteins;
membrane proteins; and lysosomas enzymes?
Rough E.R.
Q1864:How can you make a protein to be delivered to the
R.E.R.?
N-terminal hydrophobic signal sequence has to be added to be
secreted or placed in the membranes
Q1865:How do you direct a prtoein to go inside a lysosome?
It is phosphorylated with a mannose residue in the R.E.R;-
usually this protein is an enzyme to be delivered to the
lysosome
Q1866:What happens to misfolded proteins?
They are mark with ubiquitin to be destroyed by
proteosomes;you will be liquidated!!!;Ubiquinated!!!!
Q1867:What are proteosomes?
They are large cytoplasmic complexes that digest damaged
proteins
Q1868:What enzyme is deficient in Fabry's Dz?
alpha-galactosidase A
Q1869:What enzyme is deficient in Krabbe's Dz?
BB for beta-galactosidase
Q1870:What enzyme is deficient in Gaucher's Dz?
beta-glucocerebrosidase;(It is in the center of the reactions)
Q1871:What enzyme is deficient in Niemann-Pick's Dz?
Sphingomyelinase
Q1872:What enzyme is deficient in Metachromic
Leukodystrophy?
Arylsulfatase A
Q1873:What enzyme is deficient in Tay Sachs Dz?
Hexosaminidase A
Q1874:What accumulates from Fabry's Dz?
ceramide trihexoside
Q1875:What is the finding in Fabry's Dz?
renal failure
Q1876:What accumlulates in Krabbe's Dz?
galactocerebroside in the brain
Q1877:What is the finding in Krabbe's Dz?
Optic atrophy;spasticity;early death;The krabbe got your
eyes!
Q1878:What accumlulates in Gaucher's Dz?
glucocerebroside;- brain;- liver;- spleen;- bone marrow;G is for
Glucocerebrosidase
Q1879:What accumlulates in Neimann Pick's Dz?
sphingomyelin and cholesterol;No man Picks his nose with
hiSPHINGER
Q1880:Wnat are the findings in Neimann Pick Dz?
increase cholesterol and sphyhingomyelin in
reticuloendothelial and parenchymal cells;- Patients die by age
3
Q1881:What accumlulates in Tay Sachs Dz?
GM2 ganglioside 2
Q1882:What are the findings in Tay Sachs Dz?
Cherry-red spot on macula;1:30 carrier in European
Jews;Death by age 3;Got a Sach of Cherries in your Macula
Q1883:What accumlulates in Metachromatic
Leukodystrophy?
sulfatide in;- brain;- kidney;- liver;- peripheral nerves
Q1884:Which lysosomal storage diseases (of the
sphingolipidoses) are autonomal recessive?
All except Fabry's!!!
Q1885:What lysosomal storage disease are x-linked?
sphingolipidosis: Fabry's;mucopolysaccharidoses:
Hunter's;Hunter's hit the X
Q1886:What mucopolysacharidose Dz has no corneal
clouding?
Hunter's;They need to see what they hunt!
Q1887:What enzyme is deficient in Hurler's Dz?
alpha-L-iDURONidase
Q1888:What enzyme is deficient in Hunter's Dz?
iDURONate sulfatase
Q1889:What increases in Hurler's Sx?
heparan and dermatan sulphate;mucopolysaccharides
Q1890:What are the signs of Hurler's Sx?
Halted growth ;Progressive mental retardation ;Thick; coarse
facial features with low nasal bridge ;Cloudy corneas
;Deafness ;Joint disease; including stiffness ;Heart value
problems ;Abnormal bones of spine and claw hand
Q1891:How do we screen for Hurler's Sx?
Urine Heparan and Dermatan sulfate
Q1892:What is a term associated with Hurler's Sx?
Gargoylism since there are facial deformities
Q1893:What signs and symtoms are associated with Hunter's
Sx?
protuberant abdomen; claw hands; excessive hair growth;
coarsening of the face with grotesque facial features; retarded
growth; and behaviour problems.
Q1894:Which syndrom is severe? Hurler or Hunter?
Hurler's Syndrome ;It is termed MPS I;Hunter is MPS II
Q1895:What amino acid is unique to collagen?
Hydroxyproline
Q1896:Where does glycosylation occur?
E.R. and Golgi apparatus
Q1897:Which enzymes are requiered to make collagen?
proline and lysine hydroxylases
Q1898:What vitamin is needed to make collagen?
Vitamin C;- Hydroxylates Proline and Lysine in the RER
Q1899:What are some co-factors of lysyl oxidase?
O2 and Copper (Cu)
Q1900:What Dz results from deficiency of Lysyl Oxidase and
why?
Deficient Copper (Cu2+);Menke's Dz is a genetic deffect that
decrease collagen synthesis
Q1901:What enzyme is deficient in Ehler's Danlos?
Lysine Hydroxylase
Q1902:What are the signs and symptoms of Menke's Dz?
Depigmented (steely) hair;Arterial tortuosity;
rupture;Cerebral degeneration;Osteoporosis
Q1903:What collagen is affected in Osteogenesis Imperfect?
Type I for bONE
Q1904:What do you see in patients with Osteogenesis
Imperfecta?
skeletal deformities;fractures;blue sclera
Q1905:What other disease is involved in Copper usage?
Wilson's Dz but it is a Copper (Cu2+) toxicity
Q1906:What are some symptoms of Wilson's Dz?
Liver Cirrhosis;Cu damages nerves and causes Brown Kaisser-
Fleischner Rings
Q1907:What inhibits eEF-2? Elongation factor 2 in
Eurkaryotes
Diphtheria and Pseudomonas Toxins
Q1908:What inhibits protein translation in Eukaryotes?
Diphteria and Pseudomonas
Q1909:Where do Diphtheria and Pseudomonas act?
eEF-2 is inhibited
Q1910:How many ATPs high energy bonds are needed to
translate an amino acid?
4 Total for each amino acid;breakdown;2 ATP for charging;1
GTP for initiation;1 GTP for Elongation
Q1911:What is the antibiotic of choice for pertussis?
Erythromycin; blocks transLOcation;macrOLide
Q1912:What results in Menkes Dz?
- Fragile bones;- Fragile blood vessels;from poorly crosslinked
connective tissue
Q1913:What blocks ADP ribosylation of EF-2?
Diphtheria and Pseudomonas
Q1914:What is an operon?
group of proteins required for a particular metabolic function
Q1915:Where is the regulatory region in Prokaryotes?
Upstream on the 5' end
Q1916:What kind of mRNA does the operon produce?
Polycistronic mRNA
Q1917:What two ways of transcriptional control exist in
prokaryotes?
regulation of activator and repressor proteins;Attenuation
Q1918:Where do we find Attenuation?
Histidine Operon
Q1919:What model do we use for activator and repressor
proteins?
Lac Operon
Q1920:What two regulatory proteins exist in the Lac Operon
Control?
lac repressor protein;c-AMP-dependent activator protien
(CAP)
Q1921:What does the lac operon sense?
glucose is preferred but in the absence lactose is taken as
energy
Q1922:What regulates the CAP?
cAMP levels;if glucose is low; cAMP increases and activates
it
Q1923:What happens to the lactose operon if glucose is
present?
it is shutdown;glucose decreases cAMP;so CAP doesn't bind
to CAP site
Q1924:When does CAP bind to CAP site?
when glucose is low since cAMP is high
Q1925:When is the repressor protein made?
Always since it is embedded in the mRNA sequence
Q1926:What does lactose do to the lac operon?
lactose induces gene expression since it prevents the repressor
protein from binding to the operator sequence
Q1927:If lactose is high and glucose is low what happens?
1) lactose binds to repressor and stimulates gene
expresssion;2) cAMP is high so it binds to CAP protein and
Q1928:When does the lactose operon stop sequence?
when the repressor protein is bound to the operator
Q1929:When is high expression of the lac operon found?
High lactose and no glucose
Q1930:When glucose is present does cAMP go up or down?
they are inversely proportional;Glucose high cAMP
low;glucose low cAMP high
Q1931:When glucose is high; what happens to the repressor?
it remains active since CAP can't block it (cAMP is low)
Q1932:What compounds are formed when lactose is broken
down?
galactose and glucose
Q1933:What enzyme degrades lactose?
Beta-Galactosidase
Q1934:Where does RNA polymerase work on?
Promoter
Q1935:Lactose goes with
Repressor
Q1936:Repressor attaches to?
Operator
Q1937:Attenuation??? Which operon?
Histidine Operon
Q1938:What happens when histidine is absent?
enzymes are produced
Q1939:What other a.a. work similar to the Histidine Operon?
Tryptophan;Leucine;Phenylalanine
Q1940:What is attenuation?
premature termination of transcription
Q1941:What does attenuation in prokaryotes depdend on?
The fact that transcription and translation occur
simultaneously in prokaryotes
Q1942:What happens if histidine is present?
Transcription is terminated before RNA pol reaches operon
Q1943:Can attenuation occur in Eukaryotes?
No! Transcription and translation are two separate;
independent events
Q1944:What starts translation after leader peptide is made?
Shine-Dalgarno sequence
Q1945:What happens when histidine is low?
the ribosomes will stall and not form the stem and loop +
poly U that stops the ribosomes and they will continue to
transcribe the genes of the operon
Q1946:What are activator proteins called in Eukaryotes?
Response Elements
Q1947:Where are response elements located?
Some upstream in promoter region;Most in an enhancer region
outside of promoter even more upstream
Q1948:Where are upstream promoter elements located?
Just upstream of -25 sequence TATA Box
Q1949:What does the upstream promoter elements include?
CCAAT Box (-75) NF-1;GC-rich SP-1 (in between -25 and -
75)
Q1950:What are the characteristics of enhancers?
Contain activator proteins;- may be 1000 bp away from
gene;- upstream; downstream; within an intron;-they are
tissue specific
Q1951:What are repressor proteins in Eukaryotes called?
Silencers
Q1952:What are cis regulators?
DNA regulatory base sequences/binding sites for proteins
Q1953:What are trans regulators?
transcription factors
Q1954:What are the properties of a trans regulatory
property?
they can diffuse through the cell to their point of action.
Q1955:What protein class are steroid receptors?
Zinc Finger
Q1956:What protein class are cAMP response element
binding prtoeins? (CREBs)
Leucine Zipper
Q1957:Homeodomain proteins are what protein class and
what are they involved in?
Helix-turn-helix;Regulate gene expression during
development;- embryonal development
Q1958:What protein class are peroxisome proliferator-
activated receptors? (PPARs)
Zinc finger proteins
Q1959:What is the response element for 1) steroid
receptors?;for 2) cAMP?;for 3) peroxisome (PPARs)
1) HRE;2) CRE;3) PPREs
Q1960:Which response element is induced with the new tx for
insulin resistance?
PPARs;- thiazolidinediones
Q1961:What is a new drug that targets Peroxisime
proliferator-activated receptors? (PPARs)
Clofibrate;-affects lipid metabolism
Q1962:What happens when glucose is low?
Glucagon released
Q1963:What is the effect of glucagon on gene regulation?
increases cAMP
Q1964:What happens in time of stress?
Cortisol secreted
Q1965:What does cAMP do?
- activates Protein Kinase A;- CREB is activated via
phosphorylation
Q1966:CREB binds to what in the nucleus?
CREB enters the nucleus and binds CRE region in the
enhancer region
Q1967:What does the GRE and CRE region do?
They enhance or activate PEPCK gene
Q1968:Who activates GRE enhancer region?
cortisol (glucocorticoid response element)
Q1969:Who activates CRE enhancer region?
Active CREB (cAMP response element) which is activated
by cAMP
Q1970:What are two homeodomain protein regulator genes?
HOX and PAX genes;Homeobox and Paired-Box genes
Q1971:What disease is associated with PAX (paired-box)
genes?
Klein Waardenburg syndrome (WS-III);dystopia canthorum;
pigment abnormalities;congenital deafness;limb abnormalities
Q1972:What are some exceptions to codominat expression?
- Barr Body (inactive X chromosome) in women;- Ig heavy
and light chain loci;- T-cell receptor loci
Q1973:What happens when genes become acetylated?
The histones are acetylated and it increases gene expression
Q1974:How do genes become silenced? Give two diseases
that follows this;
Methylation of DNA silences genes;Prader-Willi and
Angelman Sx
Q1975:What chromosome is involved in defect of imprinting?
Chromosome 15
Q1976:What is the problem in Prader-Willi Sx?
Prader-Willi region is inherited from Paternal Origin (P for
P);so; if father has defective chromosome 15 then symptoms
will occur
Q1977:What are the symptoms of Prader-Willi Sx?
- Childhood obesity + hyperphagia;- Hypogonadotrophic
hypogonadism;- Mental Retardation;- Hypotonia
Q1978:How else can you get Prader-Willi Sx?
uniparental (maternal) disomy of chromosome 15
Q1979:When does upstream termination occur?
When histidine is present
Q1980:When does downstream termination occur?
when histidine is absent;* this is a normal termination
Q1981:What kindo of domain do HOX and PAX have?
helix-turn-helix domain
Q1982:What is the first step in increase activity of beta-
galactosidase activity?
increase in cAMP due to glucose depletion
Q1983:Why does beta-galactosidase activity decrease?
depletion of lactose;- dissociation of repressor protein;-
binding of repressor to operator control region
Q1984:Chp. 6
Recombinant DNA
Q1985:WHat does restriction sites provide?
Usually defense against DNA viruses
Q1986:How do palindromes get protected in bacterial DNA?
methylase enzyme modification
Q1987:How is infecting viral DNA recognized?
unmethylated palindromes are recognized by restriction
endonuclease
Q1988:What is a vector?
piece of DNA that is capable of autonomous replication in a
host cell
Q1989:What is recombinant DNA?
when a fragment is placed inside a vector
Q1990:What is a genomic DNA library?
colonies produced by plating the recombinant DNA with
antibiotic resistance and sensitivity
Q1991:What can restriction site polymorphisms be used for?
These enzymes cut DNA sequences and detect defects in
longer sequences or shorter sequences;Example: Sickle Cell
Mutation which results in ONE long 1.35 kb fragment instead
of a 1.15kb and a 0.2kb fragment (2 fragments is normal)
Q1992:What do cDNA lack?
introns
Q1993:What must cDNA contain?
complete coding sequence of a gene
Q1994:What is produced at the end of a cloning procedure?
An expression library
Q1995:What do you do after reverse transcriptase has created
the first strand of cDNA?
Treat DNA with NaOH to remove mRNA template
Q1996:What enzyme do you use to create cDNA?
reverse transcriptase
Q1997:How do you remove mRNA template strand in
making cDNA?
NaOH (sodium hydroxide)
Q1998:What must be inserted in order to produce proteins as
the end product of cloning?
- Bacterial Promoter;- Shine-Dalgarno Sequence
Q1999:What are 3 examples in which cDNA expression
libraries are being used?
1) Recombinant Human Insulin;2) Recombinant Factor VIII
(treating Hemophilia A);3) Recombinant HBsAg
(antigen(protein) is made and given to patients to immunize
them against hepatitis B without introducing the live virus)
Q2000:Does the gene therapy cure the patient and subsequent
generations?
NO! it cures only the patient since it is only introduced into
the affected organ and not into the reproductive tissues of the
afected individual
Q2001:What is a transgenic animal?
animal in which a new gene has been introduced into its
germline
Q2002:How is gene therapy different from Transgenic
Animals?
transgenic animals have virtually new gene in every cell;
including the gametophytes so that they get passed on to their
offspring and these are no longer affected by the defect
Q2003:What are genomic libraries used for?
studying DNA sequences that are not expressed;- response
elements;- introns;- promoters;Constucting restriction maps
of DNA (sickle cell);Id genetic markers (microsatellites)
Q2004:Chp. 7
Genetic Testing
Q2005:What are the Autosomal Dominant Dz characteristics?
- Only one mutant allele needed;- both sexes affected;- male to
male transmission
Q2006:What are the Autosomal Dominant Dz?
1) Familian Hypercholesterolemia (LDL receptor def.);2)
Huntington Dz;3) Neurofibromatosis I;4) Marfan Sx;5) Acute
Intermitent Porphyria
Q2007:What are the characteristics of autonsomal recessive?
- two mutant alleles are requiered;- born to unaffected
parents;- either sex;- male to male transmission
Q2008:What are some of the autosomal recessive dz?
* Sickle Cell Anemia;* Cystic Fibrosis;* Phenylketonuria;*
Tay-Sachs Dz (Hexosaminidase A def.)
Q2009:What are the traits of X-linked dominant?
- One mutant allele ;- either sex;- affected male passes on to all
daughters;- affected female passes trait to both fem and males
Q2010:What are 2 X-linked Dominant Dz?
- Hypophosphatemic Rickets;- Fragile X syndrome
Q2011:What are the traits of X-linked recessive dz?
- usually males are affected;- no male to male transmission
Q2012:What are some of the X-linked recessive dz?
1) Duchene Muscular Dystrophy;2) Lesch Nyhan Sx
(Hypoxanthine-guanine phosphoribosyltransferase
HGPRT)self mutilation;3) Glucose-6-Phosphate
Dehydrogenase def;4) Hemophilia A and B
Q2013:What is the trait of Mitochrondrial Inheritance?
- inherited maternally;- ALL offspring of affected female are
affected!
Q2014:What are the 3 diseases that arise from Mitochrondrial
Inheritance?
LHON; MELAS and MERRF;1) Leber Hereditary Optic
Neurophathy;2) Mitochondrial Encephalomyopathy; lactic
acidosis; stroke-like episodes;3) Myoclonic epilepsy with
ragged red muscle fibers
Q2015:What form of inheritance is Cystif Fibrosis?
autosomal recessive
Q2016:How do you inherit Hungtington Dz?
Autosomal Dominat
Q2017:What form of inheritance is sickle cell disease?
autosomal recessive
Q2018:What form of inheritance is Fragile X Sx?
X-linked Dominant
Q2019:How is Phenylketonuria inherited?
autosomal recessive
Q2020:How is Lesch-Nyhan Sx inherited?
X-linked recessive
Q2021:How is Neurofibromatosis I inherited?
Autosomal Dominant
Q2022:What is the mode of inheritance of Marfan Sx?
Autosomal Dominant
Q2023:How is Leber Hereditary Optic Neuropathy inherited?
Mitochrondrial Inheritance
Q2024:How is Duchenne Muscular Dystrophy inherited?
x-linked recessive
Q2025:How is myoclonic epilepsy inherited?
mitochrondrial inheritance
Q2026:How is Acute intermittent porphyria inherited?
Autosomal DOMINANT
Q2027:How is Rickets inherited?
X-linked Autosomal Dominant
Q2028:How is Cystic Fibrosis inherited?
autosomal recessive
Q2029:How is Phenylketonuria inherited?
autosomal recessive
Q2030:How is Familia Hypercholesterolemia inherited?
Autosomal Dominant;LDL receptor deficiency
Q2031:How is Tay-Sachs Disease inhertied?
autosomal recessive
Q2032:How is Lesch-Nyhan Sx inherited?
HGPRT def;X-linked recessive
Q2033:How is Hemophilia A and B inherited?
X-linked recessive
Q2034:How is Glucose-6-phosphatase inherited?
X-linked recessive
Q2035:How is cystic fibrosis inherited?
CFTR autosomal recessive
Q2036:Which RNA is identical to the coding strand?
the mRNA
Q2037:What is the template strand?
The strand that is compelementary and antiparallel to the
mRNA
Q2038:What amino acids (a.a.) are precursors of
catecholamines?
Phenylalanine and Tyrosine
Q2039:What does tryptophan form?
Serotonin and Niacin
Q2040:What a.a. is involved in depression?
Tryptophan--> makes Serotonin
Q2041:What a.a. are involved in maple syrup disease?
Isoleucine; Leucine and Valine;I Love Vermont maple syrup!!!
Q2042:What a.a. is a secondary amine?
Proline
Q2043:What does Proline do to the protein structure?
disrupts secondary structure
Q2044:What are the acidic a.a.?
aspartic acid and glutamic acid ;negatively charged coo-
Q2045:What a.a. are basic? 3
Histidine; Arginine; Lysine;BASE HAL;They are positively
charged NH+
Q2046:What a.a. is associated with the Golgi apparatus? 2
serine and threonine;O-linked glycosylation;Mannose-6-
phosphate;lysosomes
Q2047:What a.a. is associated w/ endoplasmic reticulum and
export of proteins?
Asparagine;N-linked glycosylation
Q2048:What are two a.a. that conatin sulfure?
Cysteine and Methionine
Q2049:What does cysteine do to the protein structure?
stabilize the shape of proteins (3ry structure)
Q2050:What two a.a. are linked with post-translational
modificacion?
serine; threonine and asparagine
Q2051:What a.a. is a methyl donor?
methionine ;S-adenosaylmethionine (SAM)
Q2052:What does tyrosine make?
Catecolamines;Thyroid T3/T4;Melanin
Q2053:What is the smallest a.a.?
glycine
Q2054:What a.a. makes tyrosine?
Phenylalanine
Q2055:What is made with tyrosine?
cathecholamiens;thyroid T3T4;melanin
Q2056:What a.a. is associated with Vitamin B3?
tryptophan is asociated with B3 (niacin) ;NAD
Q2057:What disease is also related to tryptophan deficiency
and pellagra?
Hartnup Dz;since decreases Niacin B3 and causes Pellagra
(dermatitis; diarrhea; demetnia)
Q2058:What a.a. contributes to the negative charge of
proteins?
aspartic acid coo-;glutamic acid
Q2059:What a.a. contributes to the positive charge of
proteins?
lysine and arginine
Q2060:What a.a. is abundant in RBC?
histidine since it brings the pH to 7.0
Q2061:What is the only a.a. that is useful in maintaining the
physiologic pH (7.2-7.4)?
Histidine pK at 7.0
Q2062:What are the essential amino acids?
PVT TIM HALL;Private tim hall
Q2063:What does PVT TIM HALL stand for?
Phenylalanine;Valine;Tryptophan;Threonine;Isoleucine;Methi
onine;Histidine;Arginine;Leucine;Lysine
Q2064:What charge is the protein if the pH is lower than the
pI?
positive ;- it is trying to compensate and neutralize it
(buffering it)
Q2065:What charge is the protein if the pH is higher than the
pI?
negative
Q2066:What are Cooperative Enzymes called?
Allosteric Enzymes
Q2067:WHat happens when Km increases?
the affinity is low
Q2068:What do enzymes do with chemical Rx?
decrease energy of activation
Q2069:What happens when there is a competitive inhibitor?
Km increases; Vmax stays the same;Thin Kompetitive
Increases
Q2070:What happens when noncompetitive inhibitor binds?
Km no effect; Vmax decreases
Q2071:What happens when an irreversible inhibitor binds?
Km no effect; Vmax decreases
Q2072:What are two examples of competitive inhibitors?
HMG-coA reductase;Methotrexate (inhibits folic acid
dihidrofolate reductase)
Q2073:What hormones affecte near-by organs?
paracrine
Q2074:What hormones go around the body through long
distances?
telecrine
Q2075:What are two examples of paracrine hormones?
prostaglandins and neurotransmitters
Q2076:What are two classes of telecrine hormones?
endocrine and GI hormones
Q2077:What are the two classes of hormones?
Hydrophocis and Hydrophilic
Q2078:Where is the receptor for water soluble hormone?
Lipid soluble?
Water- receptor on cell membrane;Lipid - inside the cell Zinc
Finger
Q2079:What happens to the hormone inside the cell?
Water - second messengers;Lipid - hormone receptor complex
binds to response elements (HRE in enhancer region)
Q2080:How are water soluble hormones controlling gene
expresion?
Through proteins like cAMP respones element binding
(CREB)
Q2081:Which process (water or lipid soluble) is faster?
water soluble
Q2082:What hormone group uses Leucine Zippers?
Water Soluble
Q2083:Give three examples of water soluble?
Insulin;Glucagon;Catecholamines
Q2084:What are 4 examples of lipid soluble hormones?
Steroids;Calcitriol;Thyroxines;Retinoic Acid
Q2085:What are the three second messengers for water
soluble hormones?
cAMP;PIP2 (DAG; IP3; Ca2+);cGMP
Q2086:What does cAMP control? protein/enzyme/kinase?
Gs protein; adenyl cyclase enzyme; protein kinase A
Q2087:What does PIP2 control? protein/enzyme/kinase?
Gq; phospholipase C; protein kinase C
Q2088:What does cGMP control? protein/enzyme/kinase?
none; guanyl cyclase; protien kniase G
Q2089:What are two examples of cAMP control?
glucagon;epinephrine (alpha2 and beta)
Q2090:What are two examples of PIP2?
vasopressin;epinephrine (alpah 1)
Q2091:What are two examples of cGMP?
Atrial Natriuretic Factor (ANF);Nitric Oxide (NO)
Q2092:What does insulin; growth factor control?
protein/enzyme/kinase?
monomeric p21ras; none; tyrosine kinase
Q2093:What are some examples of insulin and growth factors
control?
insulin;insulin-like growth factor (IGF);platelet-derived
growth factor (PDGF);Epidermal Growth Factor (EGF)
Q2094:What water soluble hormone system has a 7 helix-
span?
cAMP and PIP2 system
Q2095:Whioh system works inside the nucleus?
cAMP through CREB protein
Q2096:Which system works with the E.R.?
PIP2; releases Ca2+ from E.R.
Q2097:What system doesn't requiere G proteins?
cGMP for example Atrial Natriuretic Factor (ANF)
Q2098:What two protoncogenes are associated with G
proteins?
1) p21ras oncogene ;colon; lung; breast and bladder CA;2) gsp
oncogene;pituatary tumor; adenomas; endocrine ovarian
turmos
Q2099:What protein is stimulated in Cholera toxin?
Gs alpha stimulates increase in cAMP
Q2100:What is similar in ADP-ribosylation of Gs alpha?
Cholera toxin and E. coli toxin
Q2101:What bacteria inhibits Gi alpha?
Pertussis;Increase activity of adenyl cyclase
Q2102:What does p21 ras do?
stimulates monomeric G protein
Q2103:What is SH2 linked with?
Tyrosine Kinase
Q2104:What is sildenafil associated with?
inhibits cGMP phosphodiesterase (PDE) in vascular smooth
muscle
Q2105:What is the correct sequence in cGMP and sildenafil?
increase cGMP--> increase protein kinase--> vasodilation
Q2106:What is associated with growth factor?
tyrosine kinase
Q2107:Vitamins
Chp. 10
Q2108:What enzymes is biotin involved in?
All cabroxylases;pyruvate; acetyl coA; propionyl coA
carboxylase
Q2109:Which enzymes is vitamin B1 involved in?
Thiamine is B1 involved in ;pyruvate dehydrogenase;alpha-
ketoglutarate dehydrogenase;transketolase
Q2110:What pathways are involved with Thiamine?
B1 is involved in;PDH (pyruvate DHG);TCA cycle (alpha-
kg);HMP Shunt (transketolase)
Q2111:What are the symptoms of B1 deficiency?
Wernicke- ataxia; nystagmus; ophtalmoplegia;Korsakoff-
confabulation; psychosis;Wet Beri-beri cardiac failure lots of
ATP needed
Q2112:What is vitamin B3?
Niacin
Q2113:What is vitamin B3 involved with?
dehydrogenases
Q2114:What co-factors are made by B3?
NAD and NADP
Q2115:What disease comes about with B3 deficiency?
diarrhea; dementia; dermititis ;pellagra
Q2116:What is a.a. deficient in B3 deficiency?
tryptophan (in corn)
Q2117:What is folic acid involved in? (enzyme)
thymidylates synthase;purine synthesis enzymes
Q2118:What is the MCC of B1 deficiency?
Alcoholism
Q2119:What is the MCC of thiamine def.?
alcoholism and pregnancy
Q2120:How long is thiamine stored?
3 months
Q2121:What are the risks of folic acid deficiency?
homocystinemia;deep vein thrombosis and atherosclerosis
Q2122:What happens to fetus if there is folic acid def.?
neural tube defects
Q2123:What is vitamin B12 involved in? enzymes?
Homocysteine methyltransferase;Methymalonyl CoA mutase
Q2124:What pathways is B12 involved in?
methionine; SAM;odd-carbon FA;val; met; ile; thr
Q2125:What two vitamins cause megaloblastic anemia?
B12 and folic acid
Q2126:What is the MCC of B12 def?
pernicious anemia
Q2127:What are other causes of B12 def.?
aging; poor nutrition; bacterial overgrowth of terminal
ileum;resection of terminal ileum secondary to Crohn's
DZ;chronic pancreatitis;vegans;infection with D. Latum
Q2128:What is the difference between B12 and folic acid def.?
B12 has progressive peripheral neuropathy
Q2129:What enzymes are involved with B6 vitamin?
B6 is pyridoxine;Aminotransferases;AST (GOT);ALT
(GPT);Lamba-Aminolevulinate synthase
Q2130:What pathways is B6 involved in?
protein catabolism;heme synthesis
Q2131:What is the MCC of vit. B6 def.?
isoniazis therapy
Q2132:What do you find in B6 def.?
sideroblastic anemia;cheilosis and stomatitis;convulsions
Q2133:What is B2 vitamin?
RI BO flavin
Q2134:What cofactors are derived from B2?
FAD(H2)
Q2135:What enzymes are involved with B2?
dehydrogenases
Q2136:What findings w/ B2 def?
Corneal neovascularization;Cheilosis;Stomatitis;Magenta-
Colored Tongue
Q2137:What enzymes are involved with vitamin C?
prolyl hydroxylases;Lysyl hydroxylases;DOPAMINE
hydroxylase
Q2138:What pathways are involved with vitamin C?
collagen syntehsis;catecholamine synthesis;(absoprtion of
Iron from GI tract)
Q2139:What is the MCC of vit. C def?
diet deficient in fruit and green vegetables
Q2140:What vitamin is involved in carboxylation of glutamic
acid?
vitamin K
Q2141:What factors are involved in vitamin K?
2; 7; 9; 10 Protein C and S
Q2142:What is vitamin A involved in? Dz?
night blidness;follicular hyperkeratosis;xerophtalmia
Q2143:What is another name for vitamin A?
carotene;involved in retinoic acid and retinol ;ol oic;behave as
steroid hormones
Q2144:What part of vitamin A is involved in rod and cone cell
division?
Retinal;al
Q2145:What vitamin prevents oxidation of LDL particles?
vitamin E
Q2146:What is another name for vitamin E?
tocopherol
Q2147:What vitamin acts as steroid hormone uptake of
dietary Ca+ from gut?
Vitamin D
Q2148:What two disease cause by vitamin D def?
children: Rickets;Adults: Osteomalacia
Q2149:What is the physiologic response to hypocalcemia?
- increase PTH;- PTH binds to proximal tubules;- cAMP
activate 1-alpha-hydroxylase;- 1;25 DHCC acts on duodenal
epithelial cells;- Zinc finger proteins binds to response
elements (in enhancer region of DNA);- induce synthesis of
calcium binding proteins
Q2150:What vitamin is toxic in pregnancy?
Vitamin A (Acutane) used to treat ACNE
Q2151:What vitamin D does the skin produce?
cholecalciferol (vitamin D3)
Q2152:What happens to vitamin D in the liver?
25-hydroxylation in the liver
Q2153:What is needed in patients with renal dz; fanconi sx;
and genetic deficiency of 1-alpha-hydroxylase?
they all need to be supplemented with 1;25
DHCC;dihydrocolecalciferol;since renal 1-alpha-hydroxylase
is not working
Q2154:Patients with liver damage should be given what?
25-DHCC or 1;25 DHCC
Q2155:Liver provides what to vitamin D?
two things;1st cholesterol to skin to make 7-
dehyrocholesterol;2nd 25-hydroxylation
Q2156:What is the comercial name for a retinoic acid that is
teratogenic?
isotretinoin
Q2157:When does vitamin K act?
it is a co-translational modification;it acts during translation
Q2158:What causes vit. K deficiency? (drug)
1) phenylhydantoins during pregnancy ;vit. k deficient
baby;2) breast-fed newborns;3) fat malabsoprtion (bile duct
occlusin);4) prolong tx w/ antibiotics
Q2159:What is seen in the lab for vit K def?
increase PT;factor II
Q2160:What drug is a direct inhibitor of vit. K?
warfarin and coumadin
Q2161:Inheritance of CF?
Autosomal recessive
Q2162:Inheritance of albinism?
Autosomal recessive
Q2163:Inheritance of alpha-1 antitrypsin deficiency?
Autosomal recessive
Q2164:Inheritance of phenylketonuria?
Autosomal recessive
Q2165:Inheritance of thalassemias?
Autosomal recessive
Q2166:Inheritance of sickle cell anemia?
Autosomal recessive
Q2167:Inheritance of glycogen storage diseases?
Autosomal recessive
Q2168:Inheritance of mucopolysaccharidoses (EXCEPT
HUNTER'S)?
Autosomal recessive
Q2169:Inheritance of sphingolipidoses (EXCEPT FABRY'S)?
Autosomal recessive
Q2170:Inheritance of infant polycystic kidney disease?
Autosomal recessive
Q2171:Inheritance of hemochromatosis?
Autosomal recessive
Q2172:Inheritance of Fragile X syndrome?
X-linked recessive
Q2173:Inheritance of Duchenne's muscular dystrophy?
X-linked recessive
Q2174:Inheritance of Hemophilia A and B?
X-linked recessive
Q2175:Inheritance of Fabry's (a sphingolipidosis disease)?
X-linked recessive
Q2176:Inheritance of G6PD deficiency
X-linked recessive
Q2177:Inheritance of Hunter's syndrome (a
mucopolysaccharidosis disease)?
X-linked recessive
Q2178:Inheritance of OCULAR albinism?
X-linked recessive
Q2179:Inheritance of Lesch-Nyhan syndrome?
X-linked recessive
Q2180:Inheritance of Bruton's agammaglobulinemia?
X-linked recessive
Q2181:Inheritance of Wiskott-Aldrich syndrome?
X-linked recessive
Q2182:how many chromosomes and autosomes does each cell
have?;what is the genetic term?
46 chromosomes;22 pairs of autosomes;1 pair of sex
chromosomes;"Diploid"
Q2183:Definition;a chromosome number that is not a multiple
of 23 (the normal haploid number)
Aneuploidy
Q2184:(2) ways that a haploid can become aneuploidy
Nondisjunction;(ex - Down's);Anaphase Lag;(monosomy)
Q2185:what most commonly occurs w/ a polyploidy
fetus?;give an example of what polyploidy means
spontaneous abortion;Polyploidy = multiples of 23
chromosomes;ex) 69 chromosomes in patient
Q2186:Definition;two acrocentric chromosome are joined by
common centromere causing the joining of the long arms (and
possible loss of the short arms)
Robersonian Translocation
Q2187:Definition;the normal inactivation of one X
chromosome
Lyonization;(creation of Barr Body)
Q2188:How many Barr Bodies;1. XX;2. XY;3. XXXY
XX = 1 barr body;XY = NO barr bodies;XXXY = 2 barr
bodies
Q2189:Definition;when the cells in the body have a different
genetic make-up (such as random X inactivation in females)
Mosaicism
Q2190:Dx;large forehead; broad nasal bridge; epicanthal folds;
Brushfield spots; simian crease;genetic problem?
Down's syndrome;;(Trisomy 21)
Q2191:MCC of Down's syndrome;what is the other
cause?;what is the "Familial form"?
Nondisjunction;;(Robertsonian) Translocation;(familial form)
Q2192:MC heart defect w/ Down's syndrome;what is it due
to?
Septum primum-type ASD;due to: Endocardial Cushion
defect
Q2193:(3) MC complications of Down's syndrome
AAA;ASD;ALL;Alzheimers
Q2194:what is the maternal screening for Down's;1. Alpha-
fetoprotein;2. hCG;3. Unconjugated estriol
AFP = Low;hCG = High;E2 = Low
Q2195:Dx;severe mental retardation; microcephaly; wide-set
eyes; low birth weight; round face; unusual cry;genetic
problem?
Cri du chat;(deletion: 5p-)
Q2196:Dx;cardiac abnormalities; hypocalcemia; thymic
aplasia; abnormal facies; cleft palate;genetic problem?
DiGeorge syndrome;(also called Velocardialfacial
synd);(22q11 microdeletion);*signs = CATCH-22
Q2197:Dx;mental retardation; prominent occiput;
Micrognathia; Rocker-bottom feet; index finger overlaps 3rd
and 4th fingers; Congenital heart dz;genetic problem?
Edwards syndrome;(Trisomy 18 - nondisjunction);*18 =
Election = Edwards
Q2198:Dx;mental retardation; microcephaly; Microphthalmia;
cleft lip and palate; Polydactyly; rocker-bottom feet;genetic
problem?
Patau syndrome;(Trisomy 13 - nondisjunction);13 = Puberty
= Patau (= 13 fingers)
Q2199:Definition;disorder when there are at least two X-
chromosomes and one or more Y-chromosomes
Klinefelter syndrome
Q2200:Dx;male w/ Atrophic testes; Tall stature;
Gynecomastia; decreased testosterone; increased pituitary
gonadotropins; male infertility;genetic problem?
Klinefelter syndrome;(maternal meiotic nondisjunction);[Kline
felt her TAG him]
Q2201:Dx;Violent behavior; tall; severe Acne
XYY syndrome
Q2202:MCC of Primary Amenorrhea
Turner's syndrome
Q2203:Dx;Short stature; shield-like chest; Amenorrhea;
Webbed neck; Ovary replaced by Fibrous Streaks;genetic
problem?
Turner's syndrome;(XO w/o Barr bodies);[Turner WAS not
feminine]
Q2204:MC cardiac problem w/ Turner's syndrome
Coarctation of the aorta
Q2205:(2) common cardiac defects w/ 22q11 syndromes
(DiGeorge)
Truncus Arteriosus;Tetralogy of Fallot;[where all the T's
went to;]
Q2206:Dx;mental retardation; long face w/ large jaw; large
everted ears; Autism; Macro-orchidism;genetic problem?
Fragile X syndrome;(X-lined defect w/ CGG repeats);[big
testicles = X-rated]
Q2207:what is the underlying (biochemical) cause of Fragile X
syndrome?;what is unusual about this syndrome?
defect in Methylation of FMR1 gene;X-linked problem that
may show signs of retardation in male and female offspring
Q2208:Definition;severity of Dz worsens or age of onset of
dz is earlier in succeeding generations;(example)
Anticipation;(Huntingtons)
Q2209:Definition;hereditary disorders in which differing
phenotypes occur depending on whether an abnormal gene is
of maternal or paternal origin;(examples)
Genomic Imprinting;(Prader-Willi or Angelman)
Q2210:Dx;mental retardation; hypogonadism; hypotonia;
behavior problems; uncontrolled appetitie leading to obesity
and DM;genetic problem?
Prader-Willi syndrome;(5q11-13 deletion on father's
chromosome)
Q2211:Dx;mental retardation; ataxia; seizures; inappropriate
laughter;genetic problem?
Angelman syndrome;(5q11-13 deletion on mother's
chromosome)
Q2212:Definition;Not all individuals w/ mutant genotype
show mutant phenotype
Incomplete penetrance
Q2213:Definition;one gene has greater then one effect on the
individual's phenotype
Pleiotropy
Q2214:Definition;a heterozygote produces a nonfunctional
altered protein that also prevents the normal gene product
from functioning
Dominant Negative mutation;(exerts a Dominant effect)
Q2215:Definition;mutations at different loci can produce the
same phenotype;(example)
Locus Heterogenicity;(Albinism)
Q2216:Equation for Hardy-Weinberg population
genetics;Disease Prevalence
p^2 + 2pq + q^2 = 1
Q2217:Equation for Hardy-Weinberg population
genetics;Allele Prevalence
p+q=1
Q2218:Equation for Hardy-Weinberg population
genetics;Heterozygote Prevalence;(p and q on separate alleles)
HP = 2pq
Q2219:type of inheritance;often due to Structural defects
Autosomal Dominant
Q2220:type of inheritance;often due to Enzyme deficiencies
Autosomal Recessive
Q2221:Main sign of;X-linked Recessive
no male-to-male transmission
Q2222:Main sign of;X-linked Dominant
All females are affected by father
Q2223:Lysosomal storage Dz;peripheral neuropathy of hands
and feet; angiokeratomas; CV and renal
Dz;Enzyme?;inheritance?
Fabry's Dz;(alpha-Galactosidase A);X-recessive
Q2224:Lysosomal storage Dz;hepatosplenomegaly; aseptic
necrosis of femur; bone pain; unique macros;Enzyme?
Gaucher's Dz;(Glucocerebrosidase)
Q2225:Lysosomal storage Dz;progressive neurodegeneration;
hepatosplenomegaly; cherry-spot on macula;Enzyme?
Niemann-Pick;(Sphingomyelinase)
Q2226:Lysosomal storage Dz;progressive neurodegeneration;
developmental delay; cherry-spot macula; lysozymes w/
onion skin;Enzyme?
Tay-Sachs Dz;(Hexosaminidase A)
Q2227:Lysosomal storage Dz;peripheral neuropathy;
developmental delay; optic atrophy;Enzyme?
Krabbe's Dz;(beta-Galactosidase);[Krabs have small eyes]
Q2228:Lysosomal storage Dz;developmental delay;
gargoylism; airway obstruction; corneal clouding;Enzyme?
Hurler's syndrome;(Alpha-L-IDuronidase);[A Lit-ID in
Quasimoto caused him to HURL the GARGOYLE]
Q2229:Lysosomal storage Dz;mild developmental delay; mild
gargoylism; airway obstruction; aggressive
behavior;Enzyme?;inheritance?
Hunter's syndrome;(Iduronate Sulfatase);[Hunter's Aggressive
ID Shot the X];X-recessive
Q2230:Familial Dyslipidemia type;Inc Chylomicrons
only;pathology?
Type I;(hyperchylomiconemia);Lipoprotein Lipase
deficiency
Q2231:Familial Dyslipidemia type;Increased LDL only;(high
blood cholesterol);pathology?
Type IIa;(hypercholesterolemia);Low LDL receptors
Q2232:Familial Dyslipidemia type;Increased LDL and
VLDL;pathology?
Type IIb;(combined hyperlipidemia);Hepatic overproduction
of LDL
Q2233:Familial Dyslipidemia type;Increased IDL;
VLDL;pathology?
Type III;(dysbetalipoproteinemia);Altered Apo-E
Q2234:Familial Dyslipidemia type;Increased VLDL
only;(high blood TG);pathology?
Type IV;(hypertriglyceridemia);Hepatic overproduction of
VLDL
Q2235:Familial Dyslipidemia type;Increased VLDL;
chylomicrons;pathology?
Type V;(mixed hypertriglyceridemia);Inc production or Dec
clearance of VLDL and chylomicrons
Q2236:Autosomal Dominant Dz;cafe-au-lait spots; neural
tumors; pigmented iris hamartomas; scoliosis
Neurofibromatosis Type 1;(Von Recklinghausen Dz);(chrom
17)
Q2237:Autosomal Dominant Dz;bilateral acoustic neuromas;
optic pathway gliomas; juvenile cataracts
Neurofibromatosis Type 2;(chrom 22)
Q2238:Autosomal Dominant Dz;facial lesions;
hypopigmented "ash leaf spot" on skin; cortical and retinal
hamartomas; seizures; mental retardation
Tuberosus Sclerosis

Potrebbero piacerti anche